Pathphys Module

You might also like

Download as pdf or txt
Download as pdf or txt
You are on page 1of 102

TESTS FOR MODULE 1 COMPUTER CONTROL

SUBMODULE 1 “GENERAL NOSOLOGY”


General nosology. Pathophysiology general principles.
1. What is the modern definition of DISEASE? Disease is a 8. Which of the processes listed below is a TYP IC AL
disturbance of living activity of the organism under the effect PATHOLOGIC PROCESS?
of pathogenic factor … a. burn
a. with development of pathological process b. poisoning
b. with development of functional and structural changes in c. birth defect
the organism d. allergy
c. with loss of capacity for work e. tumor
d. with clinical features of disease 9. The knowledge about which period of disease is important
e. with disturbance of adaptation to environment conditions for prophylaxis of infectious diseases?
and ability for work a. incubation period
2. What is the modern definition of PATHOLOGICAL b. prodromal period
PROCESS? Pathological process is: c. manifestation of disease
a. sum of protective and pathological reactions of the d. outcomes of the disease
organism e. convalescence
b. reaction of organs and tissues to the pathological irritant 10. The patient complains of irritability, pain in bones and
c. inadequate reaction of organs and tissues to the muscles, loss of appetite, headache. No specific signs of the
pathological irritant disease are observed. Which period of disease is described?
d. changes in living activity of the organism disturbance of a. incubation period
adaptation to environment conditions and ability for work b. prodromal period
e. inadequate reaction of the organism to the adequate c. manifestation of disease
irritation d. outcomes of the disease
3. What is the modern definition of PATHOGENESIS? e. convalescence
Pathogenesis is: 11. From the list of ‘civilization diseases ’ choose the one
a. causes and conditions of disease ’s onset which doesn’t belong to this group:
b. the mechanism of onset, development, course and a. atherosclerosis
outcome of the disease b. diabetes
c. internal and external mechanisms of disease ’s onset c. pneumonia
d. conditions of onset, development, course and outcome of d. allergy
the disease e. stenocardia
e. reaction of organs and tissues to the pathological irritant 12. From the list of ‘civilization diseases ’ choose the one
4. What is the modern definition of PATHOLOGICAL which doesn’t belong to this group:
REACTION? Pathological reaction is: a. atherosclerosis
a. adequate reaction of the organism to the adequate b. burns disease
irritation c. diabetes
b. inadequate reaction of the organism to the inadequate d. allergy
irritation e. stenocardia
c. inadequate reaction of the organism to the adequate 13. Choose the example of etiologic therapy from the given:
irritation a. infectious disease treatment with antibiotics
d. adequate reaction of organs and tissues to the pathological b. allergic disease treatment with anti-histamine medicines
irritant c. diabetes mellitus treatment with insulin
e. internal and external reaction of the organism of disease d. arthritis treatment with immune-stimulating medicines
onset e. arthritis treatment with analgetics
5. What is the modern definition of ETIOLOGY? Etiology is 14. Choose the example of pathogenic therapy from the given:
a science about… a. infectious disease treatment with antibiotics
a. factors of internal and external environment that take part b. allergic disease treatment with anti-histamine medicines
in the onset of disease c. headache treatment with analgetics
b. internal and external causes of disease onset d. arthritis treatment with immune-stimulating medicines
c. conditions of the organism during the disease onset e. arthritis treatment with analgetics
d. causes and origins of disease ’s onset 15. Choose the example of substitutive therapy from the given:
e. main causes of the disease onset a. infectious disease treatment with antibiotics
6. Which of the examples listed below are b. allergic disease treatment with anti-histamine medicines
PATHOLOGICAL REACTIONS? c. diabetes mellitus treatment with insulin
a. fever d. arthritis treatment with immune-stimulating medicines
b. erythema on the skin after thermal influence e. arthritis treatment with analgetics
c. allergic reaction 16. Choose the example of symptomatic therapy from the
d. inflammatory reaction given:
e. dilation of the pupil to the light a. infectious disease treatment with antibiotics
7. Which of the processes listed below is a TYP IC AL b. allergic disease treatment with anti-histamine medicines
PATHOLOGIC PROCESS? c. diabetes mellitus treatment with insulin
a. inflammation d. arthritis treatment with immune-stimulating medicines
b. burn e. arthritis treatment with analgetics
c. poisoning 17. Which mechanism is forming general reactivity and
d. birth defect resistance of the organism?
e. acquired trauma a. activation of mononuclear phagocytes system
b. chain complement reactions e. pathological condition
c. activation of vegetative nervous system 25. The student has caught pneumonia after super-cooling at
d. immunological mechanisms the end of the winter. He had a nervous overstrain. Which was
e. activation of the respiratory system the reason of the disease?
18. Dosed physical loading was made to a patient with the a. Nervous overstrain
stenocardia of tension for studying of reserve ability of heart. b. Super-cooling
Which kind of pathophysiology method was used in this case? c. Insufficient feeding
a. instrumental research d. Hypovitaminosis
b. clinical experiment e. Pathogenic microorganism
c. clinical observation 26. Which type of resistance underlies the steadiness of human
d. functional test organism towards effect of both specific and wide ranges of
e. stationary observation non-specific damaging agents?
19. Experiment is the one of basic methods of a. individual resistance
pathophysiology. Choose the correct sequence of the stages of b. active resistance
making an experiment. c. cross-resistance
a. discussion of possible results => making => analysis of d. passive resistance
results => conclusions e. specific resistance
b. planning => carrying out => conclusions => analysis of 27. The patient arrived to the hospital from the smelting
results workshop in the condition of hyperthermia. Which is the direct
c. planning => carrying out => analysis of results => cause of consciousness loss during the heat stroke?
conclusions a. decrease of the brain blood supply
d. carrying out => analysis of results => planning => b. arterial pressure drop
conclusions c. increased water loss through sweating
e. discussion of possible results => carrying out => d. decrease of heart reject
conclusions => analysis of results e. dilatation of peripheral vessels
20. Many diseases develop on the certain stages. Choose the 28. The animal with the fever was injected with an antipyretic
typical sequence of the stages of development of many diseases. agent and a change of body temperature was observed. Which
a. Prodromal period => latent (incubative) period => of the following pathophysiological methods was used?
manifestation of disease => outcome of disease a. study of primary signs
b. manifestation of disease => prodromal period => outcome b. chronic experiment
of disease => latent (incubative) period c. modelling of pathological process
c. latent (incubative) period => prodromal period => d. experimental therapy
manifestation of disease => outcome of disease e. planning of the experiment
d. latent(incubative) period => period of hidden signs => 29. The electrodes were implanted into rabbit’s hypothalamic
prodromal period => manifestation of disease area. During several weeks hypotalamus’ activity was studied
e. latent period(incubative) => prodromal period => under the conditions of chronic inflammation. Which from the
outcome of disease following types of experiment is described?
21. Choose the disease that can be named “illness of a. acute experiment
civilization ”? b. chronic experiment
a. rheumatism c. physiological experiment
b. piles d. biological experiment
c. diabetes mellitus e. vivisection
d. hepatocirrhosis 30. The researcher has to investigate an effect of a profound
e. pancreatitis bleeding on the cardiovascular system function. Which from
22. Patient with stomach peptic ulcer disease after the the following types of experiment should be performed?
treatment felt himself better. Digestion was normalized, pains a. acute experiment
disappeared. However in a few weeks pains and heartburn b. chronic experiment
appeared again. How will you describe such flow of illness? c. physiological experiment
a. Period of remission d. biological experiment
b. Complication of illness e. vivisection
c. Relapse 31. When a 5-year-old child was brought home from the
d. Prodromal period kindergarten he presented with weakness, headache, body
e. Latent period temperature rise up to 37,5oC. What period of disease
23. The stenosis of pylorus was found at the X-ray develompent is the case?
examination of a patient with peptic ulcer disease. How can a. prodromal
you characterize the presence of stenosis of the pylorus in the b. latent
patient? c. incubative
a. Pathological condition d. recovery
b. Pathological process e. disease outcome
c. Disease 32. A man aged 49, who was 12 years ago sick with rheumatic
d. Pathological reaction myocarditis and endocarditis now has mitral valve
e. Compensatory reaction insufficiency. Studies have shown that inflammatory process is
24. Patient was admitted with a peptic ulcer disease of not present, cardiac output is sufficient. Describe the condition
stomach. He has been ill for 3 years. Now he complains of pain of the patient:
in epigastrium, heartburn, nausea, signs of blood in the stool. a. pathological state.
How can you describe such state of the patient? b. pathological reaction.
a. relapse c. pathological process.
b. complication d. model of pathological process.
c. remission e. compensatory response.
d. pathological reaction
Physical factors influence on human body (UV rays, ionizing radiation)
33. The ultraviolet radiation is used in medicine with the c. it will not change
different aims. Which mechanism from listed below underlies 43. Which medicines from the listed below can be
the positive therapeutic effect of ultraviolet radiation upon the photoreactive?
organism? a. sulfonamides
a. activation of medicine effects b. non-steroid anti-inflammatory drugs
b. melanin synthesis increasing in the skin c. barbiturates
c. activation of vitamin D synthesis d. vitamin B group
d. activation of cellular division e. all of them
e. activation of lipid peroxidation 44. The main portion of spectrum of solar radiation belongs to
34. Which of the following is the principal ultraviolet infra-red, visible and ultraviolet rays that have both positive
wavelength responsible for sunburn? and negative effects. Which causes and conditions lead to the
a. UVC (100-280 nm) development of sunstroke?
b. UVB (280-320 nm) a. effect of solar radiation upon the retina of the eye
c. UVA (320-400 nm) b. long-term effect of solar radiation upon the head
d. all of them c. combination of high temperature and solar radiation
e. none of them d. combination of solar radiation with high humidity
35. The UV radiation is causing both negative and positive e. in all cases listed above
influences on the human organism. Choose the sign of UV-rays 45. Patients with disturbance of porphyrinic exchange might
deficiency from the following: have accumulation of photosensitizers in the organism. How do
a. early ageing these substances influence on the organism?
b. reduction of skin elasticity a. increase sensitivity of the organism to antibodies
c. sunburns b. cause development of allergic reaction to the visible light
d. vitamin D deficiency c. increase sensitivity of retina photoreceptors to visible light
e. skin tumors development d. increases sensitivity of the organism to UV rays
36. The UV radiation is causing both negative and positive e. increase sensitivity of the organism to the allergens
influences on the human organism. Choose the sign of UV-rays 46. Which conditions may cause hyperthermia with extreme
overdose from the following: degree – heat stroke?
a. vitamin D deficiency a. intensive infra-red radiation
b. skin tumors development b. intensive ultraviolet radiation
c. immune deficiency c. ionizing radiation
d. disturbance of psychic activity d. normal temperature but high humidity of the air
e. increase of skin elasticity e. normal temperature but low wind speed
37. Each kind of radiant energy can cause both positive and 47. During running parallel experiment rats were exposed to
negative effect. Which tissue is the most sensitive to ultraviolet prolonged direct solar radiation in open chambers and in ones
radiation affection? covered by glass. Tumor development at hair uncovered places
a. epidermis of skin was marked in animals in open chambers. Which factor
b. red marrow influence is this phenomena connected with?
c. vascular epithelium a. sun heat
d. retina b. ultraviolet radiation
e. subcutaneous fat c. biologic carcinogens
38. Which diseases treatment can include irradiation with d. exogenous chemical carcinogens
ultra-violet rays? e. infrared radiation
a. heart diseases 48. Which is the definition of ultraviolet rays biodose for the
b. GIT diseases human?
c. blood diseases a. The least time, which is necessary for the appearance of
d. skin diseases erythema on a skin
e. eyes diseases b. The least distance, not causing the damage of epidermis
39. Which rays can be used in treatment of inflammatory c. Intensity of ultraviolet radiation in international units
processes? d. Amount of radiation, which is necessary for pigmentation
a. UV rays appearance on the skin
b. IR rays e. The most time of radiation which does not initiate burn of
c. UV rays and IR rays the skin
d. UV rays and X-rays 49. What is the mechanism of the selective effect of gamma-
e. IR rays and gamma rays radiation on the tumor cells that can be used in medical practice?
40. Which is the most dangerous removed consequence after a. Affection of tissues with high blood supply
ionizing radiation influence? b. Affection of tissues with high maintenance of water
a. malignant neoplasm c. Activation of the immune system
b. asthenia syndrome d. Affection of tissues with high mitotic rate
c. early aging e. Initiation of the mutations in tumor cells
d. bone marrow hypoplasia 50. What mechanism from listed below form the basis of
41. How will biodose of ultraviolet radiation change if the skin infrared radiation biological effect on an organism?
will be smeared with photosensitizing substance? a. Thermal effect
a. it will increase b. Photochemical effect
b. it will decrease c. Radiolysis of water
c. it will not change d. Sensitization of organism
42. How will biodose of ultraviolet radiation change if the skin e. Desensitization of organism
will be smeared with photoprotective substance? 51. What mechanism from listed below form the basis of
a. it will increase ionizing radiation biological effect on an organism?
b. it will decrease a. Thermal effect
b. Photochemical effect e. electronic radiation
c. Radiolysis of water 61. Choose the list in which the tissues of the organism are
d. Sensitization of organism situated due to their radiosensitivity in the decreasing order:
e. Desensitization of organism a. epithelial, hematopoietic, cartilages, bone, muscular,
52. What mechanism from listed below form the basis of nervous
ultraviolet radiation biological effect on an organism? b. nervous, hematopoietic, epithelial, cartilages, bone,
a. Thermal effect muscular
b. Photochemical effect c. bone, muscular, nervous, hematopoietic, epithelial
c. Radiolysis of water d. muscular, hematopoietic, bone, nervous, epithelial,
d. Sensitization of organism cartilages
e. Desensitization of organism e. hematopoietic, epithelial, cartilages, bone, muscular,
53. What does primary effect of ionizing radiation upon the nervous
organism consists in? 62. The preventive radioprotector was given to the worker of
a. water radiolysis nuclear power station. Which mechanism from listed below is
b. lipid peroxidation considered to be the main pathophysiological mechanism of
c. DNA mutations radioprotection?
d. Cells destruction a. prevention of tissue’s hypoxia
e. Intoxication b. activation of oxidation reactions
54. Each tissue of the organism suffers from ionizing radiation, c. inhibition of free radicals formation
but sensitivity to radiation (radiosensitivity) is different in d. increasing of tissue blood supply
different tissues. What does tissue’s radiosensitivity depend on? 63. Radioprotector was given to the patient who received a
a. content of water in tissues dangerous dose of ionizing radiation. Which mechanism is
b. proliferative activity of cells considered to be the main pathophysiological mechanisms of
c. specificity of tissue function radioprotection?
d. intensity of tissue innervation a. prevention of tissue hypoxia
e. partial pressure of oxygen in the blood b. increase of intensity of oxidation reactions
55. Pathogenetic mechanism that leads to the increased c. activation of the cell’s anti-oxidative system
frequency of malignant tumors appearance under effect of d. increase of tissue blood supply
ionizing radiation is: e. stimulation of the organism’s life support system
a. somatic mutations of the cells 64. Which blood count correctly reflects the period of disease
b. mutations of gametes in reproductive organs manifestation of acute radiation sickness bone marrow form
c. depression of immunological control and changes in DNA a. leukocytosis, lymphocytosis, monocytopenia
d. activation of lipid peroxidation in the cells b. lymphocytosis, anemia, thrombocytopenia
e. increase amount of carcinogenic substances c. erythrocytosis, leukopenia, thrombocytosis
56. Each kind of radiant energy can cause both positive and d. leukopenia, anemia, thrombocytopenia
negative effect. How is X-radiation applied in medicine? e. leukocytosis, lymphopenia, anemia
a. for treatment of oncologic diseases 65. In consequence of accident at nuclear power station
b. for endogenous probes radioactive products escape occurred. People, who were in
c. for ultrasound diagnosing zone of increased radiation got dose about 12-14 Gray. Which
d. for diagnosis of internal disease form of acute radiation sickness will appear in these people?
e. for NMR investigation (nuclear magnetic resonance) a. bone marrow
57. A 45-year-old wom an has been receiving radiation therapy b. intestinal
for breast cancer. Explain the effects of ionizing radiation in c. toxic
eradicating the tumor cells. It is successful because tumor cells: d. hemorrhagic
a. are rich with water e. cerebral
b. have high ATP store 66. Which dose of ionizing radiation causes typical bone
c. have high mitotic rate marrow form of acute radiation sickness?
d. have low metabolic activity a. less than 0,5 Gray
e. have high level of protein synthesis b. 1-10 Gray
58. Each kind of radiant energy can cause both positive and c. 10-20 Gray
negative effect. How is gamma radiation applied in medicine? d. 20-80 Gray
a. for prevention of rickets e. more than 80 Gray
b. for diagnosis of internal diseases 67. Which dose of ionizing radiation causes intestinal form of
c. for ultrasound diagnosing acute radiation sickness?
d. for NMR investigation (nuclear magnetic resonance) a. less than 0,5 Gray
e. for tumors treatment b. 1-10 Gray
59. Which is the main cause of increased frequency and c. 10-20 Gray
severity of infectious diseases during radiation sickness d. 20-80 Gray
development? e. more than 80 Gray
a. decrease of antibodies production 68. Which dose of ionizing radiation causes toxemia form of
b. depression of hemopoiesis acute radiation sickness?
c. depression of cellular immunity a. less than 0,5 Gray
d. anemia b. 1-10 Gray
e. disturbance of regeneration capability of epithelial tissues c. 10-20 Gray
60. Which kind of ionizing radiation is the most dangerous for d. 20-80 Gray
people due to the highest biological effect? e. more than 80 Gray
a. gamma radiation 69. Which dose of ionizing radiation causes cerebral form of
b. beta radiation acute radiation sickness?
c. alpha radiation a. less than 0,5 Gray
d. X-radiation b. 1-10 Gray
c. 10-20 Gray 73. In the patient with chronic radiation sickness the signs of
d. 20-80 Gray haemorrhagic syndrome are observed. Which factor is most
e. more than 80 Gray important in the pathogenesis of this syndrome?
70. A patient was admitted to hospital with complaints about a. thrombocytopenia
weakness, headache, increase of temperature, diarrhea which b. violation of the structure of vascular walls
have appeared after a single radiation exposure. Leukocytosis c. increased activity of fibrinolysis
with lymphopenia are found in a blood count. Which stage of d. increase anticoagulant system activity
radiation sickness does the patient have? e. decrease of clotting factors activity
a. latent period 74. A worker of the nuclear power plant took a dose of
b. period of primary reactions radiation – 5 Gr. He complains of headache, nausea, dizziness.
c. manifestation What kind of changes of blood count will take place in 10 hors
d. prodromal period after irradiation&
e. period of convalescence a. eosinophila
71. In the period of illness manifestation the patient had b. lymphocytosis
leukopenia, thrombocytopenia, autoinfection, autointoxication, c. leukopenia
bleeding, and fever. Which form of radiation sickness is this d. agranulocytosis
clinical picture characteristic for? e. neutrophilia
a. Bone marrow 75. A person was admitted to a hospital in 3 days after being
b. Intestinal exposed to a 3 Gr dose of ionizing radiation. What
c. Toxic physiological system will be damaged in this case
d. Cerebral a. digestive
e. Hemorrhagic b. cardiovascular
72. The mechanisms of protection and adaptation are activated c. nervous
at reversible cellular injury by ionizing radiation. Name the d. blood
reactions directed on renewal of changed intracellular e. endocrine
homeostasis in this case: 76. Ionizing radiation influence can cause many negative
a. activation of cell’s antioxidant system components consequences in human organism. Choose the form of acute
b. activation of calcium channels radiation sickness which may result in patient ’s recovery:
c. accumulation of sodium ions in the cells a. intestinal
d. inhibition of adenylate cyclase b. toxemia
e. hypertrophy of mitochondria c. cerebral
d. bone marrow
e. there can ’t be recovery from the radiation sickness

Chemical factors influence on human body


77. Choose the most correct definition of an abstinent c. yes, the usage of large dosages of drugs promote better
syndrome: adaptation of the organism to the environment
a. complex of psychoneurological disorders after stop of d. no, there are no endogenous narcotic substances in the
alcohol or toxic substance taking organism
b. complex of somatic, vegetative and psychoneurological e. no, endogenous narcotic substances are synthesized only
disorders after stop alcohol or toxic substance taking during pathological processes
c. complex of somatic and vegetative disorders after stop of 81. Does endogenous alcohol have physiological effect upon
taking alcohol the organism?
d. somatic, vegetative and psychoneurological reactions a. yes, it is natural anti-oxidant
during taking alcohol or toxic substance b. yes, the usage of large dosages of drugs promotes better
e. psychoneurological reaction on the alcohol or toxic adaptation of the organism to the environment
substance taking c. yes, it takes part in adaptation processes and motivation
78. Which stage of alcoholic and narcotic dependence is behavior
characterized by invincible attraction to the substance (alcohol, d. no, there is no endogenous alcohol in the organism
drug) consumption without the abstinent syndrome? e. no, endogenous alcohol is synthesized only during
a. physical dependence syndrome pathological processes
b. tolerance development 82. Which main pathologic mechanism causes cellular damage
c. sensitization development during activation of lipid peroxidation?
d. cumulative effect phenomena a. decrease of ATP-formation
e. mental dependence syndrome b. disturbance of RNA-synthesis
79. Which stage of alcoholic and narcotic dependence is c. release of proinflammatory mediators
characterized by manifestation of vegetative, somatic, psychic d. destruction of membranes
and neurological disorders after stopping the use of substance e. damage of MHC proteins
(alcohol, drug)? 83. Which pathogenic mechanism causes the development of
a. physical dependence syndrome abstinent syndrome during alcoholism?
b. mental dependence syndrome a. sensitization of nervous cells to the alcohol
c. tolerance development b. formation of the cells pathological tolerance in the
d. sensitization development organism
e. phenomenon of cumulative effect c. appearance of polyorganic functional insufficiency
80. Do endogenous narcotic substances have physiological d. decrease of activity of alcohol dehydrogenase in the liver
effect upon the organism? e. decrease of endogenous alcohol synthesis
a. yes, they are natural anti-oxidants 84. The patient developed abstinent syndrome as a result of
b. yes, they take part in adaptation processes and motivation narcotic usual dose absence. Which pathogenic mechanism
behavior causes the development of abstinent syndrome during drugs
addiction?
a. decrease of endogenous opiates synthesis a. lycopene
b. development of sensorialization of nervous cells towards b. carotin
alcohol c. tocopherol
c. formation of pathological tolerance of the cells in the d. ascorbic acid
organism e. glutathione peroxidase
d. appearance of multiple organ functional insufficiency 93. Each cell in the organism has anti-oxidation systems for
e. decrease of activity of enzymes that metabolize narcotic protection from effect of free radicals. Which of the substances
85. Free radicals constantly form in the healthy cell due to the listed below can be relate d to the class of enzyme anti-oxidants?
effect of external and internal causes. What is the physiological a. superoxide dismutase
role of free radicals in cell activity? b. licopene
a. are exclusively pathogenic factors c. carotin
b. take part in synthesis of biologically active substances d. tocopherol
c. take part in processes of tissue respiration e. ascorbic acid
d. take part in processes of cellular detoxication 94. Which type of necrosis develops after alkali application?
e. take part in development of inflammation a. coagulative
86. A patient suffering alcoholism revealed disorders in heart, b. colliquative
lungs, kidney and liver activity. Which stage of alcoholism is c. liquifactive
characterized by multiple organ failure? d. caseous
a. stage of psychic dependence e. fat necrosis
b. stage of physical dependence 95. Coagulation of which substances is the leading mechanism
c. terminal stage of coagulative necrosis development?
d. stage of desadaptation a. membranes
e. stage of initiation b. lipids
87. Which substance is responsible for the formation of c. proteins
multiple organ failure during alcoholism? d. DNA
a. formaldehyde e. RNA
b. ethanol 96. The effect of toxins upon the organism is determined with:
c. methanol a. toxin chemical structure
d. acetaldehyde b. dose of toxin
e. acetylsalicylic acid c. previous state of organism
88. Which method of pathogenic therapy is the most d. duration of toxic effect
appropriate to use during acute poisoning? e. all the items are true
a. desintoxication therapy 97. Which of the terms refers to the ability of toxins to cause
b. prescription of diuretics defects in a developing fetus?
c. blood transfusion a. teratogenicity
d. prescription of anticonvulsants b. mutagenicity
e. prescription of purgatives c. general toxicity
89. Free radicals activate lipid peroxidation in the organism d. specific toxicity
and cause cellular damage. Which factor promotes formation of e. cancerogenoicity
free radicals? 98. Choose the substance that causes exogenous intoxication:
a. hypovitaminosis D a. scatol
b. infra-red radiation b. keton bodies
c. lack of oxygen c. phenol
d. ultraviolet radiation d. indole
e. excess of CO2 (carbon dioxide) e. methanol
90. Free radicals that form throughout our life have both 99. Alcoholics and drugs addicts have the skin inflammatory
positive and negative effects. What is the role of antioxidants in diseases more frequently than healthy people do. Which
the cells? disturbance form the basis of this fact?
a. increase the formation of free oxygen radicals a. dysfunction of a histohematic barrier
b. increase the oxygen consumption in the cell b. phagocytosis impeirment
c. inhibit the formation of free oxygen radicals c. metabolic disorder
d. decrease the oxygen consumption in the cell d. liver pathology
e. increase ATP -formation e. alteration in the skin pH
91. Each cell in the organism has antioxidative systems for 100. In an experiment, a tissue preparation is subjected to
protection from free radicals pathogenic effect. Which of the oxidant stress. There are increased numbers of free radicals
substances from listed below can be related to the class of non- generated within the cells and increase in number. Generation
enzyme anti-oxidants? of which of the following substances within these cells is most
a. catalase likely to reduce the number of free radicals?
b. superoxide dismutase a. glutathione peroxidase
c. ceruloplasmin b. catalase
d. beta-carotene c. hydrogen peroxide
e. ferritin d. NADPH oxidase
92. Each cell in the organism has anti-oxidation systems for e. myeloperoxidase
protection from effect of free radicals. Which of the substances
listed below can be related to the class of enzyme anti-oxidants?

The influence of heredity on human pathology development


101. Hereditary disease – phenylketonuria was found in child in 110. Is hemophilia possible in girls?
the maternity hospital. Which reason causes the occurrence of a. no, because women are only bearers of pathological gene
hereditary diseases? b. yes, if father is sick on haemophilia
a. changes of chromosomal quantity c. no, because pathological gene is inherited only with Y-
b. qualitative and quantitative changes of genes chromosome
c. effect of surrounding environment negative factors d. yes, if 2 X-chromosomes with pathological gene are
d. hereditary pathology of the closest relatives inherited
102. Congenital disease – non-closure of fetal oval foramen e. yes, if mother is sick on haemophilia
was revealed in child in the maternity hospital. Which reason 111. Can congenital pathology be inherited?
from listed below can cause the occurrence of congenital a. yes, in all cases
diseases? b. no, not in any case
a. rhesus conflict of mother and fetus c. possible in case of additional effect of specific
b. quantitative changes of genetic information environmental factor
c. effect of teratogenic factors d. possible in case of additional effect of complex of
d. congenital pathology of the closest relatives environmental factors
e. qualitative and quantitative changes of genetic information e. yes, if mutation is in sex cells
103. In which period of life the clinical signs of hereditary 112. Can chromosomal diseases be inherited from parents?
diseases can appear? a. yes, if the disease is not accompanied by pathology of
a. right after birth sexual glands
b. in the period of puberty b. yes, in all cases
c. in the middle age c. yes, if both parents are sick
d. at any age d. yes, if the disease is not accompanied by mental
e. in senile age deficiency
104. Hereditary diseases may be connected with disorders of e. no, not in any case
chromosomes and genes structure or quality. Which of the 113. One of the most common hereditary disorders is color
diseases from listed below can be related to chromosomal blindness (daltonism). Define the type of this disorder.
diseases? a. single autosomal dominant gene disorder
a. Klinefelter ’s syndrome b. single autosomal recessive gene disorder
b. stomach ulcer c. autosomal disorder
c. essential hypertension d. sex chromosome disorder
d. syndactylism e. single X-linked gene disorder
e. phenylketonuria 114. It is known that phylketonuria is characterized by a lack of
105. Hereditary diseases may be connected with disorders of phenylalanine hydroxylase. Which method of its treatment is
chromosomes and genes structure or quality. Which of the most commonly used?
diseases from listed below can be related to gene diseases? a. elimination of phenylalanine from the diet
a. diabetes mellitus b. injections of phenylalanine hydroxylase
b. haemophilia c. oral administration of phenylalanine hydroxylase
c. X-trisomy syndrome d. correction of the patient ’s life style
d. Klinefelter ’s syndrome e. surgical treatment
e. stomach ulcer 115. Diagnosis of hereditary diseases includes detection of Barr
106. Which chromosomal disease can be observed both in men bodies in the cells. Which number of Barr bodies will be
and wom en equally? detected in the person with Turner syndrome?
a. Down’s syndrome a. zero
b. XO syndrome b. one
c. Kleinfelter ’s syndrome c. two
d. X-chromosome trisomy syndrome d. three
e. Phenylketonuria e. four
107. Which changes of karyotype are typical for Down’s 116. Diagnosis of hereditary diseases includes detection of Barr
syndrome? bodies in the cells. Which number of Barr bodies will be
a. trisomia 21 detected in the wom an with Down’s syndrome?
b. X-chromosome trisomia a. zero
c. trisomia 13 b. one
d. loss of X-chromosome c. two
e. inversion of chromosome of 21st pair d. three
108. Which changes of karyotype are typical for Kleinfelter ’s e. four
syndrome? 117. A doctor consulted a woman with defects of physical and
a. 44X0 sexual development. Microscopy of mucosal cells from the pral
b. 44XXY cavity didn’t reveal sex chromatin in the nuclei. What kind of
c. 44XXX chromosomal pathology is it typical for?
d. 44Y0 a. Turner syndrom e
e. 44XY b. Down’s syndrom e
109. The dominant pathological gene linked with X- c. Kleinfelter syndrome
chromosome was found in wom an during chromosomal set d. Trisomy of X chromosome
mapping. In Which conditions this pathological gene can cause e. Patau syndrome
diseases in children depending on sex? 118. What method of examination will be helpful in diagnosing
a. in all children not depending on sex Turner syndrome and Kleinfelter?
b. only in sons a. genealogical
c. in all daughters b. statistical
d. will lead to fetal death in prenatal period c. identification of sex chromatin
e. in 50% of daughters d. dermatoglyphic
e. biochemical a. Gametopathy
119. The frequency of heterozygotes on phenulketonuria in the b. Blastopathy
Ukraine population is 3%. What method of examination is c. Embryopathy
used for early diagnosis of phenylketonuria in newborns? d. Non-specific fetopathy
a. genealogical e. Specific fetopathy
b. statistical 125. Examination of a 12 year old boy with developmental lag
c. identification of sex chromatin revealed achondroplasia: disproportional constitution with
d. dermatoglyphic evident shortening of upper and lower limbs as a result of
e. biochemical growth disorder of epiphyseal cartilages of long tubal bones.
120. Multifactorial diseases are characterized by extreme This disease is:
variety of clinical forms and individual manifestations, and by a. Inherited, autosomal dominant
high dissemination in population, for example: atherosclerosis, b. Inherited, autosomal recessive
arterial hypertension, and diabetes mellitus. What kind of c. Inherited, sex-linked
inheritance forms the basis for predisposition of these diseases: d. Inherited. polygenic
a. polygenic e. Congenital
b. dominant 126. In genetic counseling the couple was consulted. A man
c. recessive suffers from insulin-dependent diabetes mellitus, and the
d. X-linked wom an is healthy. What is the probability of insulin-dependent
e. incomplete domination diabetes in children of this couple?
121. A couple came for medical genetic counseling. The man a. more than in the population
has hemophilia, the woman is healthy and there were no cases b. the same as in the population
of hemophilia in her family. What is the risk of having a sick c. 100%
child in this family? d. 50%
a. 0 e. 25%
b. 100 % 127. A few months after birth a child developed symptoms of
c. 75% the CNS disorders. The skin and hair became lighter. The
d. 50% solution of 5% iron trichloroacetate added to fresh urine gives it
e. 25% green coloring. What kind of hereditary disorder is
122. A wom an who was sick with rubella during the pregnancy characterized by this test?
gave birth to a deaf child with hare lip and cleft palate. This a. tyrosinosis
congenital defect is an example of: b. alcaptonuria
a. Phenocopy c. fructozuria
b. Edward’s syndrome d. phenylketonuria
c. Genocopy e. albinism
d. Patau’s syndrome 128. Examination of a child with oligophrenia revealed an
e. Down’s syndrome increased level of phenylalanine and phenylpiruvate in blood
123. A wom an who was infected with toxoplasmosis during the plasma. The reaction of urine with trichloroacetic iron is
pregnancy has a child with multiple congenital defects.This is a positive. The deficiency of which enzyme synthesis may
result of: manifest in such way?
a. Teratogenesis a. tyrosinase
b. Cancerogenesis b. dopamine hydroxylase
c. Biological mutation c. phenylalanine hydroxylase
d. Chemical mutation d. homogentisic acid oxydase
e. Genes recombination e. pyruvate oxydase
124. A couple had a child with Down's disease. Mother is 42
years old. This disease is most probably caused by the
following pathology of prenatal development:

Cell injury mechanisms


129. During the investigation of cells from biopsy material b. swelling of cells
chemical analysis of blood in patient with diabetes mellitus the c. changed cell color
signs of cellular damage described below were found. Which of d. appearance of cytoplasmic enzymes in blood
these signs can be related to morphologic signs of cellular e. accumulation of calcium in the cell
damage? 132. Which signs of cellular damage can be related to
a. disturbance of cellular division functional?
b. change of cells ’ color a. damage of nuclear membrane
c. increase of permeability of cellular membrane for proteins b. destruction of structure of mitochondrions
d. appearance of cytoplasmic enzymes in blood c. swelling of cell
e. increase of suboxidized substances in blood d. change of colour
130. The definition “increased number of the cells in a tissue or e. disturbance of cellular division
organ ” refers to… 133. Which is the most typical morphological sign of cell death
a. hypertrophy by apoptosis?
b. hyperplasia a. condensation of nucleus and cytoplasm
c. hyperactivity b. presence of inflammatory reaction
d. hypereactivity c. compensatory increase of DNA-synthesis
131. During the investigation of cells from biopsy material d. swelling of mitochondrions
chemical analysis of blood in patient with diabetes mellitus the e. increase of cell ’s size
signs of cellular damage described below were found. Which of 134. Chose the example of specific cell injury from listed below:
these signs can be related to functional signs of cellular damage? a. myocardial ischemia
a. change of cellular organoids quantity and structure b. intestinal epithelial injury due to bacterial toxins
c. immune hemolysis of RBC 144. Which tissue cells are most sensitive to hypoxic injury?
d. liver cell injury due to chemicals a. skeletal muscles
e. skin damage due to mechanical trauma b. smooth muscles
135. Ischemic heart disease develops in the patients due to c. myocardial cells
hypoxic injury of myocardial cell. But even when the d. brain cells
bloodflow is restored to the site of ischemia, the degree of e. liver cells
myocardial fiber injury may increase. Which factor is playing 145. Give the correct definition of apoptosis. Apoptosis is…
the leading role in reperfusion injury? a. a process of virus infected cells killing
a. cytoskeletal intermediate filament loss b. a programmed cell death
b. decreased intracellular pH from anaerobic glycolysis c. a death of the cell after injuring factor influence
c. increase in toxic oxygen radicals d. a cell’s death as a result of enzymes action
d. mitochondrial swelling e. an irreversible cell injury
e. nuclear chromatin clumping and decreased protein 146. People who have had a heart attack may experience
synthesis additional damage once blood flow has been restored, a
136. Disturbance of which process is primary observed in phenomenon referred to reperfusion injury. Which blood cells
hypoxic injury: from listed below take active part in reperfusion injury
a. detachment of ribosomes from EPR development?
b. reduction of intracellular pH a. lymphocytes
c. oxidative phosphorilation by mitochondria b. eosinophils
d. sodium pump activity c. neutrophils
e. activation of glycolysis d. erythrocytes
137. Which factor directly causes the decrease of intracellular e. thrombocytes
pH in the case of hypoxic injury? 147. Patient was made blood biochemical test in order to
a. detachment of ribosomes from EPR confirm hepatitis. Increased level of alanine transaminase (ALT)
b. decreased oxidative phosphorilation by mitochondria and aspartate transaminase (AST) was found. It has been
c. failure of sodium pump defined as a functional sign of hepatic cells injury. Which from
d. activation of glycolysis the listed may be the reason of it?
e. increased membranes permeability a. cell’s membrane damage
138. Which is calcium role in hypoxic cell injury? b. damage to plasma membrane sodium pump
a. detachment of ribosomes from EPR c. depletion of ATP store in the cell
b. disturbance of cells aerobic respiration d. nonperoxidative mitochondrial injury
c. disturbance of sodium pump e. disturbance of cells aerobic respiration
d. activation of glycolysis 148. Cells may adapt to external and internal stimuli by
e. activation of intracellular enzymes undergoing changes in their size, number and type. What
139. Reperfusion injury is developed mostly due to massive happens to other kidney when one is damaged? It undergoes…:
inflow to the cell of: a. hypertrophy
a. calcium b. atrophy
b. sodium c. hyperplasia
c. potassium d. metaplasia
d. aminoacids e. dysplasia
e. enzymes 149. Cells may adapt to external and internal stimuli by
140. Choose the effect which IS NOT directly caused by free undergoing changes in their size, number and type. What
radicals: happens to muscles of extremities that have been encased in
a. lipid peroxidation of membranes plaster casts? The muscles undergo …:
b. nonperoxidative mitochondrial injury a. hypertrophy
c. disturbance of cells aerobic respiration b. atrophy
d. DNA lesions c. hyperplasia
e. cross-linking of proteins d. metaplasia
141. Which mechanism of cellular adaptation is provided with e. dysplasia
anti-oxidants action? 150. A 30-year-old man sustained a fracture of his leg 2 months
a. compensation of energy metabolism disturbance ago. The leg had been encased in a cast, which was just
b. protection of cells membranes removed. The patient is amazed at the degree to which the
c. compensation of water-ion disbalance muscles in his leg have shrunk. Which is the reason of it?
d. repair of cell genome a. inadequate nutrition
e. lowering of cell’s functional activity b. loss of innervation
142. Which factors determine the type of cell’s response to c. loss of endocrine stimulation
injuring stimuli? d. decreased workload
a. kind of injuring factor e. diminished blood supply
b. injuring factor severity and time of duration 151. Every day, blood cells in our body become senescent and
c. prior state of the cell die without producing signs of inflammation, and yet, massive
d. type of the affected cell injury or destruction of tissue, such as occurs with a heart
e. all is correct attack, produces significant signs of inflammation. Why it
143. Which process distinguishes irreversible hypoxic injury happens?
from reversible one? a. due to necrosis of heart muscle
a. inability to reverse mitochondrial dysfunction b. due to apoptosis of heart muscle
b. damage to plasma membrane sodium pump c. due to atrophy of heart muscle
c. inability to re-start protein synthesis d. due to swelling of heart muscle
d. extremely low pH e. due to disturbances in calcium metabolism
e. depletion of ATP store in the cell
Disturbance of immune system function. Immunodeficiency.
152. Which is the main task of immune system during human 162. Preventive vaccination with weakened microorganisms
life-time? causes the production of antibodies against these microbes.
a. protection of the organism from pathogenic effects of Which cells are considered to be antibody-producing cells of
surrounding environment the immune system?
b. providing the genetic homogeneity of the organism a. T-lymphocytes
c. increasing of organism resistance to negative factors b. macrophages
influence c. NK-cells
d. creation of favorable conditions for the living cells d. B-lymphocytes
e. antimicrobial activity e. plasmocytes
153. Which class of immunoglobulins appears first during the 163. Which of the substances from listed below are antibodies
immune response to infectious antigens? in the organism?
a. Ig class A a. globulins of the plasma
b. Ig class E b. albumins of the plasma
c. Ig class G c. buffer systems
d. Ig class D d. lipoprotein systems
e. Ig class M e. plasma fibrinogen
154. Which class of immunoglobulins form the greater part 164. Cellular and humoral factors take part in mechanisms of
(70 – 80%) of immunoglobulins of normal blood serum? innate immunity. Which of the substances from listed below
a. Ig class A can be considered a humoral factor of specific immunity?
b. Ig class E a. complement
c. Ig class G b. immunoglobulin M
d. Ig class D c. interleukin-1
e. Ig class M d. interpheron
155. Which cells are regulating the rate of immunoglobulins e. factor of tumor necrosis
synthesis during the secondary immune response? 165. During the patient examination the signs of primary
a. plasmatic cells immunodeficienc y were found. Name the organ of immune
b. B-cells cells formation?
c. NK cells a. in thymus
d. T suppressors b. in bone marrow
e. T killers c. in bursa of Fabricius
156. Which possible quantity of antigens is B-cell receptor d. in spleen
affine to? e. in lymphatic nodes
a. all known antigens 166. During the patient examination the high Ig-G amount was
b. only one antigen found. Where are immunoglobulins synthesized in the humans?
c. group of similar antigens a. in the bone marrow
d. all protein antigens b. in thymus
e. all cellular antigens c. in lymph nodes
157. Which immune reactions do natural killer cells take part in? d. in bursa of Fabricius
a. innate immunity e. in spleen
b. cellular immunity 167. Blood analysis of patient showed signs of HIV infection
c. humoral immunity (human immunodeficiency virus). Affection of which immune
d. immune tolerance cells is typical for AIDS?
e. adaptive immunity a. T-killers
158. Which cell use “respiratory burst” with active oxygen b. T-helpers
radicals formation? c. B-lymphocytes
a. monocytes d. macrophages
b. basophils e. neutrophils
c. eosinophils 168. The immunization of population with different vaccines is
d. natural killers carried out with preventive purposes. It is followed by
e. neutrophils formation of primary immune response. Which is the longest
159. Each immunoglobulin molecule consists of…. term for immune memory remaining after the immunization?
a. 1 heavy and 1 light chains a. 1 – 5 years
b. 2 heavy and 2 light chains b. 10 – 20 weeks
c. 1 heavy and 2 light chains c. all life
d. 2 heavy and 1 light chains d. several days
160. Which cytokine from listed below can provide systemic e. till 1 year
(endocrine) effect? 169. The immune system gives responses to antigen stimulation
a. lymphotoxin by 3 specific allergic reactions, humoral and cellular immunity
b. alpha interferon responses and immunological tolerance. In which situation
c. gamma interferon from listed below can the immunological tolerance develop?
d. interleukin 8 a. if to inject excessive dose of antigen
e. interleukin 1 b. if to irradiate the animal before injection of antigen
161. Which cells quantity is markedly decreased in Bruton’s c. if to inject antigen and cytostatic agent at the same time
disease? d. if to inject antimacrophage antibodies before injecting
a. pre-B cells antigen
b. plasmatic cells e. if to hold desensitization before injection of antigen
c. natural killers 170. Cooperative interaction of which immune cells is
d. T helpers necessary for effective formation of primary humoral immune
e. T suppressors response ?
a. macrophages, T-lymphocytes, B-lymphocytes
b. T-lymphocytes, plasmatic cells, B-lymphocytes b. passive resistance
c. macrophages, plasmatic cells, B-lymphocytes c. cross resistance
d. mast cells, plasmatic cells, B-lymphocytes d. sensitization
e. B-lymphocytes, macrophages, plasmatic cells e. reactivity
171. Cooperative interaction of which immune cells is 180. Which substances from listed below can organize immune
necessary for effective formation of primary cellular immune response and provide the destruction of alien cells?
response? a. Ig A
a. T-lymphocytes, plasmatic cells b. cytokines
b. macrophages, T-lymphocytes c. opsonins
c. macrophages, plasmatic cells d. plasma albumens
d. mast cells, plasmatic cells e. plasma globulines
e. T-lymphocytes, B-lymphocytes 181. The patients with organs transplants usually receive
172. Which is the main distinction of secondary immune hormone injections in order to prevent transplant rejection.
response from primary immune response? Which hormones from the listed should be prescribed for
a. higher level of antibody ’s amount transplant rejection prevention and why?
b. elongated latent period after antigen administration a. mineralocorticoids – to increase inflammatory reaction in
c. slower rise of antibody concentration transplant
d. activation of IgE synthesis b. glucocorticoids – to inhibit immune system activity
e. shortened latent period after antigen administration c. glucocorticoids – to decrease vessels permeability
173. The main task of immune system is to support the genetic d. mineralocorticoids – to inhibit immune system activity
homogeneity of the organism. That is removal of own defective e. growth hormone – to enhance anabolic processes in the
and senescent cells. How do T-lymphocytes destroy own virus- cells
infected cells in the organism? 182. Examination of a child who frequently suffers from
a. by necrobiosis infectious diseases revealed that IgG concentration in blood
b. by dystrophy serum was 10 times less than normal, IgA and IgM
c. by necrosis concentration was also significantly reduced. Analysis showed
d. by apoptosis also lack of B-lymphocytes and plasmocytes. What disease are
e. by inflammation these symptoms typical for?
174. Which type of immune reaction is absent in the patients a. Bruton's disease
with DiGeorge syndrome? b. Swiss-type agammaglobulinemia
a. humoral c. Dysimmunoglobulinemia
b. reaginic d. Louis-Bar syndrome
c. cell-mediated e. Di George syndrome
d. imunocomplex 183. In the 12-year-old boy who often has viral and bacterial
e. cytotoxic infections, eczematous lesions are observed. Laboratory data
175. Decreased blood level of which substance usually show the decrease of T-lymphocytes and IgM and normal
accompanies DiGeorge syndrome development? content of IgA and IgG. Which kind of the immune system
a. sodium pathology is observed in the patient?
b. potassium a. combined Immunodeficiency
c. aminoacids b. hypoplasia of the thymus
d. enzymes c. Bruton hypogammaglobulinemia
e. calcium d. Turner syndrom e
176. A 5-year –old girl has teleangiectasies on the skin and e. hereditary deficiency of the complement system
conjunctiva. IgA is absent; the amount of T-lymphocytes is 184. In many cases of organ transplantation in 10 days the
reduced. What type of immunodeficiency is present? reaction of transplant rejection is observed. Which types of
a. Di George syndrome blood cells are playing the active role in this process?
b. Loui-Barr syndrome a. macrophages
c. secondary immunodeficiency syndrome b. erythrocytes
d. Bruton’s disease c. platelets
e. Chediack-Higasy syndrome d. eosinophils
177. Which drugs from listed below can cause acquired e. basophils
immunodeficiency development 185. A child 2 years old is with severe bacterial infections, the
a. sulfonamides lack of B-lymphocytes and plasmatic cells is diagnosed with
b. antibiotics Bruton disease. What changes in serum immunoglobulins
c. antipyretics content will be observed in this clinical situtation?
d. local anesthetics a. reduced Ig A, IgM
e. enzymes b. increase IgA, IgM
178. Which is the role of MHC molecules class 1 on the surface c. decrease in IgD, IgE
of body cells? d. increased IgD, IgE
a. to identify self and non-self cells e. without changes
b. to identify cells with high mitotic activity 186. The child, after laboratory investigation of immune system
c. it is a hormone specific receptor was diagnosed with primary immunodeficiency syndrome.
d. to activate protein ’s synthesis in the cells Which of these reasons may lead to development of primary
e. it is a marker of apoptosis immunodeficiency in an infant?
179. It is known that increase in resistance to hypoxia usually is a. hereditary disorders in the immune system
accompanied with the increased activity of the immune system. b. violations in the process of embryonic development
Choose the correct definition to the situation when the c. disturbances in mother's metabolism during pregnancy
increased resistance to one factor is accompanied with the d. violations of reactivity and resistance of the organism
increased resistance to other factors: e. toxic damage of B-lymphocytes
a. active resistance
Disturbance of immune system function. Allergy
187. Which is the common name for allergic reactions that have e. on the membrane of B-lymphocyte
hereditary predisposition and are observed in the whole 196. The patient R. was diagnosed an autoimmune hemolytic
families? anemia. The complex antigen - antibody fixates during
a. reaginic development of 2nd type of allergic reaction…:
b. anaphylactic a. on the mast cells membrane
c. cytotoxic b. on the B-lymphocytes membrane
d. stimulating c. on the surface of macrophages
e. atopic d. on the surface of target cells
188. Which stage of allergic reactions includes synthesis of e. in the blood plasma
specific antibodies by plasmatic cells? 197. The patient B. passed a tuberculin skin test (Mantoux
a. hidden reaction). In 24 hours the red spot formed in the place of
b. immunologic injection (d=2 cm). The complex antigene - antibody fixates
c. biochemical during development of 4th type of allergic reaction…:
d. stage of clinical manifestation a. in the blood plasma
e. outcome b. on the mast cells membrane
189. Which stage of allergic reactions are allergy mediators c. on the surface of macrophages
synthesized at? d. on the membrane of B-lymphocyte
a. hidden e. it doesn’t form
b. immunologic 198. The same mechanisms of immune system are used by the
c. sensitization organism to develop immune and allergic reactions. Which is
d. biochemical the main difference of allergic reactions from immune reactions?
e. stage of clinical manifestation a. amount of antigen
190. Which class of immunoglobulins is called “reaginic b. peculiarities of antigen structure
antibodies”? c. ways of entering the organism by antigen
a. IgA d. development of tissue damage
b. IgD e. hereditary predisposition
c. IgG 199. The formation of blood circulating immune complexes
d. IgE (antigen – antibody) is typical for allergic reactions of type:
e. IgM a. 1st type
191. Which class of immunoglobulins takes part in b. 2nd type
development of 1 type allergic reactions? c. 4th type
a. IgA and IgG d. 5th type
b. IgE e. 3rd type
c. IgG and M 200. In response to antigene stimulation, the immune system is
d. IgE and IgG able to produce several; types of immunoglobulins: IgA, IgM,
e. IgM and IgA IgG, IgE. Which is a distinguishing feature of IgE?
192. Which type of immunoglobulins take part in development a. circulation in the blood
of 3 type allergic reactions? b. ability to activate the complement
a. IgA and IgG c. fixation on the receptors of mast cells
b. IgG and M d. fixation on T-killers
c. IgE e. ability for binding antigen
d. IgE and IgG 201. The patient was given a conduction anesthesia with
e. IgM and IgA Novocain before extraction of tooth. After injection edema and
193. The patient C developed anaphylactic shock after injection hyperemia around the shot region, itch of skin, general
of antitetanus serum. Which cells produce the mediators during weakness, low arterial pressure developed. Define the
the classic variant of anaphylaxis? complication described.
a. T-lymphocytes a. idiosyncrasy
b. neutrophils b. fever
c. mast cells c. drug dependence
d. B-lymphocytes d. allergy
e. Eosinophils e. inflammation
194. The patient R. suffers bronchial asthma. The attacks rise in 202. Immune complex reactions in the organism proceed with
the period of ragweed blossoming. Point out the biologically participation of IgM and IgG. Which is the most typical
active substances that play the main pathogenic role in peculiarity of IgM and IgG?
development of atopic allergic reaction. a. fixation on the receptors of mast cells
a. complement b. fixation on tissue macrophages
b. properdin c. fixation on T-killers
c. interleukine d. ability to interact with antigens
d. histamine e. ability to activate the complement
e. interferon 203. Which disease is developing with the 2nd type of allergic
195. The patient A. developed the signs of anaphylactic shock reactions?
after a wasp sting. The complex antigene - antibody fixates a. atopic bronchial asthma
during development of 1st type of allergic reaction…: b. contact dermatitis
a. in the plasma of blood c. para-allergic reactions
b. on the surface of macrophages d. autoimmune hemolytic anemia
c. on the membrane of T-lymphocytes that carry receptor e. jaundice of the newborns
IgE (FcE) 204. It ’s been established that allergic reactions of 2nd type
d. on the membrane of mast cells play the main role in erythrocytes hemolysis in the patient with
hemolytic anemia. Which mediators cause cytotoxic effect in 213. Prevention of which type allergic reactions can be
this type of allergy? provided with the hyposensitization ( specific immunotherapy)?
a. histamine a. 1st type
b. lymphokines b. 2nd type
c. factor of lymphocytes blast transformation c. 3rd type
d. serotonin d. 4th type
e. activated components of the complement e. 5th type
205. Allergic reactions of 3rd type are typical for pathogenesis 214. Patient was injected with Novocaine solution in order to
of: provide local anesthesia during tooth extraction. In 1 minute
a. atopic bronchial asthma after injection the patient turned pale and unconscious. ABP –
b. glomerulonephritis 90/60 mmHg, heat rate – 128 bpm. Name the condition that has
c. contact dermatitis developed in the patient:
d. anaphylactic shock a. Acute heart failure
e. tuberculin reaction b. Orthostatic collapse
206. In which condition of relations between reactivity and c. Anaphylactic shock
resistance we can say: reactivity is increased and resistance is d. Bronchial asthma attack
decreased? e. A. pulmonalis embolism
a. immunity 215. Allergic reactions are characterized by increased
b. immunological tolerance sensitivity to various substances. Which substance amount can
c. immune deficit prove the presence of hyperreactivity state in the patient?
d. allergy a. histamine
e. immunological memory b. IgE
207. Type 5 allergic reactions are known as “stimulating c. IgA
reactions ”. Choose the example of disease provided with d. prostoglandins
stimulating allergic reactions e. complement
a. hay fever 216. Patient complains of the skin rashes, which appear after
b. Arthus reaction cooling the skin (cold water or cold air exposure). After
c. Quincke’s disease returning from the street in the winter the opened areas of the
d. autoimmune thyroiditis skin turn red (hyperemia) and itches appear. Which substance
e. autoimmune hemolytic anemia from the listed can cause such clinical symptoms?
208. Which immune cells contribute to tissue injury in a. histamine
immunocomplex allergic reactions (type 3 allergic reactions)? b. IgE
a. B –cell c. IgM and IgG
b. plasmatic cells d. prostoglandins
c. T-cells e. complement
d. mast cells 217. A nurse complains of the rashes on the skin of the hands.
e. phagocytes Her usual work is to do injection of antibiotics and other
209. Exposure to extremes of temperature (hot and cold) and medicines to the patients. The symptoms of skin irritation
ionizing radiation in some cases may result in pseudoallergy usually disappear after summer vacations. In 7-10 days after
development. Which is the distinctive feature of pseudo-allergy? working with the solutions of medicines the symptoms of
a. absence of immunological stage rashes appear again. Which type of allergic reaction is possibly
b. absence of biochemical stage causing her allergic disease?
c. presence in patients with immune deficiency a. 1st type
d. dependence from the type of allergen b. 2nd type
e. independence from the dose of allergen c. 3rd type
210. It is known that many cases of “food allergy” are not real d. 4th type
allergy, but pseudoallergy. Which is the distinctive feature of e. 5th type
pseudo-allergy? 218. In the patient autoimmune hemolytic anemia is diagnosed;
a. absence of biochemical stage the pathogenetic mechanism of which is cytotoxic type of
b. absence of clinical manifestation allergic reaction. To which antigens antibodies are synthesized
c. presence in patients with immune deficiency in this clinical situation?
d. dependence from the type of allergen a. modified receptor of RBC membranes
e. dependence from the dose of allergen b. mast cell receptors
211. Which mechanisms are considered to be general c. hormones
mechanisms of autoimmune pathology? d. foreign proteins
a. direct antibody mediated effects e. foreign cells
b. T cell mediated effects 219. Leukopenia followed the treatment with pyramidon.
c. immune complex mediated effects Antileukocytic antibodies were found in patient ’s blood. What
d. all of listed type of allergic reaction takes place in this situation?
e. none of listed a. anaphylactic
212. Development of sympathetic ophtalmia (inflammation of a b. cytotoxic
healthy eye after the development of inflammatory process in c. stimulating
an injured eye) is considered to be an autoimmune disorder. d. cell-mediated
Which mechanism from listed below initiates autoimmune e. immunocomplex
process? 220. An injection of a large dose of antibodies to the basal
a. over expression of MHC 2 class molecules membrane of glomeruli of the expewrimental animal kidney
b. altering of self-antigens led to the development of acute glomerulonephritis. What type
c. similarity of antigens of allergic reaction takes place in this situation?
d. damage of physiological isolation a. anaphylactic
e. primary changes of immune system. b. cytotoxic
c. stimulating e. immunocomplex
d. cell-mediated

SUBMODULE 2 “TYPICAL PATHOLOGICAL PROCESSES”


Disturbances of peripheral bloodflow and microcirculation
221. Patient G. has inflammatory infiltration on the right 227. Patient A., 60 years old, with varicose veins, has
forearm. Skin around inflammatory center is red, hot and cyanosis, decreased skin temperature, single petechiae, pasting.
painful. What kind of blood circulation disorder does patient What kind of hemodynamic disorder does the patient have?
have? a. compression ischemia
a. venous hyperemia b. obstructive ischemia
b. sludge syndrome c. thrombus embolism
c. ischemia d. respiratory acidosis
d. arterial hyperemia e. venous hyperemia
e. embolism 228. Student X during passing exams couldn ’t answer the
222. What are the main reasons of thrombus formation? questions correctly. He turned red, felt fever and diffidence.
a. vessels endothelium infiltration, BAS influence, What type of arterial hyperemia student had?
anti-coagulation system activation a. neuroparalytic
b. coagulation deficiency, platelets activation, b. neurotonic
hemodilution c. metabolic
c. anticoagulation system activation, BAS influence, d. pathologic
blood vessel injury e. postischemic
d. hemoconcentration, turbulent blood flow, vessel 229. Patient P., 40 years old, with opened fracture of the hip
wall injury suddenly developed arteria pulmonalis embolism. Choose the
e. vessel wall injury, coagulation system activation, possible kind of embolism.
bloodflow speed lowering a. thrombus-embolus
223. Diabetes mellitus patient has venous hyperemia. What b. air
signs of this pathologic process does the patient have? c. tissue
a. hyperemia, local temperature increase, tissues d. fat
edema e. foreign body
b. paleness, local temperature decrease, tissue 230. What signs are describing arterial hyperemia
resistance decrease development?
c. cyanosis, local temperature decrease, tissues edema a. arterioles constriction
d. hyperemia, local temperature increase, tissue b. venules dilation
volume increase c. erythrocytes aggregation in capillaries
e. cyanosis, local temperature increase, tissue volume d. functioning capillaries quantity increase
decrease e. new capillaries growth
224. Patient M. has stenocardia attack in the result of 231. Sportsman had arterial hyperemia features in humeral
myocardium ischemia after physical load. Choose correct region after intensive training. What mechanism could lead to
ischemia definition. working arterial hyperemia development?
a. disparity between tissues blood supply and a. neurotonic and metabolic
requirement b. humoral and metabolic
b. erythrocytes quantity decrease in circulation blood c. neurotonic
c. local vasodilatation under BAS influence d. neuroparalytic
d. oxygen partial pressure decrease in blood under e. metabolic
physical load 232. What are the negative manifestations during arterial
e. inconsistency between blood oxygen capacity and hyperemia in organs and tissues?
oxygen tissues need a. lymph formation with edema development
225. Patient 65 years old with diabetes mellitus, diabetic b. ruptures of microcirculation vessels walls
angiopathy diagnosis has respiratory insufficiency, blood c. cells and tissues hypertrophy and hyperplasia
spitting, pronounced cyanosis, consciousness losing, and mouth d. immunity depression
spume. Patient was dead in two minutes. Arteria pulmonalis e. cells function activity increase
thrombus embolism was found under cutting. In what blood 233. Which factors are especially important in development
system region is primary thrombus formation possible? of phlebothrombosis (vein thrombosis)?
a. leg veins a. Paradoxical inhibition of blood coagulation factors
b. vena portae system X and XIII
c. mesenteric arteries b. Growth of granulation tissue
d. pulmonal veins c. Activation of plasminogen
e. leg arteries d. Endothelial injury and stasis
226. Skin-diver, 10 minutes after lifting from depth of 15 m, e. Abnormally thick glycocalyx coat and DIC
developed such clinical features: pain in articulations and (disseminated intravascular coagulation)
muscles, transitory consciousness losing. Choose correct 234. Ischemia is characterized with the following signs
pathogenic mechanism. EXC EPT:
a. gas mixture poisoning a. paleness
b. lung vessels thrombosis b. redness
c. gas embolism c. local hypothermia
d. cerebral vessels spasm d. pain
e. respiratory acidosis e. slowing of the bloodflow
235. Pain in the leg at walking, cyanosis and edema of calf c. Embolism
appeared in a patient with varicosity. His foot is cold. What d. Ischemia
kind of disturbances of regional blood flow appeared in the e. Disorders of lymph outflow
patient? 243. Instantaneous death of pilots occurs under
a. Angiospastic ischemia depressurization of airplane at the altitude of 19 km. What is
b. Ischemic stasis the reason of the death?
c. Compressive ischemia a. Multiple gas embolism
d. Venous (passive) hyperemia b. Hemorrhage to the brain
e. Obstructive ischemia c. Gas embolism of cerebral
236. Edema and cyanosis of low extremities appear in a food d. Bleeding
shop assistant at the end of workday. What is the main factor of e. Paralysis of respiratory center
the edema development in the patient? 244. Gas embolism developed in a diver who was lifted to
a. Dilatation of resistant vessels the surface very fast. It results from fast change:
b. Orthostatic increase of venous pressure a. From increased atmospheric pressure to normal
c. Increase of number of functional capillaries b. From normal atmospheric pressure to increased
d. Increase of collateral blood flow c. From normal atmospheric pressure to decreased
e. Increase of tissue drainage d. From decreased atmospheric pressure to normal
237. Redness and increase in volume of affected place of 245. Examination of the lower limbs of a 40-year-old patient
tissue and increase in local temperature were observed in a with coronary artery disease and vascular disease of lower
patient with burn of thigh. Which pathological process do limbs (obliterating endarteritis) revealed skin pallor and
indicated symptoms correspond to? dystrophy, local temperature decrease, pain. The patient is
a. Arterial hyperemia likely to have the following disorder of the peripheral blood
b. Venous hyperemia circulation:
c. Thrombosis a. Obstruction ischemia
d. Ischemia b. Compression ischemia
e. Stasis c. Angiospastic ischemia
238. Patient’s arm was put in plaster cast on account of d. Venous hyperaemia
humeral bone fracture. Swelling, cyanosis and decrease of the e. Arterial hyperaemia
temperature of the traumatized arm appeared next day. What 246. A female patient consulted a doctor about leg pain
kind of disturbances of regional blood flow appeared in the edema of feet and shins that arises usually after working in the
patient? evening. Clinical examination: leg skin is cyanotic, cold to the
a. Thrombosis touch. What type of peripheral circulation disorder does the
b. Venous hyperemia patient have?
c. Ischemia a. Venous hyperemia
d. Embolism b. Arterial hyperemia
e. Arterial hyperemia c. Ischaemia
239. One of the most dangerous points in myocardial d. Stasis
infarction pathogenesis is enlargement of the zone of necrosis, e. Thrombosis
dystrophy and ischemia. Increase in myocardial oxygen 247. Upper neck node of sympathetic trunk was removed
consumption play important role in development of indicated from the rabbit on experiment. Reddening and increased
processes. Which substances contribute to this process? temperature of the skin of head is observed. What form of
a. Chloride ion peripheral blood circulation disorder developed in the
b. Cholesterol rabbit?
c. Catecholamines a. Neuroparalytic arterial hyperemia
d. Acetylcholine b. Neurotonic arterial hyperemia
e. Adenosine c. Metabolic arterial hyperemia
240. After surgical removing of coronary artery occlusion in d. Venous hyperemia
a patient with ischemic heart disease, secondary injury of e. Stasis
myocardium develop (reperfusion syndrome) characterized by 248. A 42 year old wom an with neuralgia of trigeminal nerve
necrobiotic changes in the focus of previous ischemia. This complains of recurrent reddening and sensation of heat in the
complication results from: right part of her face and neck, hypersensitivity of the skin.
a. Accumulation of hydrogen ions Which type of arterial hyperemia is causing these symptoms?
b. Deficiency of potassium ions a. Metabolic
c. Deficiency of adenosine triphosphate b. Neurotonic
d. Excessive accumulation of calcium ions c. Neuroparalytic
e. Deficiency of creatinephosphate d. Functional
241. A 57-year-old man complains of heart pain that has e. Reactive
developed after prolonged negative emotions. An emergency 249. A patient with obliterating endarteriitis underwent
doctor diagnosed ischemic heart disease manifesting by ganglionary sympathectomy of femoral artery. the positive
stenocardia. What kind of ischemia is the most probable? therapeutic effect of this operation is related to development of
a. Compressive arterial hyperemia of the lower limbs. Which type of arterial
b. Obliterative hyperemia will develop in the patient after operation?
c. Angiospastic a. Metabolic
d. Obturative b. Neurotonic
242. The theory exists that atherosclerosis plays an important c. Neuroparalytic
role in periodontitis development, affecting vessels of gums. d. Functional
What kind of regional blood flow disturbances develops under e. Reactive
atherosclerosis of vessels? 250. Student was confused to answer exam questions, felt
a. Active hyperemia redness and heat of facial skin. What type of arterial hyperemia
b. Passive hyperemia can developed in this situation?
a. Metabolic veins. What type of circulation pathology can develop in this
b. Neurotonic situation?
c. Neuroparalytic a. thromboembolism of mesenterial vessels
d. Functional b. thromboembolism of coronary vessels
e. Reactive c. thromboembolism of the brain vessels
251. After physical activity, a patient with the d. thromboembolism of the pulmonary artery
thrombophlebitis of the lower extremities suddenly developed e. thromboembolism of the portal vein
dyspnea, acute pain in the chest, cyanosis, swelling of cervical

Inflammation
252. Increased erythrocyte sedimentation rate which seen in b. Increased metabolism level during all inflammatory stages
acute inflammation is due to c. Increased catabolism in early inflammatory stage
a. increased lipoproteins d. Increased anabolism in early inflammatory stage
b. increased serum albumins e. Decreased anabolism in final stages of inflammation
c. decreased serum immunoglobulins 261. Patient B., 32 years old, complaints about dry cough,
d. presence of C reactive protein. myalgia, articulations ache, appetite loss, headache. Body
e. increased serum immunoglobulins temperature is 40 C, blood count: leukocytosis, increased
253. Which sign from the given belongs to systemic signs of erythrocytes sedimentation rate. Which inflammatory
inflammation? mediators can cause these symptoms:
a. swelling a. catecholamines
b. pain b. neutrophil’s proteins
c. leukocytosis c. prostaglandins
d. heat d. kinins
e. redness e. interleukins
254. Which of the given local sign of inflammation is 262. Arachidonic acids metabolites (prostoglandins,
developed due to vascular leakage? leukotriens) are known as potent mediators of inflammation.
a. Calor (heat) Which enzyme will release arachidonic acid from cell
b. Rubor (redness) membrane lipids.
c. Dolor (pain) a. Cyclooxygenase
d. Tumor (swelling) b. Lipoxygenase
e. Functio laesa (loss of function) c. Adenylate cyclase
255. Which of the given signs is NOT defined as systemic d. Phospholipase
sign of inflammation? e. Myeloperoxidase
a. peripheral blood leukocytosis 263. Which inflammatory mediator is known to be normally
b. decrease of erythrocytes sedimentation rate sequestered in intracellular granules?
c. fever a. Histamine
d. increase of blood proteins level (globulins) b. Prostaglandin E2
e. increase of cateholamins and corticosteroids c. Complement
256. Which event is primary in inflammatory pathogenesis? d. Interleukin
a. disorders of blood circulation e. Bradykinine
b. phagocytosis 264. Which substances released from activated neutrophiles
c. tissues acidosis development and macrophages may contribute to tissue damage during
d. increased vessels permeability inflammation?
e. cells damage a. Free oxygen radicals
257. Which factor can directly cause secondary alteration? b. Platelet activating factors
a. kinines c. Endothelial growth factors
b. lysosomal enzymes d. Interleukine 6
c. lymphokines e. Gamma interferon
d. fibrinogen 265. Which influence results in arterial hyperemia
e. cytokines development under inflammation?
258. Patient B., 32 years old, complaints about dry cough, a. histamine secretion
myalgia, articulations ache, appetite loss, headache. Body b. compression of vessels by the exudate
temperature is 40 C, blood count: leukocytosis, increased ESR. c. vessel wall elasticity decrease
What is primary alteration mechanism under viral infections? d. endothelium swelling
a. cell genetic program realization violation e. blood viscosity increase
b. cell membranes damage 266. The main reason for the rapid onset of vasodilation after
c. cell energy supply violation tissue injury is
d. cell receptors damage a. release of histamine from mast cells.
e. cell lysosome destruction b. neural reflexes.
259. Fusion of lysosomes with the developing phagocytic c. release of leukotrienes.
vacuole before the latter has completely formed leads to: d. release of prostaglandins from mast cells.
a. formation of multinucleated giant cells. e. activation of complement
b. loss of all lysosomal activity. 267. The main reason for the rapid onset of arterioles spasm
c. regurgitation of lysosomal contents and tissue injury. after tissue injury is
d. none of the above a. release of histamine from mast cells
e. decreased hydrolase activity b. neural reflex
260. Which of the statements from listed below is correctly c. release of leukotrienes
describing metabolic changes in the site of inflammation? d. release of prostaglandins from mast cells
a. Decreased metabolism level during all inflammatory e. activation of complement
stages
268. Which mechanism is the most important in B. stasis, margination, and emigration.
inflammatory exudate formation? C. margination, emigration, and chemotaxis.
a. prostaglandin’s synthesis D. adhesion, margination, and emigration.
b. tissue basophils degranulation E. emigration, chemotaxis, and phagocytosis.
c. lysosomal enzymes release 277. Patient K., 28 years old. Diagnosis: pleuritis. In pleural
d. leukocytes migration punctate the quantity of neutrophils is high, some neutrophiles
e. increased vessels permeability include intact microbe cells inside. Define the state of
269. The patient V., has painful vesicles, surrounded with phagocytosis in this case.
hyperemia zone with bright liquid, as a result of sunburn. a. phagocytosis activation
Which mechanism is the leading one in inflammatory b. phagocytosis inhibition
exudation process? c. incomplete phagocytosis
a. increased tissue colloid and osmotic pressure d. immune phagocytosis
b. prostaglandin’s synthesis 278. What substances can act as opsonins (activate
c. decreased tissue proteins level phagocytosis).
d. increased lysosomal enzymes amount a. specific antibodies and oxygen
e. leukocytes migration from vessels b. leukotrienes and prostaglandins.
270. Leakage of fluid out of blood vessels during acute c. fibrinopeptides and C3
inflammation is due to d. prostaglandins and C3b
a. increased vascular permeability, hydrostatic and tissue e. specific antibodies and C3
osmotic pressure 279. Monocytes and activated macrophages are
b. decreased tissue osmotic pressure a. rarely found in chronic inflammation.
c. increased vascular permeability with decreased osmotic b. closely related, in that macrophages can be derived from
pressure monocytes.
d. increased hydrostatic pressure and permeability. c. derived from different precursor cells.
e. increased vascular permeability and hydrostatic pressure d. found only in acute inflammation.
with decreased tissue osmotic pressure e. equivalent cells.
271. Choose the negative consequence of exudate formation 280. The function of the enzymes of the activated
in the inflammation development from the given: macrophage (particularly in the lysosomes) is to:
a. transport of plasma-derived inflammatory mediators a. digest foreign material.
b. transport of antibodies b. remain in storage until the next chronic inflammatory
c. elimination of toxins and metabolites from the vessels of event.
inflammatory site; c. synthesize new lysosomes.
d. localization of the agent which caused inflammation. d. process antigen for lymphocytes.
e. squeezing of tissues and organs with the exudates e. assist in the synthesis of collagen
272. Patient G., 32 years old. Diagnosis: acute peritonitis. 281. Patient P., complained about fever, breast pain, which
Muddy yellow liquor with pH 3.0 was got after abdominal increases under deep breathing. In order to definite diagnosis
cavity puncture. Which cells should be predominately found in pleural cavity puncture was made and 20 ml of light liquid with
inflammatory exudate of the patient with acute inflammation? 3-5% protein amount was got. Which kind of inflammation
a. macrophages does the patient have?
b. monocytes a. fibrinous
c. lymphocytes b. purulent
d. neutrophiles c. serous
e. eosinophiles d. putrescent
273. Patient P., 45 years old, during last year had e. hemorrhagic
pyelonephritis exacerbation three times. Which leucocytes are 282. Patient F., with acute inflammation of appendix was
predominately found in inflammation center in chronic operated in a surgical department. The abdominal cavity
inflammation? contained exudate with dark yellow color. Microscopic analysis
a. neutrophiles and adipose cells revealed big amount of microorganisms, neutrophiles,
b. neutrophiles and fibroblasts monocytes and purulent bodies. Which kind of inflammation
c. adipose cells and lymphocytes does the patient have?
d. eosinophiles and macrophages a. fibrinous
e. monocytes and lymphocytes b. purulent
274. Choose the sequence of leucocytes migration towards c. serous
inflammation cite: d. putrescent
a. monocytes – neutrophils – lymphocytes e. hemorrhagic
b. neutrophils – monocytes - lymphocytes 283. Patient K., 28 years old, has quickly healed wound
c. lymphocytes – neutrophils - monocytes without scar formation after furuncle cutting. Point out cells,
d. neutrophils – lymphocytes - monocytes which play important role in proliferation process:
e. lymphocytes – eosinophils - neutrophils a. neutrophiles
275. During inflammation, leukocytes emigrate from blood b. eosinophils
vessels by means of c. fibroblasts
a. natural holes in vessel walls. d. lymphocytes
b. directional active migration. e. monocytes
c. ruptures in the vessel wall. 284. What cells secrete intercellular matrix components in a
d. random active migration. healing wound?
e. passive pressure mediated mechanisms. a. Macrophages
276. The three steps involved in movement of leukocytes b. Polymorphonuclear leukocytes
from blood vessels into the extravascular space during acute c. Multinucleated giant cells
inflammation are d. Endothelial cells
A. adhesion, margination, and chemotaxis. e. Fibroblasts
285. The process of new capillary growth in granulation 295. Patient S., with rheumatoid arthritis was prescribed
tissue during wound healing is called glucocorticoids. What is the main mechanism of
a. recanalization. glucocorticoids anti-inflammatory action?
b. endothelialization. a. inhibition of histamine secretion
c. diapedesis. b. microcirculation improvement
d. angiogenesis. c. secondary proliferation decrease
e. hemogenesis d. immune system activity depression
286. Extensive or massive wounds heal by e. tissue acidosis prevention
a. necrosis of margins. 296. Inflammatory process development was studied in
b. neoplastic transformation. experiment on rats. Inflammation was caused with 0,1%
c. secondary intention. formalin solution subcutaneous injection. Which hormones can
d. primary intention. be used to strengthen inflammation under modeling?
e. ulcer formation. a. mineralocorticoids
287. Patient K., 28 years old, has quickly healed wound b. female sexual hormones
without scar formation after furuncle cutting. How can you c. glucocorticoids
name this process? d. male sexual hormones
a. necrosis of margins. e. neurohypophisis hormones
b. neoplastic transformation. 297. Patient S., 56 years old, in postoperative scar region one
c. secondary intention. could find granulation tissue intensive growth. In order to
d. primary intention. inhibit inflammation proliferative stage the patient was
e. ulcer formation. prescribed glucocorticoid treatment. What mechanism of
288. The strength of a healed wound depends fundamentally proliferation processes is inhibited by glucocorticoids?
upon a. macrophages proliferation activation
a. how many multinucleated giant cells develop in the b. fibroblasts proliferation inhibition
disrupted tissue c. collagen resorption stimulation by eosinophils
b. the size of the wound d. collagen fibers synthesis increase
c. whether the wound healed by first or second intention. e. collagenases activation
d. whether granulation tissue formed in the wound 298. It is know that inflammation is characterized with a
e. the amount and nature of the collagen produced. series of microcirculation alterations. What is the first response
289. Scar tissue is of arterioles to injury?
a. nonfunctional collagenous and fibrotic tissue a. vasoconstriction
b. functional tissue that follows wound healing b. vasodilation
c. regenerated tissue formed in the area of injury c. redness
d. fibrinogen which has entrapped phagocytes and neurons d. edema
290. The main human defense mechanisms against injury are e. hyperemia
a. inflammation and wandering phagocytes. 299. Leukocytes are taking active part in inflammatory
b. cell proliferation, wandering phagocytes, and process. What is the name of the phenomenon where WBC's
inflammation. marginate and become attached to the edge of the endothelium?
c. cell proliferation and immunity. a. cementing
d. wandering phagocytes and immunity. b. pavementing
e. inflammation and immunity c. margination
291. The inflammatory response d. adhesion
a. prevents blood from the injured tissue e. rolling
b. rises body temperature to prevent spreading of infection 300. Leukocytes are taking active part in inflammatory
c. prevents formation of abscesses process. They can move from the bloodstream to the site of
d. minimizes injury and promote healing inflammation. Active movement of neutrophils along a
e. has only negative consequences concentration gradient is known as what?
292. What disease is an example of an autoimmune disease a. passive diffusion
that leads to chronic inflammation. b. chemotaxis
a. Herpes pneumonitis c. facilitated diffusion
b. Chronic pyelonephritis d. chemotactic diffusion
c. Silicosis e. adhesion
d. Rheumatoid arthritis 301. Leukocytes are taking active part in inflammatory
e. Asbestosis process be neutralizing bacteria and clearing the cell’s debris
293. Substances or conditions that typically lead to or cause from the site of inflammation. The process by which the
chronic inflammation include all of the following EXCEPT cytoplasm of the polymorphonuclear leucocytes surrounds the
a. foreign bodies. bacteria and encloses it into an invagination of the cell
b. highly virulent bacteria such as Staphylococcus aureus. membrane is known as what?
c. smoldering infections. a. phagolysosome
d. factors that lead to autoimmune reactions. b. phagolysis
e. inert, inhaled particles. c. phagolum
294. Inflammation development was studied after skin septic d. phagocytosis
damage in experiment on rabbits. Which hormones have anti- e. phagophobia
inflammatory action? 302. Inflammation is characterized by increased vessels
a. thyroid hormones permeability, increase of their fluid dynamic blood pressure.
b. catecholamines Increase of the osmotic and oncotic pressure is present in the
c. mineralocorticoids intercellular fluid. What kind of edema will appear in this case?
d. glucocorticoids a. hydrodynamic
e. neurohypophisis hormones b. colloid-osmotic
c. lymphogenic
d. membranogenic a. epinephrine
e. mixed b. cortisone
303. Necrosis focus appeared in the area of hyperemia and c. insulun
skin edema in few hours after burn. What is the main d. thyroid hormone
mechanism that causes destructive process in the inflammation e. vasopressin
area? 306. Modeling of inflammation on the intestine mesentery of
a. primary alteration a frog revealed peripheral orientation of leukocytes and their
b. secondary alteration migration through the vascular wall. Which factor from the
c. emigration of lymphocytes given detemines this process?
d. diapedesis of erythrocytes a. decrease of oncotic pressure in the vessels
e. proliferation of fibroblasts b. increase of oncotic pressure in the site of inflammation
304. In patient who had undergone trauma of the knee and c. increase of chemotactic substances in the site of
subsequent hemorrhagic bursitis on it in 3 months the limited inflammation
amount of movements in the joint is observed due to scar d. increase of hydrostatic pressure in the vessels
formation. Which inflammatory event is related for this e. decrease of hydrostatic pressure in the vessels
complication development? 307. An experimental model of inflammation with abscess
a. primary alteration formation was provided on laboratory animal. Then a lethal
b. secondary alteration dose of tetanin was injected into the abscess cavity, but the
c. violation of microcirculation animal didn’t die. How can you explain the absence of animal
d. exudation lethal outcome in this case?
e. proliferation a. activation of antibodies synthesis
305. A 6-year-old child had hyperergic inflammation of the b. formation of a barrier around the site of inflammation
upper respiratory tract. There was a threat of serious respiratory c. stimulation of leukopoiesis
disorder so the doctor had to use anti-inflammatory hormone d. intensification of vascularization in the site of inflammation
for the immediate therapy. Which from the given hormones has e. activation of phagocytosis the site of inflammation
anti-inflammatory action?

Thermoregulation pathology
308. Which primary mechanism onset fever reaction under 313. Patient M., 52 years old with bronchial asthma, was
inflammation? treated with glucocorticoids and fever reaction appeared as a
a. C-reactive protein formation in inflammation center result of postinjective abscess. Patient had subfebrile
b. increased leucocytes quantity temperature, which didn’t correspond to severity of
c. phagocytosis activation inflammatory process. Why did patient have low fever reaction?
d. humoral immune response activation a. inhibited endogen pyrogens production
e. long secondary alteration b. violation of heat loss through lungs
309. Primary bacterial pyrogens are starting fever onset. c. inflammatory barrier formation in injection place
What bacteria component can be the primary pyrogen? d. violation of heat-producing mechanisms
a. Membrane proteins e. thermoregulation center inhibition
b. Membrane lipids 314. How can you estimate thermoregulation of the patient
c. Lysosomal content with fever 1nd stage?
d. Lipopolysaccharides a. Thermoregulation is normal
e. Bacterial cell nucleus b. Heat loss is prevailing
310. Pyrogens are known to be the cause of the fever onset. c. Heat production is prevailing
What from the following may be the source of the secondary d. Heat production is restricted
pyrogens? e. Heat loss is normal
a. Bacteria 315. How can you estimate thermoregulation of the patient
b. Necrotic tissue with fever 2nd stage?
c. Platelets a. Thermoregulation is normal
d. Macrophages b. Heat loss is prevailing
e. Erythrocytes c. Heat production is prevailing
311. What mechanism of temperature increase is the earliest d. Heat production is restricted
one in the first stage of fever development? e. Heat loss is decreased
a. tachycardia 316. The patient with acute bronchitis had body temperature
b. increase of basal metabolism rate yesterday up to 38.50C week ago and now it decreases up to
c. shivering 37.00. What mechanism is the main in the 3rd stage of fever?
d. skin vessels constriction a. increased heat production
e. sweat secretion decrease b. shivering
312. Patient I., 24 years old, soldier. Diagnosis: double lobar c. peripheral vessels dilatation
pneumonia. Temperature was increased up to 370C. Blood d. diuresis increase
count: anemia, leucopenia with marked granulocytopenia, e. breathing frequency increase
increased erythrocytes sedimentation rate. Patient worked with 317. In the 3rd fever stage heat loss is prevailing. Heat loss is
myelotoxic poison. What is the cause of fever reaction absence accompanied by the following processes except:
under inflammation? a. radiation
a. hemic hypoxia development b. evaporation
b. inflammatory barrier formation c. conduction
c. thermoregulation center damage with toxic d. convection
substance e. fluctuation of metabolism
d. leucopenia with granulocytopenia 318. Overload of what functional system may be fatal to the
e. heat-producing mechanisms violation patient who has critical decrease of the temperature in the 3rd
fever stage?
a. nervous 326. The patient with secondary syphilis was prescribed
b. respiratory pyrotherapy in order to increase immune defense against
c. circulatory infectious agent. What means are used for this purpose?
d. endocrine a. hot baths
e. digestive b. hot drinks
319. The child with gastroenteritis, 10 years old, had fever c. cauterization of the specific sites of the body
reaction with 380C body temperature; it lasted 1 week and was d. injection of bacterial pyrogen
accompanied with 3 kg weight lost and mouth acetone smell e. using of local warming with infrared rays
appearance. What mechanism could lead to child’s weight lost? 327. Patient with cardiac valve defect was given
a. disturbances in digestion hypothermia during operation on open heart in order to nervous
b. increased contractive thermogenesis tissue hypoxic damage decrease. What hypothermia effect was
c. pancreatic cells damage used?
d. lost of appetite under fever a. tissues oxygen need decrease
e. lipolysis activation b. decrease of heart contraction frequency
320. Patient S., 27 years old ill with flu. Body temperature is c. arterial pressure decrease
about 39 - 380C. The patient has a variety of complaints d. nervous cells activity inhibition
connected wit disturbances of many organs and systems e. cellular membranes stability increase
functions. Which physiological system doesn’t directly take 328. Experiment was performed for the purpose
part in fever development, but it is affected due to pathogenic of alcohol role research in hypothermia mechanisms. 2 rats
features of the fever? were put in camera with ice for 3 hours. First rat was given
a. nervous alcohol in the middle dose of intoxication through stomach.
b. respiratory First rat hypothermia developed earlier. What thermoregulation
c. circulatory mechanism does alcohol lead to violation?
d. endocrine a. heat irradiation increase
e. digestive b. heat-producing decrease
321. At what fever pathogenesis stage do antipyretic drugs c. thermoregulation center neurons activity inhibition
(aspirin, paracetamol, indometacin) influence on? d. retractive thermogenesis inhibition
a. decrease receptors sensitivity e. thermoreceptors sensitivity decrease
b. inactivate cyclooxygenase 329. Fever can be divided into several types due to level
c. break temperature increase mechanisms of body temperature increase in the temperature standing stage.
d. inhibit phospholipase A2 activity The patient ’s temperature is 38,7 0 C. Which type of fever does
e. destruct prostaglandin E this temperature index correspond to?
322. Patient D., 27 years old, had body temperature increase a. Subfebrile temperature
up to 380C during week. He was treated with glucocorticoids b. Febrile temperature
for the purpose of temperature decrease. What mechanism of c. Pyretic temperature
glucocorticoids influence can cause anti-fever effect? d. Hyperpyretic temperature
a. Heat producing decrease 330. Fever can be divided into several types due to level
b. phagocytes activity decrease of body temperature increase in the temperature standing stage.
c. heat loss increase Which type of fever usually accompanies chronic local
d. thermoregulation center inhibition infections current?
e. primary pyrogenes destruction a. Subfebrile temperature
323. Patient V., 32 years old, had headache, weakness, b. Febrile temperature
extremities pain, stuffy nose, rigor in the morning. Temperature c. Pyretic temperature
was risen up to 38.20 Diagnosis: influenza. What accompanying d. Hyperpyretic temperature
disease needs the prescription of anti-pyretic drugs? 331. A warmly dressed child has spent a long time in
a. double pneumonia doors. This resulted in body temperature elevation and general
b. heart failure weakness development. What form of thermoregulation
c. diabetes mellitus disorder is observed in this case?
d. hypoxia development a. Exogenous hyperthermia
e. increased blood sedimentation rate b. Endogenous hyperthermia
324. Patient V., 32 years old, had headache, weakness, c. Fever
extremities pain, stuffy nose, rigor in the morning. Temperature d. Heat shock
was risen up to 38.20 Diagnosis: influenza. The patient has no e. Neurogenous hyperthermia
accompanying diseases. The patient was not prescribed 332. A 25 year old man has spent a long time in the sun
antipyretic drugs. Is it correct? Why? under condition of high air humidity. As a result of it his body
a. No. The patient must be prescribed antipyretic drugs. temperature rose up to 39 C. What pathological process has
b. Yes. Because fever activates immune system developed in the patient?
c. Yes. Because body temperature is relatively low a. Hyperthermia
d. Yes. Because flu is not treated at all b. Infectious fever
325. It is know that pyrotherapy can be used in different c. Hypothermia
clinical cases. Which clinical case from the following can NOT d. Noninfectious fever
be treated with pyrotherapy? e. Burn disease
a. Chronic infectious diseases 333. In patient with relapsing fever, several days are
b. Acute infectious diseases characterized with the periods of high fever, which alternate
c. Trauma of bones and muscles with the periods of normal temperature. Such type temperature
d. Cancer treatment curve is called:
e. Intensification of reparative processes after surgical a. Febris recurrens
operations b. Febris hectica
c. Febris intermittens
d. Febris continua
e. Febris atypica a. decrease of heat production due to decrease of metabolism
334. Development of fever state is accompanied with b. activation of immune system protective function
the increase of "acute phase" proteins level (ceruloplasmin, c. decrease of endogenous pyrogens formation
fibrinogen, C-reactive protein). Name the possible mechanism d. development of tolerance to pyrogens
this phenomenon: e. destruction of pyrogens
a. stimulating effect of interleukin-1 on hepatocytes 337. During a day the patient ’s body temperature rises
b. the destructive effect of elevated temperature on the body's and keeps at its level for 1 to 3 hours; then it drops to the
cells normal level. such type of fever is observed regularly every
c. proliferative effect of interleukin -2 on T-lymphocytes fourth day. What type of temperature curve is described?
d. degranulation of tissue basophils a. Febris recurrens
e. activation of the complement system b. Febris hectica
335. The patient with acute pneumonia developed c. Febris intermittens
febrile fever. Which mediator of inflammation can play a role d. Febris continua
of endogenous pyrogen? e. Febris atypica
a. interleukin-1 338. After overcooling patient ’s body temperature rose
b. histamine up to 39,7 C and varied from 39 C to 39,8 during 3 days. What
c. bradykinin type of temperature curve is described?
d. serotonin a. Febris recurrens
e. leukotriene b. Febris hectica
336. The patient with acute respiratory viral infection c. Febris intermittens
has developed subfebrile fever. He was not prescribed anti- d. Febris continua
pyretic medicines. In two days his body temperature returned to e. Febris atypica
normal range. What is the most probable mechanism of
temperature decrease in this case?

Tissue growth pathology. Tumors


339. The group of individuals most susceptible to the cancer of carbohydrate metabolism are observed in malignant tumor
often … cells?
a. are anemic a. positive Paster effect
b. are immunodeficient b. activation of ATP formation
c. have neurologic dysfunction c. aerobic glycolysis activation
d. have chronic diseases d. negative Paster effect
e. have high Ca blood level e. glycogen synthesis activation
340. Carcinogens influence results in the transformation of 345. Patient Ch., 60 years old. Diagnosis: liver cancer
the proto-oncogens to oncogens, which leads to tumor (hepatoma). Blood analysis: ALT decreased level, embryonic
development. What substances synthesis is controlled by the fetoprotein presence. What is the mechanism of fetoprotein
proto-oncogens? appearance in hepatoma patient serum?
a. innate immunity humoral factors a. protein metabolism violation in tumor surrounding cells
b. growth factors b. mRNA synthesis violation
c. growth hormones c. cell division regulation violation
d. neuropeptides d. ATP synthesis violation in consequence of tumor
e. complement proteins intoxication
341. Patient K., 55 years old, complaints about weight loss, e. tumor cells protein metabolism violation
increased fatigability, dry cough, and bloody phlegm. 346. Patient N., 50 years old, complaints about weight loss,
Anamnesis: prolonged contact with organic toxic substances. fatigability. Blood analysis: hypoglycemia, hyperinsulinemia.
Diagnosis: lung tumor. What process underlies mutational Langerhans islet tumor was found after additional research.
cancerogenesis mechanism? Why insulin synthesis is increased in the patient?
a. protooncogenes transformation into oncogenes a. morphology anaplasia
b. cell division regulation violation b. biochemical anaplasia
c. mRNA synthesis violation c. functional anaplasia
d. ATP synthesis violation d. physical and chemical anaplasia
e. Ca++ insufficient entrance in cell e. immunological anaplasia
342. Patient was diagnosed liver tumor. What process 347. Patient K., 54 years old. Diagnosis: lung tumor. Biopsy:
underlies epigenetic cancerogenesis mechanism? cells with different shape and size, invasive growth. What type
a. protooncogenes transformation into oncogenes of acid-base balance violation is possible in this case?
b. activation of the cell mitosis rate a. respiratory acidosis
c. mRNA synthesis violation b. metabolic acidosis
d. ATP synthesis violation c. respiratory alkalosis
e. Ca++ insufficient entrance in cell d. metabolic alkalosis
343. Stomach malignant neoplasm was experimentally e. negative Paster effect
modeled on animal, and tumor cells were cultivated for the 348. Patient K., complaints about weight loss. Diagnosis:
purpose of biochemical anaplasia research. What changes in mediastinum tumor with intestines metastases. Blood analysis:
glycolisis will be observed after oxygen adding in cell culture? uncompensated acidosis. What possible mechanism led to
a. anaerobic glycolysis activity increase acidosis development?
b. anaerobic glycolysis activity decrease a. positive Paster effect in tumor cells
c. aerobic glycolysis activity increase b. CO2 accumulation in the blood
d. no changes after oxygen adding c. alkaline accumulation in the intestines
e. aerobic glycolysis activity decrease d. anaerobic glycolysis in tumor cells
344. Patient K., complaints about weight loss. Diagnosis: e. buffer systems blocking by tumor toxins
mediastinum tumor with intestines metastases. What changes
349. Patient T., 59 years old. X-raying: oval dark patch in M0). What stage of tumor pathogenesis development is in this
right lung lower lobe with distinct bounds 3x5 cm in size, case?
which is typical for tumor. What feature is typical for non- a. promotion
cancerous growth? b. tumor progression
a. metastasizing c. protooncogene transformation into oncogene
b. cancer cachexia d. oncoproteins formation
c. germination in surrounding tissue e. tumor immune inhibition
d. infiltrative growth 357. It is known that proto-oncogenes mutations may result
e. expansive growth in cancer development. Which substances synthesis is encoded
350. Patient A., 45 years old. Mamma tumor with increased by proto-oncogenes?
surrounding lymph nodes was found after clinical examination. a. growth factors
Biopsy: cancerous growth. What is the main mechanism of b. growth suppressors
infiltrating pattern of cancer growth? c. natural antioxidants
a. contact inhibition absence d. apoptosis inhibitors
b. tumor cells ability to amoeboid movement e. immunoglobulins
c. tumor cells adhesion increase 358. Cancer development may result from mutational and
d. growth factors activation in tumor cells epigenetic carcinogenesis. Which from the listed factors may
e. lysosomes enzymes activity increase initiate epigenetic carcinogenesis?
351. Patient R., 52 years old, complaints about weakness, a. ionizing radiation
stomach aches, digestion violation, and weight loss during last b. ultraviolet rays
3 months. Pancreas tumor was found with metastases into c. formaldehyde
intestines mesentery under laparotomy. Carbohydrate, protein, d. chronic tissue injury
lipid metabolisms violation was found. What mechanism could e. tobacco smoke
lead to patient ’s weight waste? 359. Choose the most correct definition for the described
a. nutritious substances consumption decrease because of process “A pathologic process in which a permanent alteration
digestion violation in a cell ’s growth controlling mechanism permits its continuous
b. plastic and energetic reserves decrease because of proliferation”
metabolism violation a. tumor
c. tumor absorption of plastic and energetic reserves b. neoplasia
d. organism intoxication because of metabolism violation c. neoplasm
e. increase of plastic and energetic reserves usage by d. cancer
antitumor protecting system e. proliferation
352. Patient R., 52 years old, complaints about weakness, 360. Benign and malignant neoplasms have characteristics
stomach aches, digestion violation, and weight loss during last both common and unique to for these types of neoplasms.
3 months. Pancreas tumor was found with metastases into Which from the following characteristics is unique to
intestines mesentery under laparotomy. Choose the correct malignant neoplasm?
definition of metastasis. Metastasis is… a. absence of cell division limit
a. an alteration in normal cell growth b. irreversible new growth
b. growth of benign or malignant neoplasmsl c. autonomy
c. the ability of secondary tumor nodes growth d. less degree of differentiation
d. a mutation in normal cells e. epigenetic carcinogenesis
353. Which of the following is the correct sequence of events 361. Benign and malignant neoplasms have characteristics
during the process of metastasis? both common and unique to for these types of neoplasms.
a. vascularization, adherence of neoplastic cells, invasion Which from the following characteristics is seen only in benign
into lymph and vascular system neoplasms?
b. transport, vascularization, adherence of neoplastic cells a. anaplasia
c. vascularization, extravasation, transport b. metastases
d. cell detachment, invasion into lymph and vascular c. loss of differentiation
system, migration d. autonomy
354. Worker M. works at factory, where chemical e. encapsulation
cancerogenes are used, but he doesn’t have tumor. What type 362. Malignant tumor cells clone was grown in laboratory to
of represented below phenomenon is referred to anticellular investigate the following cellular phenomena: Hayflick limit,
mechanisms of anti-tumor defense? contact inhibition and biochemical anaplasia. Which process is
a. tumor cell elimination regulated by Hayflick limit?
b. oncogenes elimination in cell a. speed of ATP synthesis
c. oncogene expression inhibition b. speed of DNA synthesis
d. cancerogenes elimination in cell. c. proto-oncogens activation
e. blastomogene factors inactivation d. maximal number of cell divisions
355. Which from the following mechanisms is supporting e. speed of cell division
antimutational mechanism of anti-tumor defense? 363. Clinical examination of the patient revealed the initial
a. inactivation of carcinogens by phagocytosis stage of the liver cancer. Presence of which type of protein in
b. activation of cells anti-oxidative system blood serum will confirm this diagnosis?
c. inactivation of carcinogens by the natural metabolic a. gamma-globulin
processes b. properdin
d. activation of cytotoxic T-lymphocytes c. paraprotein
e. elimination or inhibition of oncogenes d. c-reactive protein
356. Patient R., 53 years old. Stomach tumor with metastases e. alpha-fetoprotein
in perigastrial lymph nodes were found. Distant metastases are 364. A man has been working at the petroleum refining
absent. Tumor development third stage is determined (T3, N1, industry for a long time. Which class of occupational
carcinogens is present at this type of industry?
a. nitrosamines tobacco smoking. Choose from the list of carcinogens the
b. aromatic amines substance which is present in tobacco smoke:
c. polycyclic aromatic hydrocarbon s a. benzpyrene
d. pesticides b. aminoazotoluol
e. arsenic compounds c. aflatoxin
365. A person with complaints about weakness, cough with d. methylcholanthrene
bloody phlegm and chest pain was diagnosed lung cancer. e. diethylnitrozamine
Choose the name of the first stage of carcinogenesis from the 368. Clinical examination of the patient with cancer of
following list: esophagus revealed cancer metastases to mediastinal lymphatic
a. transformation nodes, general cachexia. Which stage of cancer development is
b. promotion described in this clinical situation?
c. activization a. transformation
d. progression b. promotion
e. implantation c. activization
366. A 56-year-old female patient complains of a fast d. progression
growing hard neoplasm in the mammary gland that e. implantation
appeared a month ago. Objectively: the tumor is fused with the 369. Malignant neoplasm is characterized with the alteration
surrounding tissues, with uneven shape, slightly painful. of carbohydrate ’s metabolism in comparison to the normal
Choose the charteristic of malignant tumor cell, which tissue. The same amount of glucose in normal tissue cells
determines the infiltrative type of its growth results in 20-25 times higher energy production then in
a. lack of contact inhibition malignant tumor cells. Which alteration in carbohydrate ’s
b. negative Paster effect metabolism is present in malignant tumor cells?
c. increased contact inhibition a. activation of oxidative reactions
d. tight intercellular junctions b. activation of reduction reactions
e. presence of embryonal antigens c. increase of anaerobic glycolysis
367. Epidemiological study of the spread of tumors showed a d. increase of aerobic glycolysis
high correlation between the lung tumors development with e. decrease of anaerobic glycolysis

Hypoxia
370. Patient L., 47 years old, has pain shock as a result of a. diminishment of tissues blood supply
arm trauma. The patient is in a hard state, humid, pale skin with b. inactivate cytochromeoxidase
acrocyanosis, confused consciousness, tachypnoe, tachycardia, c. activates membranes lipids oxidation
AP decreased. What type of hypoxia does this patient have? d. damage mitochondria
a. hemic e. lead to increased cells oxygen consumption
b. tissue 375. The cause of 20 years old girl death was acute hypoxia
c. respiratory which was developed as a result of cyanides intoxication. What
d. substrate stage of O2 transport was violated?
e. circulatory a. oxygen transport by hemoglobin
371. A group of tourists went to the mountains travel. On the b. hemoglobin synthesis
third day two of them felt symptoms of mountain sickness: c. oxygen diffusion in lungs
increased fatigueability, noise in the ears, palpitations, short d. tissue oxygen consumption
breath. Which pathological process was developed in the e. alveolar ventilation
tourists? 376. Patient O., 65 years old, used big doses of sulfanilamide
a. physical overload medicines for a long time. Now he has breathlessness,
b. normobaric hypoxic hypoxia weakness, appetite loss, sleep violation. Methemoglobinemia
c. hypobaric hypoxic hypoxia was found in the blood. What is mechanism of hemoglobin
d. respiratory hypoxia inactivation in erythrocytes under methemoglobin formation?
e. unknown poisoning a. hemoglobin joining with sulfanilamides
372. Patient E., was put on a strict diet (decreased b. reconstructive enzyme systems inactivation
consumption of carbohydrates) in order to lose weight. In a six c. iron oxidation in hemoglobin
days she began to complain of weakness, short breath, d. protein damage in hemoglobin
impossibility to work normally. What type of hypoxia has e. oxidative enzyme systems inactivation
developed in this patient? 377. Woman has aniline intoxication. She complains of:
a. tissue nausea, retching, headache, tinnitus, midges in the eyes,
b. respiratory weakness, drowse. She has cyanosis of skin and mucous
c. circulatory membranes, breathlessness, tachycardia. What hypoxia is in
d. hemic this case?
e. substrate a. hemic
373. Patient S., has an alcohol intoxication. He was found b. circulatory
pale skin, tachypnoe, and tachycardia. One of alcohol toxic c. hystotoxic
influence mechanisms in organism is Krebs cycle enzymes d. substrate
activity decrease. What type of hypoxia does this patient have? e. hypoxic
a. tissue 378. Patient E., 26 years old has hypoxia, which was
b. respiratory developed as a result of larynx edema. The patient is in a hard
c. circulatory state, humid, pale skin with acrocyanosis, tachypnoe (increased
d. hemic frequency of breathes), tachycardia, decreased arterial pressure.
e. substrate What symptom of acute hypoxia is the manifestation of
374. The cause of man death was hypoxia which was organism urgent protective adaptation reactions?
developed as a result of cyanides intoxication. What is a a. skin pallor
mechanism of cyanide ’s hystotoxic action? b. decreased AP
c. increase of frequency and intensity of breath altitude of 3000 m. What is the reason for their increased RBC
d. acrocyanosis development number?
e. increased sweat secretion a. increased vitamin B12 synthesis
379. Patient R., 46 years old, has hypoxia as a result of b. increased blood clotting
emetic masses aspiration. The patient is in a hard state, humid, c. changes in spleen function
pale skin with acrocyanosis, tachypnoe, tachycardia, decreased d. increased erythropoietin production
arterial pressure. What symptom of acute hypoxia is the e. increased blood circulating volume
manifestation of organism urgent protective adaptation 387. A child has been brought to the hospital. He had nitrates
reactions? poisoning symptoms: cyanosis, dyspnoea and cramps. What is
a. skin pallor the reason of the symptoms development
b. decreased AP a. methemoglobin formation
c. acrocyanosis development b. oxyhemoglobin formation
d. tachycardia c. reduced hemoglobin formation
e. increased sweat secretion d. carbhemoglobin formation
380. Patient M., was given hypoxic trainings in normobaric e. carbooxyhemoglobin formation
conditions. What urgent protective adaptation reactions can be 388. There are several classifications of hypoxia. One of
developed in reply to acute developing hypoxia first séance? them is dividing hypoxia on subtypes due to time of appearance
a. erythopoesis stimulation and duration of hypoxia features. Define the type of hypoxia
b. parasympathetic nervous system activation which resulted from cyanide poisoning from the following:
c. venous inflow decrease to heart a. Fulminant
d. breath frequency increase b. Acute
e. reserve alveoli are included into breath c. Subacute
381. Patient S., 54 years old, has hypoxia, which was d. Chronic
developed as a result of emetic masses aspiration. Can e. Endogenous
erythrocytes quantity be changed in periphery blood in hypoxia 389. There are several classifications of hypoxia. One of
first hours? them is dividing hypoxia on subtypes due to time of appearance
a. won ’t be changed in hypoxia first stages and duration of hypoxia features. Define the type of hypoxia
b. is decreased as a result of erythrocytes hemolysis which resulted from cardiac arrest from the following:
c. is increased, by means of hemopoiesis increase a. Fulminant
d. is decreased as a result of blood accumulation in depot b. Acute
e. is increased, by means of their leaving blood depot c. Subacute
382. Patient M., 35 years old, lives in the mountains. What d. Chronic
index of external respiration will be changed in this patient? e. Endogenous
a. increased lungs vital capacity 390. Which from the listed hypoxia types is observed more
b. increased inspiration duration frequently than others?
c. increased expiration duration a. respiratory
d. lung breathlessness development b. hemic
e. breath frequency increase c. circulatory
383. Patient T., 27 years old, lives in the mountains. He was d. histotoxic
made heart-vessel system instrumental research. What heart e. combined
activity change does patient have? 391. Which from the listed hypoxia types is observed in the
a. respiratory arrhythmia case of prolonged organism malnutrition or starvation?
b. heart stroke volume increase a. hemic
c. bradycardia b. circulatory
d. heart minute volume decrease c. histotoxic
e. tachycardia d. substrate
384. Patient T., 46 years old. Diagnosis: chronic respiratory e. combined
insufficiency. He was found: acrocyanosis, breathlessness, 392. It is known that interval hypoxic trainings can increase
heart bounds widening, arterial pressure increased, erythrocytes the working capacity of the body’s organs and systems. Which
quantity increased, blood clotting, leukocytosis. What symptom type of hypoxia should be used with this aim?
of chronic hypoxia is referred to organism prolonged a. respiratory
compensatory mechanisms? b. hemic
a. blood clotting c. hypoxic
b. increased arterial pressure d. circulatory
c. increase quantity of erythrocytes e. histotoxic
d. increased respiration frequency 393. Choose the example of circulatory hypoxia from the
e. leukocytosis listed clinical cases:
385. Patient Yu., 40 years old, has acrocyanosis, a. bronchial asthma attack
breathlessness, heart bounds widening, increased arterial b. anaphylactic shock
pressure, erythrocytes quantity increase, blood clotting, c. starvation
leukocytosis. Diagnosis: pulmonary tuberculosis. Which d. iron deficiency anemia
symptom of chronic hypoxia is the manifestation of organism’ e. nitrates poisoning
prolonged compensatory mechanisms? 394. Choose the example of hypoxia caused by hemoglobin
a. neutrophilic leucocytosis inactivation from the listed clinical cases:
b. AP increase a. bronchial asthma attack
c. respiration frequency increase b. anaphylactic shock
d. heart cavities dilatation and myocardium hypertrophy c. starvation
e. blood clotting d. iron deficiency anemia
386. Increased number of RBC was revealed in people e. nitrates poisoning
residing in village which is situated in the mountains at the
395. What form of hypoxia develops during shock and e. substrate
collapse? 398. Choose the possible reason of histotoxic (tissue)
a. circulatory hypoxia from the given:
b. respiratory formation of methemoglobin
c. hypoxic decreased activity of tissue respiratory enzymes
d. hemic acute blood loss
e. tissue increased synthesis of prostoglandin E
396. The patient for a long time suffers from thyrotoxicosis. poisoning with carbon monoxide
What type of hypoxia can develop in this patient? 399. Which alterations in cell’s metabolism in a person with
a. tissue chronic hypoxia can verify the adaptation to hypoxia?
b. hemic decreased activity of glycolysis
c. circulatory activation of phospholipase A2
d. respiratory activation of lipids peroxidation
e. mixed increased activity of glycolysis
397. A healthy person after intensive physical work in the increased inntracellular sodium
opened air manifested with the symptoms of hypoxia: 400. Choose the possible reason of mixed type of hypoxia
weakness, paleness of skin, dizziness, increased heart and from the given:
breath rate. Which type of hypoxia may develop in the patient? acute blood loss
a. hemic chronic blood loss
b. respiratory carbon monoxide poisoning
c. histotoxic lung emphysema
d. overload mountain sickness

Submodule 3. “Metabolism disturbances”


Disturbance of carbohydrate metabolism. Diabetes mellitus.
401. The effects of insulin action listed below can be divided a. absolute insulin insufficiency
according to the time of their expression. Which is the fastest b. glucose-tolerance test violation
insulin effect? c. tissues insulin-resistance
a. anabolism stimulatuion d. blood keton bodies high level
b. hypoglycemic e. stable hyperglycemia
c. catabolism inhibition 407. Patient T., 55 years old, with obesity developed diabetes
d. cellular division stimulation mellitus type II. What is the main pathogenic factor in this
402. Muscular and adipose tissue form the majority of body pathology development?
weight. They are insulin-dependent tissue. Choose the correct a. inherited diathesis
definition: insulin dependent tissues… b. obesity
a. consume glucose with the help of insulin c. old age
b. functions are determined by insulin d. tissues insulin-resistance
c. consume glucose directly from the blood e. viral infection
d. are affected in type 1 diabetic patients 408. The patient with obesity has diabetes. What is obesity
e. are affected in type 2 diabetic patients for diabetes?
403. Patient Ts., 35 years old, has glucose-tolerance test a. complication
violation. Diagnosis: diabetes mellitus. What does this violation b. risk factor
testify? c. outcome
a. increased glucagon secretion by alpha-cells d. stage of development
b. insulin secretion inhibition by beta-cells e. obesity is not connected with diabetes
c. decreased glucose consumption in insulin-dependent 409. Patient P., 52 years old, has diabetes mellitus type II and
cells obesity. What is the main feature of lipid ’s usage for energy
d. increased tissues insulin-resistance needs?
e. increased contra-insulin hormones secretion a. hyperketonemia
404. Choose the characteristic feature of type 1 diabetes b. hyperlipidemia
mellitus c. hyperglycemia
a. Middle age at onset d. hypernitrogenemia
b. Associated obesity e. hyperphagia
c. Low plasma levels of endogenous insulin 410. Patient D., 28 years old, has protein metabolism
d. Insulin resistance violation, which is clinically developed as wounds healing
e. Presence of antibodies to islet cells violation, decreased. anti-bodies synthesis. Diagnosis: diabetes
405. Patient E., 25 years old. Polydipsia, poliuria, glucosuria, mellitus type I. What clinical symptom can verify this violation?
stable hyperglycemia were developed in 2 weeks after viral a. blood proteins concentration decrease
infection disease. Diagnosis: diabetes mellitus type I. What is b. hyperproteinemia
the main pathogenic mechanism in this case? c. gluconeogenesis inhibition
a. alpha-cells destruction d. aminoacidemia
b. beta-cells destruction e. α-fetoprotein appearance in blood
c. tissues insulin resistance increase 411. Patient with diabetes mellitus has hyperglycemia 19
d. glucose toxic action mmol/ l, which is clinically developed as glucosuria, polyuria,
e. metabolism violation polydipsia. What mechanism is responsible for polyuria
406. Patient has diabetes mellitus type I. Blood glucose development?
concentration is 18 mmol/l. What is a characteristic feature of a. hyperphagia
this disease? b. hyperlipidemia
c. polydipsia 420. A patient was delivered to the hospital by an emergency.
d. tissues dehydration Patient is unconscious, the skin dry, face is cyanotic. Heart rate
e. glucosuria is 132 bpm. There is acetone smell from the mouth. Blood
412. Patient with diabetes mellitus has glucosuria, polyuria, glucose level – 20.1 mmol/L, urine glucose – 3,5 g/L. What is
polydipsia. What is the minimal blood glucose level, which is the probable diagnosis?
accompanied with glucosuria? a. hypoglycemic coma
a. 8 mmol/ L b. acute infectious intoxication
b. 10 mmol/ L c. anaphylactic shock
c. 12 mmol/ L d. acute heart failure
d. 14 mmol/ L e. hyperglycemic coma
e. 16 mmol/ L 421. A patient with diabetes mellitus was delivered to the
413. One of the diabetes mellitus clinical symptoms is hospital in coma. It is known that coma is an acute
hyperphagia. It is developed due to… complication of diabetes mellitus. Which diabetic coma
a. lack of energy in the organism development is the most severe life threatening condition?
b. lack of fatty acids in the blood a. hypoglycemic
c. lack of insulin b. hyperglycemic
d. excess of glucose in the blood c. hyperlactacidemic
e. affection of appetite controlling centers d. hyperosmolar
414. Patient with diabetes mellitus has hyperglycemia 19 e. ketonemic
mmol/ l, which is clinically developed as glucosuria, polyuria, 422. A patient with type 2 diabetes mellitus is complaining
polydipsia. What mechanism is responsible for polydipsia of progressive loss of vision. Which complication development
development? may underlie this process?
a. low osmotic pressure of blood plasma a. diabetic angiopathy
b. lack of insulin b. diabetic retinopathy
c. tissues dehydration c. diabetic neural dystrophy
d. glucosuria d. diabetic neuropathy
e. hyperglycemia 423. A patient with constant thirst and increased urination
415. Which coma often occurs in the patients with diabetes was done oral glucose tolerance test that proved diabetes
mellitus type 1 when diet is not balanced with insulin injections? mellitus diagnosis. Which sign of diabetes is typical only to
a. hyperglycemic type 1 diabetes mellitus?
b. hyperlactatacidemic a. hyperglycemia
c. hyperosmolar b. hypoglycemia
d. ketonemic c. relative insulin deficiency
e. hypoglycemic d. obesity
416. Patient R., 46 years old, has diabetic neuropathy. What e. absolute insulin deficiency
is the main mechanism in nervous fibers damage under diabetes? 424. A patient with type 1 diabetes mellitus manifests
a. glucose toxic action disturbances in protein metabolism: high level of
b. ketones toxic action aminoacidemia. Which mechanism is responsible to
c. nervous fibers dehydration aminoacidemia development?
d. metabolic acidosis development hyperproteinemia
e. glucose accumulation in nervous tissue a. increased proteolysis
417. Patient O., 49 years old, has hypoglycemia symptoms. b. decreased aminoacids blood concentration
Violation of cerebral cortex neurons and sympathoadrenal c. increased blood osmotic pressure
system activation are mechanisms of hypoglycemia symptoms d. increased blood onkotic pressure
development. Which body tissue need constant blood glucose 425. A patient with type 1 diabetes mellitus has disturbances
supply? in protein metabolism which clinically manifest as decreased
a. respiratory system wounds healing. Which mechanism is responsible to low
b. kidney tissue activity of regeneration process in the patient?
c. nervous system a. decreased blood pH
d. endocrine system b. accumulation of keton substances in the blood
e. gastro-intestinal system c. decreased synthesis of proteins
418. Patient with diabetes mellitus type I was done insulin d. increased synthesis of insulin counterregulatory
prolonged intravenous infusion in order to decrease glycemia hormones
high level. Hypoglycemic coma was developed. Choose the e. increased activity of lipid metabolism
mechanism of hypoglycemia development? 426. The patient is suffering from diabetes mellitus type 1 for
a. increased glucose excretion from organism by urine a long time lost consciousness after extreme physical loading.
b. glucagon secretion inhibition He was immediately hospitalized in the endocrinology ward.
c. glucose interaction with insulin Clinical observation: superficial breathing, heart rate – 132 bpm,
d. tissues glucose increased consumption BP -80/40 mmHg, glycemia level – 1,88 mmol/L. Which
e. inhibition of gluconeogenesis and ketogenesis in liver diabetic complication has developed in the patient?
419. A patient is ill with diabetes. Glycemia fasting level is a. hyperglycemic coma
7,5 mmol/L. The level of which blood plasma protein allows to b. hyperosmolar coma
estimate the glycemia rate retrospectively (4-8 weeks before c. diabetic neuropathy
examination)? d. hypoglycemic coma
a. fibrinogen e. diabetic nephropathy
b. glycosylated hemoglobin 427. A patient was done fasting blood level test after 12
c. albumin hours of fasting. Glycemia level is 3,7 mmol/L. Which
d. ceruloplasmin mechanism is maintaining blood glucose level in the state of
e. C-reactive protein zero calorie intake?
a. activation of glycogenolysis
b. activation of glycogenesis irritability, increase of sweating, blood glucose level -3,2
c. activation of glycolysis mmol/l. What is the basic mechanism of hypoglycemia
d. inhibition of glycogenesis manifestation development?
e. inhibition of gluconeogenesis a. increase of ketone bodies synthesis
428. A boy 12 yeas old in 6 months after a severe viral b. increase of glycogenolysis
disease lost 7 kg of weight in spite of increased appetite. Daily c. carbohydrate starvation of the brain
diuresis is 3 L. Casual blood glucose level is 12 mmol/L. What d. increase of lipogenesis
is the probable diagnosis for this patient? e. decrease of gluconeogenesis
a. type 2 diabetes mellitus 431. Diabetes mellitus is often characterized by the
b. type 1 diabetes mellitus development of acute complications. Which life-threatening
c. polyuria from the given list may be determined by diabetes mellitus
d. insulin resistance development.
e. hyperglycemia a. shock
429. A patient with diabetes mellitus was delivered to b. coma
hospital in the unconscious state. ANP – 80/50 mmHg, acetone c. collapse
smell from the mouth, and Kussmaul respiration are present in d. acute liver failure
the patient during clinical examination. Accumulation of which e. acute renal failure
substances may cause such abnormalities? 432. The key mechanism of diabetes mellitus type 2
a. ketone bodies development is insulin resistance. Give the correct definition of
b. beta-lipoproteins insulin resistance:
c. lactic acid a. disturbance of cellular response to insulin influence
d. complex carbohydrates b. decrease of insulin synthesis in the pancreas
e. aminoacids c. increase of insulin synthesis in the pancreas
430. A patient with type 1 diabetes mellitus was given an d. increase of cellular response to insulin influence
insulin injection. In 2 hours he developed general weakness, e. synthesis of insulin with altered structure

Acid-base balance and water – electrolyte balance disorders.


433. At examination of the patient alteration of ABB was 438. The patient who suffered from diabetes mellitus was
found.. Which buffer system of blood will change first of all? delivered to the hospital because of worsening of his condition.
a. Bicarbonate He has general sickness, polyuria, lethargy and sleepiness.
b. Phosphate Kussmaul respiration, heart arrhythmia and acetone’s smell in
c. Hemoglobin expired air were noticed in this patient. What kind of acid-base
d. Oxyhemoglobin balance’s shift described in this case?
e. Protein a. Gas alkalosis
434. Prolonged convulsions occur in the patient suffered b. Gas acidosis
from epilepsy. Blood count: pH – 7.14 (N-7.34-7.44), pCO2 – 45 c. Non-gas metabolic alkalosis
(N- 35-45) mmHg, HCO3- – 14 mmol/L (N- 22-26). What kind d. Non-gas metabolic acidosis
of acid-base balance disturbances occur in this case? e. Non-gas excretory alkalosis
a. Metabolic ketoacidosis 439. A group of alpinists undergone blood analysis in
b. Metabolic lactoacidosis mountains at height 3000 meters. Blood count: decrease of
c. Respiratory alkalosis HCO3- to 15 mmol/l (norm is 22-26 mmol/l). What is the
d. Metabolic alkalosis mechanism of HCO3- decrease in the blood?
e. There are no disorders of acid-base balance a. Decrease of HCO3- reabsorption in kidneys
435. Alpinist excitation developed during ascending to b. Hyperventilation
mountain. It was replaced with headache, giddiness, c. Intensification of acidogenesis
breathlessness and after that apnea occurred. What kind of d. Hypoventilation
acid-base balance disorder occurs in this case? e. Reduction of ammoniogenesis
a. Non-gas acidosis 440. Buffer capacity of blood decreases in worker as a result
b. Excretory acidosis of exhausting muscle work. What acid substance income to the
c. Gas alkalosis blood this may be explained?
d. Non-gas alkalosis a. α-ketoglutaric acid
e. Gas acidosis b. 3-phosphoglycerate
436. The patient who suffered from chronic c. lactic acid
glomerulonephritis has increased general sickness, tachycardia d. pyruvic acid
with recurrent arrhythmia, confusion and sleepiness. What kind e. 1,3-biphosphoglycerate
of acid-base balance disturbances accompanies uremic coma? 441. Repeated vomiting occurs in patient with
a. Metabolic acidosis pylorostenosis which is accompanied by loss of chloride ions
b. Metabolic alkalosis from the organism and development of metabolic alkalosis.
c. Gas acidosis What conditions may result from these changes of acid-base
d. Gas alkalosis balance?
e. Respiratory alkalosis a. Hyperchloremia
437. Pregnant wom an has toxicosis accompanied by b. Hyponatremia
vomiting of 24 hours duration. Tetany and dehydration were c. Hypokalemia
developed soon. What kind of acid-base balance’s shift leads to d. Hypernatremia
described changes? e. Hyperphosphatemia
a. Gas alkalosis 442. 48-years-old patient with diabetes mellitus was
b. Gas acidosis delivered to the hospital in severe pre-coma condition.
c. Non-gas metabolic acidosis Metabolic acidosis was found after examining of acid-base
d. Non-gas metabolic alkalosis balance. Patient was treated with complex therapy including
e. Non-gas excretory alkalosis intramuscular injections of insulin and intravenous infusion of
sodium bicarbonate solution. What is the main possible a. Metabolic acidosis
mechanism of the acid-base balance change? b. Respiratory acidosis
a. Disorders of O2 use in cells c. Respiratory alkalosis
b. Disorders of buffer systems of blood d. Metabolic alkalosis
c. Decrease of CO2 remove by lungs e. Excretory acidosis
d. Excessive excretion of alkalis with urine 451. The patient was injected with 500 ml of 5% solution of
e. Accumulation of lipid incomplete oxidation compounds glucose. What type of water electrolyte misbalance may take
443. Patient has disturbances of airways passage at the small place in this case?
and medium bronchi level. What kind of acid-base balance a. hypoosmolar dehydration
disorder may develop in this patient? b. hyperosmolar hyperhydration
a. Respiratory alkalosis c. isoosmolar hyperhydration
b. Metabolic alkalosis d. hypoosmolar hyperhydration
c. Respiratory acidosis e. hyperosmolar dehydration
d. Metabolic acidosis 452. During mountain climbing a sportsman developed
e. Acid-base balance does not change severe short breath, headache, giddiness, and palpitation. What
444. The patient had been under artificial ventilation during type of ABB disorder did the mountain climber manifest?
operation for cardiac valve insufficiency. When the patient a. respiratory alkalosis
was awakened he developed muscles tetany, decrease of b. metabolic alkalosis
kidney function and overexcitability of the nervous system. it c. non-gas alkalosis
was found out that the volume of artificial ventilation was d. respiratory acidosis
calculated incorretly. What kind of acid-base balance disorders e. excretory acidosis
develops in this case? 453. A patient ill with enteritis accompanied by massive
a. Exogenous acidosis diarrhea has low water rate in the extracellular space, high
b. Respiratory acidosis water rate inside the cells and low blood osmolarity. What is
c. Metabolic acidosis such disturbance of water-electrolytic metabolism called?
d. Respiratory alkalosis a. Hypoosmolar hypohydration
e. Non-gas alkalosis b. Hyperosmolar hypohydration
445. A wom an has distinct hypersalivation syndrome. She c. Osmolar hypohydration
has removed saliva from her mouth with napkin because of d. Hypoosmolar hyperhydration
hard pain during swallowing. What kind of acid-base balance e. Hyperosmolar hyperhydration
disorder may develop in this patient in some time? 454. An infant has apparent diarrhea resulting from improper
a. Non-gas excretory acidosis feeding. One of the main diarrhea effects is loss of sodium
b. Gas alkalosis bicarbonate from the intestines. What form of acid-base
c. Metabolic acidosis balance disorder may take place in this case?
d. Non-gas excretory alkalosis a. Metabolic acidosis
e. Gas acidosis b. Metabolic alkalosis
446. Gas acidosis (hypercapnia) was developed during c. Respiratory acidosis
bronchial asthma attack. What buffer system of blood plays the d. Respiratory alkalosis
main role in compensation of this condition? e. No disorders of acid-base balance will be observed
a. Bicarbonate 455. A pregnant woman had been having toxicosis with
b. Hemoglobin severe repeated vomiting for 24 hours. In the end of the day
c. Phosphate there appeared tetanic convulsions and fluid loss. What shift of
d. Protein acid-base state caused these changes?
e. Ammoniogenesis a. Gaseous alkalosis
447. Hyperglycemia, ketonuria, polyuria, hyperstenuria and b. Gaseous acidosis
glucosuria were found during patient ’s examination. What kind c. Excretory alkalosis
of acid-base balance disorder takes place in this case? d. Metabolic acidosis
a. Metabolic acidosis e. Excretory acidosis
b. Gas acidosis 456. What is the purpose of sodium bicarbonate infusion
c. Metabolic alkalosis during resuscitation actions?
d. Non-gas alkalosis a. Increase of survival time of brain under hypoxia
e. Gas alkalosis b. Respiratory acidosis prevention
448. The 65-years-old patient with multiple fractures of ribs c. Metabolic acidosis correction
was delivered to the hospital. What type of acid-base balance d. Metabolic alkalosis therapy
disorder may develop in this case? e. Hypoxia liquidation
a. Gas acidosis 457. A patient with pneumosclerosis has blood pH at the rate
b. Gas alkalosis of 7,34. Analysis of gas formula of blood showed hypercapnia.
c. Non-gas acidosis Urine analysis revealed the increase in its acidity. What form of
d. Non-gas alkalosis acid-base misbalance is present in this case?
e. There are no disorders of acid-base balance a. Secretory alkalosis
449. Disorders of ABB can manifest as acidosis or alkalosis. b. Gaseous alkalosis
Name the possible reason of the gas alkalosis: c. Non-gaseous alkalosis
a. Pulmonary hyperventilation d. Non-gaseous acidosis
b. Loss of gastric juice e. Gaseous acidosis
c. Loss of intestine juice 458. Effective regulation of ABB is provided with the
d. Pulmonary hypoventilation following physiological systems:
e. Hyperaldosteronism a. lungs, kidneys, liver, GIT
450. What kind of acid-base balance disturbances may b. lungs, kidneys, liver, heart
develop in the patient with gastric ulcer who uses sodium c. adrenals, kidneys, liver, GIT
bicarbonate for relieving pain without control of physician? d. lungs, kidneys, liver, skin
e. lungs, kidneys, liver b. membranogenous
459. What mechanism activation in renal tubules underlies c. disregulation
metabolic alkalosis development in the person with primary d. hypooncotic
hyperaldosteronism? e. hyperosmolar
a. Excessive Na + secretion 465. Inflamation is characterised by increasing penetration of
b. Lack of H+ reabsorption vessels of microcirculation stream, increasing of their fluid
c. Decrease of H+ secretion dynamic blood pressure. Increasing of the osmotic
d. Excessive Na+ reabsorption concentration and dispersity of protein structures present in the
e. Decrease of Na+ secretion intercellular fluid. What kind of edema will appear in this case?
460. What process determines the decrease of Ca2+ plasma a. Mixed
level in the state of metabolic alkalosis? b. Hydrodynamic
a. Ca2+ binding to plasma proteins c. Colloid-osmotic
b. loss of Ca2+ with urine d. Lymphogenic
c. decrease of Ca2+ dietary intake e. Membranogenic
d. increased use of Ca2+ by the osseal tissue 466. A patient who suffers from heart failure has enlarged
e. calcification of inner organs liver, edemata of lower extremities, ascites. What is the leading
461. Depression of nervous system function was found in mechanism in the development of this edema?
the patient with respiratory failure. What is the probable a. Colloid osmotic
mechanism of it? b. Hydrodynamic
a. disturbances in blood buffer systems c. Lymphogenous
b. increased acid intake (drugs) d. Membranogenic
c. increased carbon dioxide outflow 467. A 56 year old patient suffering from cardiac
d. accumulation of carbon dioxide in the blood insufficiency has edema of feet and shins, edematous skin is
e. decreased oxygen inflow pale and cold. What is the leading mechanism of edema
462. A patient was admitted to the infectious department. His pathogenesis?
symptoms: dry skin, decreased skin turgor, rice-water stool. a. Drop of oncotic pessure in capillaries
The patient was diagnosed with cholera. What disorder of b. Rise of hydrostatic pressure in venules
water-electrolyte balance is most often observed in this disease? c. Increase of capillary permeability
a. Hyperosmotic hyperhydration d. Disorder of lymph outflow
b. Hypoosmotic hypohydration e. Positive water balance
c. Isoosmotic hypohydration 468. A patient with low blood albumens content has massive
d. Hyperosmotic hypohydration edema of his face and limbs. What is the leading pathogenetic
e. Hypoosmotic hyperhydration mechanism of edema development?
463. A woman with intractable vomiting was admitted to the a. Increase of vascular permeability
infectious disease ward. What changes of water-salt b. Rise of hydrodynamic blood pressure
metabolism are likely to be observed? c. Drop of oncotic blood pressure
a. Isoosmolar dehydration d. Lymphostasis
b. Hyperosmolar dehydration e. Increase of lymph outflow
c. Hypoosmolar hyperhydration 469. A patient was stung by a bee. Examination revealed that
d. Hypoosmolar dehydration his left hand was hot, pink, edematic, there was a big red blister
e. Hypersmolar hyperhydration on the site of sting. What is the leading mechanism of edema
464. Prolonged consumption of carbohydrate food only by development?
experimental animals caused accumulation of water in the a. Reduced vessel filling
tissues. What is the main pathogenetic mechanism of edema b. Injury of vessels caused by the sting
development in this case? c. Drop of oncotic pressure in tissue
a. lymphogenous d. Drop of osmotic pressure in tissue
e. Increased vessel permeability

Disturbance of lipid and protein metabolism.


470. It is known that gout is most often causing recurrent a. chylomicrons
acute or chronic arthritis. Which of the following substances b. high-density lipoprotein
level in the blood is the leading pathogenic mechanism of this c. very-low-density lipoprotein
disease? d. intermediate-density lipoprotein
a. hyperlipidemia e. low-density lipoprotein
b. hyperproteinemia 473. The knowledge about lipoproteins metabolism is very
c. hyperuricemia important in lipid metabolism maintance study. Which from the
d. hypoproteinemia following substances are synthesized in the liver, and transport
e. hypouricemia triglycerides and cholesterol to peripheral tissues ?
471. The precipitation of monosodium urate crystals in a. chylomicrons
specific tissues underlies the clinical manifestation of the gout. b. high-density lipoprotein
Which tissues are primarily affected? c. very-low-density lipoprotein
a. kidneys d. intermediate-density lipoprotein
b. cartilages and tendons e. low-density lipoprotein
c. vessels and connective tissue 474. The knowledge about lipoproteins metabolism is very
d. lungs and bronchi important in lipid metabolism maintance study. Which from the
e. muscles following substances are the most cholesterol-rich of all
472. The knowledge about lipoproteins metabolism is very lipoproteins?
important in lipid metabolism maintance study. Which from the a. chylomicrons
following substances transport triglycerides and cholesterol b. high-density lipoprotein
from within enterocytes through lymphatics into the circulation? c. very-low-density lipoprotein
d. intermediate-density lipoprotein pathogenic factors plays the leading role in edema development
e. low-density lipoprotein in the case of total starvation?
475. The knowledge about lipoproteins metabolism is very a. proteins deficiency
important in lipid metabolism maintance study. Which from the b. energy deficiency
following substances are initially cholesterol-free and that are c. fatty acids deficiency
synthesized in both enterocytes and the liver? d. vitamins deficiency
a. chylomicrons e. minerals deficiency
b. high-density lipoprotein 484. Which tissues usually have a glucose store in a form of
c. very-low-density lipoprotein glycogen, which can be used for energy needs during the
d. intermediate-density lipoprotein periods of fasting?
e. low-density lipoprotein a. brain and heart
476. The pathways regulating food intake include interaction b. kidneys
between the various substances and nervous centers. Which of c. bone marrow and spleen
the following substances high blood level corresponds to d. liver and muscles
increased body fat amount? e. fatty (adipose tissue)
a. cholecystokinin 485. Which process is characterizing the terminal stage of
b. grelin starvation which is often fatal to the patient?
c. glucagon a. increased glycogenolysis
d. leptin b. depletion of protein stores
e. insulin c. depletion of lipid stores
477. Hypersecretion of which hormones may result in d. depletion of glycogen stores
specific abdominal obesity? e. increased gluconeogenesis
a. glucocorticoids 486. Which disturbance of acid base balance is typical for
b. thyroid hormones starvation process?
c. epinephrine a. gaseous acidosis
d. mineralocorticoids b. metabolic acidosis
e. parathtyroid hormones c. metabolic alkalosis
478. A man 35 years old is 175 cm tall and weighs 95 d. gaseous alkalosis
kilograms. His body mass index is 31. How would you classify e. there are no changes of ABB
his state of nutrition? 487. Why protein-energy undernutrition (cachexia) is often
a. underweight accompanying cancer development?
b. normal a. due to decreased protein intake
c. overweight b. due to decreased carbohydrates intake
d. obesity c. due to prevalence of anabolism
e. extreme obesity d. due to prevalence of catabolism
479. A wom an 25 years old is 162 cm tall and weighs 48 e. due to absence of appetite
kilograms. Her body mass index is 18,3. How would you 488. The main index of protein metabolism is nitrogen
classify her state of nutrition? balance. Choose the condition from the listed below that will be
a. underweight accompanied by positive nitrogen balance
b. normal a. recovering from illness
c. overweight b. physical stress
d. obesity c. emotional stress
e. extreme obesity d. starvation
480. A man 35 years old is 175 cm tall and weighs 76 e. acute infectious disease
kilograms. His body mass index is 24,8. How would you 489. The main index of protein metabolism is nitrogen
classify his state of nutrition? balance. Choose the condition from the listed below that will be
a. underweight accompanied by negative nitrogen balance
b. normal a. childhood
c. overweight b. pregnancy
d. obesity c. massive wound healing
e. extreme obesity d. acute infectious disease
481. A wom an 25 years old is 162 cm tall and weighs 67 490. Give the definition to such condition when a person is
kilograms. Her body mass index is 25,5. How would you totally deprived of food and water is available:
classify her state of nutrition? a. total starvation
a. underweight b. absolute starvation
b. normal c. complete starvation
c. overweight d. incomplete starvation
d. obesity e. partial starvation
e. extreme obesity 491. Early starvation stage is characterized with complete
482. A patient who was starving for a long time has depletion of muscles and liver glycogen stores. The time of
developed edema on the extremities. Which from the given glycogen stores depletion is usually about…
pathogenic factors plays the leading role in edema development a. 1- 10 hours
in the case of total starvation? b. 6-12 hours
a. low hydrostatic pressure of the blood c. 12-24 hours
b. low osmotic pressure of the blood d. 1- 2 days
c. high oncotic pressure in the tissues e. 2-3 days
d. low oncotic pressure of the blood 492. Which stage of starvation is accompanied with the
e. high osmotic pressure of the interstitial liquid maximal body weight loss?
483. A patient who was starving for a long time has a. early starvation
developed edema on the extremities. Which from the given b. prolonged
c. adapted c. general obesity
d. terminal phase d. abdominal obesity
493. Body ’s metabolism is changed during starvation. Which e. peripheral obesity
substance from the following is the source of energy for the 500. Which type of obesity will you suppose in a child 9
brain in the early starvation stage? years, body weight 52 kg.
a. glycogen a. hyperplastic obesity
b. glucose b. hypertrophic obesity
c. ketone bodies c. general obesity
d. amnoacids d. abdominal obesity
e. free fatty acids e. peripheral obesity
494. Body’s metabolism is changed during starvation. 501. Endocrine mechanisms of obesity determine fat
Which substance from the following is the source of energy for accumulation due to abnormalities in hormones metabolism.
the brain in the prologed starvation stage? Which of the following substance deficiency is responsibly for
a. glycogen primary obesity development?
b. glucose a. neuropeptide Y
c. ketone bodies b. leptin
d. amnoacids c. thyroxine
e. free fatty acids d. cortisone
495. The terminal stage of starvation may be fatal for the e. insulin
patient. Which body’s index from the following determines the 502. Endocrine mechanisms of obesity determine fat
time of terminal phase duration and duration? accumulation due to abnormalities in hormones metabolism.
a. blood glucose level Which substance from the listed stimulates appetite and feeding
b. glycogen stores behaviour?
c. protein reserves a. neuropeptide Y
d. lipid reserves b. leptin
e. ketone bodies blood level c. thyroxine
496. Choose the clinical situation in which an abnormally d. cortisone
high level of protein will be found in the patient ’s blood plasma. e. insulin
a. intestinal malabsorption 503. Endocrine mechanisms of obesity determine fat
b. Bruton’s disease accumulation due to abnormalities in hormones metabolism.
c. liver failure Which hormone from the listed determines basal metabolic rate
d. nephrotic syndrome and its deficiency will result in weight gain?
e. cholera a. neuropeptide Y
497. Gout is defined as precipitation of sodium urate crystals b. leptin
in the body. What is the most common cause of monosodium c. thyroxine
urate accumulation in the blood? d. cortisone
a. increased rate of cell’s proliferation e. insulin
b. increased rate of cell’s death 504. A chemical burn caused esophagus stenosis. Difficulty
c. intake of purine-rich food of ingestion led to the abrupt loss of weight. Blood protein
d. overactivity of enzymes responsible for urates synthesis content - 57 g/L. What type of starvation is it?
e. decreased renal excretion of urates a. Proteinic
498. Which typical pathological process primarily develops b. Complete
in the patient with monosodium urate crystals accumulation in c. Incomplete
the joint? d. Water
a. fever e. Absolute
b. inflammation 505. As a rule, during starvation the mass of organs and
c. tumor tissues decreases. Which organ loses the most of its mass
d. hypoxia during the first period of starvation?
e. allergy a. brain
499. Choose obesity type which is more often complicated b. kidneys
with hyperinsulinemia, diabetes mellitus and hypertension c. liver
a. hyperplastic obesity d. heart
b. hypertrophic obesity e. muscles
1. A 12-year-old teenager has significantly put off weight within 3 months; glucose
concentration rose up to 50 mmol/L. He fell into a coma. What is the main mechanism of
its development?
a. *Hyperosmolar
b. Hypoglycemic
c. Ketonemic
d. Lactacidemic
e. Hypoxic
2. The patient with complaints about permanent thirst applied to the doctor. Hyperglycemia,
polyuria and increased concentration of 17-ketosteroids in the urine were revealed. What
disease is the most likely?
a. *Steroid diabetes
b. Insulin-dependent diabetes mellitus
c. Myxedema
d. Type I glycogenosis
e. Addison`s disease
3. Before the cells can utilize the glucoze, it is first transported from the extracellular space
through the plasmatic membrane inside them. This process is stimulated by the following
hormone:
a. *Insulin
b. Glucagon
c. Thyroxin
d. Aldosterone
e. Adrenalin
4. According to the results of glucose tolerance test, the patient has no disorder of
carbohydrate tolerance. Despite that, glucose is detected in the patients`s urine (5
mmol/L). The patient has been diagnosed with renal diabetes. What renal changes cause
glucosuria in this case?
a. *Decreased activity of glucose reabsorption enzymes
b. Increased activity of glucose reabsorption enzymes
c. Exceeded glucose reabsorption threshold
d. Increased glucose secretion
e. Increased glucose filtration
5. Examination of a 56-year-old female patient with a history of type 1 diabetes revealed a
disorder of protein metabolism that is manifested by aminoacidemia in the laboratory
blood test values, and clinically by the delayed wound healing and decreased synthesis of
antibodies. Which of the following mechanisms causes the development of
aminoacidemia?
a. *Increased proteolysis
b. Albuminosis
c. Decrease in the concentration of amino acids in blood
d. Increase in the oncotic pressure in the blood plasma
e. Increase in low-density lipoproteinLevel
6. A patient with diabetes mellitus suffers from persistently nonhealing surgical wound, which
is a sign of disrupted tissue trophism. What is the cause of such disorder?
a. *Disruption of protein metabolism regulation
b. Hypoglycemia
c. Ketonemia
d. Increased lipid catabolism
e. Anemia
7. A 40-year-old woman with Cushing`s disease presents with steroid diabetes. On
biochemical examination she has hyperglycemia and hypochloremia. What process
activates in the first place to such patient?
a. *Gluconeogenesis
b. Glycogenolysis
c. Glucose reabsorption
d. Glucose transport to the cell
e. Glycolysis
8. A woman complains of visual impairment. Examination revealed obesity in the patient and
her fasting plasma glucose level is hyperglycemic. What diabetes complication can cause
visual impairment/blindness?
a. *Microangiopathy
b. Macroangiopathy
c. Atherosclerosis
d. Neuropathy
e. Glomerulopathy
9. A 30-year-old man with diabetes mellitus type I was hospitalised. The patient is comatose.
Laboratory tests revealed hyperglycemia and ketonemia. What metabolic disorder can be
detected in this patient?
a. *Metabolic acidosis
b. Metabolic alkalosis
c. Respiratory acidosis
d. Respiratory alkalosis
e. Normal acid-base balance
10. A 15-year-old patient has fasting plasma glucose level 4,8 mmol/L, one hour after glucose
challenge it becomes 9,0 mmol/L, in 2 hours it is 7,0 mmol/L, in 3 hours it is 4,8 mmol/L.
Such parameters are characteristic of:
a. *Subclinical diabetes mellitus
b. Diabetes mellitus type 1
c. Diabetes mellitus type 2
d. Healthy person
e. Cushing`s disease
11. 2017 2018 A 50-year-old inpatient during examination presents with glucosuria and blood
glucose of 3,0 mmol/L, which are the most likely to be caused by:
a. *Renal disorder
b. Diabetes insipidus
c. Pellagra
d. Myxedema
e. Essential hypertension
12. A newborn child with pylorostenosis has often repeating vomiting accompanied by apathy,
weakness, hypertonicity, sometimes convulsions. What disorder of acid-base balance is
it?
a. *Nongaseous alkalosis
b. Gaseous alkalosis
c. Gaseous acidosis
d. Metabolic acidosis
e. Excretory acidosis
13. An infant has pylorospasm, weakness, hypodynamia, convulsions as a result of frequent
vomiting. What kind of acid-base disbalance is it?
a. *Excretory alkalosis
b. Excretory acidosis
c. Metabolic acidosis
d. Exogenous nongaseous acidosis
e. Gaseous alkalosis
14. 2008 A patient with enteritis accompanied by massive diarrhea has low water rate in the
extracellular space, high water rate inside the cells and low blood osmolarity. What is such
disturbance of water-electrolytic metabolism called?
a. *Hypo-osmolar hypohydration
b. Hyperosmolar hypohydration
c. Osmolar hypohydration
d. Hypo-osmolar hyperhydration
e. Hyperosmolar hyperhydration
15. 2018 A woman with enteritis accompanied by severe diarrhea presents with loss of water
in the extracellular space, increased water content in the cells and decreasing blood
osmolarity. Name this type of water-electrolyte imbalance:
a. *Hypoosmolar hypohydration
b. Hyperosmolar hypohydration
c. Isoosmolar hypohydration
d. Hypoosmolar hyperhydration
e. Hyperosmolar hyperhydration
16. 2014 D A female patient with toxemia of pregnancy has hypersalivation resulting in a daily
loss of 3-4 liters of saliva. What disorder of water-salt metabolism occurs in such cases?
a. *Hyperosmolar hypohydration
b. Hypoosmolar hypohydration
c. Isoosmolar hypohydration
d. Hypokalemia
e. Hyponatremia
17. A 56 year old patient suffering from cardiac insufficiency has edema of feet and shins,
edematous skin is pale and cold. What is the leading mechanism of edema pathogenesis?
a. *Rise of hydrostatic pressure in venules
b. Drop of oncotic pessure in capillaries
c. Increase of capillary permeability
d. Disorder of lymph outflow
e. Positive water balance
18. A 49 year old woman spent a lot of time standing. As a result of it she got leg edema. What
is the most likely cause of the edema?
a. *Increase in hydrostatic pressure of blood in veins
b. Decrease in hydrostatic pressure of blood in veins
c. Decrease in hydrostatic pressure of blood in arteries
d. Increase in oncotic pressure of blood plasma
e. Increase in systemic arterial pressure
19. After taking poor-quality food a patient developed repeated episodes of diarrhea. On the
next day he presented with decreased arterial pressure, tachycardia, extrasystole. Blood
pH is 7,18. These abnormalities were caused by the development of:
a. *Nongaseous acidosis
b. Gaseous acidosis
c. Nongaseous alkalosis
d. Gaseous alkalosis
e. Metabolic alkalosis
20. A patient with diabetes developed a diabetic coma due to the acid-base imbalance. Specify
the kind of this imbalance:
a. *Metabolic acidosis
b. Metabolic alkalosis
c. Respiratory acidosis
d. Gaseous alkalosis
e. Non-gaseous alkalosis
21. 2018 Ketosis develops in the patients with diabetes mellitus as the result of activation of
fatty acids oxidation process. What acid-base imbalance can result from accumulation of
excessive ketone bodies in the blood?
a. *Metabolic acidosis
b. Metabolic alkalosis
c. Respiratory acidosis
d. Respiratory alkalosis
e. No imbalance occurs
22. A patient with respiratory failure has blood pH of 7,35. pCO2 test revealed hypercapnia.
Urine pH test revealed an increase in the urine acidity. What form ofacid-base imbalance
is the case?
a. *Compensated respiratory acidosis
b. Compensated metabolic acidosis
c. Decompensated metabolic acidosis
d. Compensated respiratory alkalosis
e. Decompensated respiratory alkalosis
23. A hypertensive patient had been keeping to a salt-free diet and taking antihypertensive
drugs together with hydrochlorothiazide for a long time. This resulted in electrolyte
imbalance. What disorder of the internal environment occurred in the patient?
a. *Hypochloremic alkalosis
b. Metabolic acidosis
c. Hyperkalemia
d. Hypermagnesemia
e. Increase in circulating blood volume
24. A patient with a pathology of the cardiovascular system developed edemata of the lower
extremities. What is the mechanism of cardiac edema development?
a. *Increased hydrostatic pressure at the venous end of the capillary
b. Increased oncotic pressure
c. Increased hydrostatic pressure at the arterial end of the capillary
d. Reduced osmotic pressure
e. Lymph outflow disorder
25. 2014 A patient has severe blood loss caused by an injury. What kind of dehydration will be
observed in this particular case?
a. *Iso-osmolar
b. Hyposmolar
c. Hyperosmolar
d. Normosmolar
26. A patient suffers from disrupted patency of the airways at the level of small and medium-
sized bronchial tubes. What changes of acid-base balance can occur in the patient?
a. *Respiratory acidosis
b. Respiratory alkalosis
c. Metabolic acidosis
d. Metabolic alkalosis
e. Acid-base balance remains unchanged
27. A patient developed increased blood content of HCO3- against the background of repeated
and uncontrollable vomiting. What will be the leading mechanism in compensation of
developed acid-base imbalance?
a. *Decreased pulmonary ventilation
b. Increased renal reabsorption of bicarbonate
c. Increased pulmonary ventilation
d. Increased renal reabsorption of ammonia
28. 2011 During starvation muscle proteins break up into free amino acids. These compounds
will be the most probably involved into the following process:
a. *Gluconeogenesis in liver
b. Gluconeogenesis in muscles
c. Synthesis of higher fatty acids
d. Glycogenolysis
e. Decarboxylation
29. 2018 A patient with a chemical burn has developed esophageal stenosis. The patient
presents with acute weight loss due to problematic food intake. Blood count: erythrocytes
3,0 x 10*12/L, Hb - 106 g/L, total protein - 57 g/l. What type of starvation does this patient
suffer from?
a. *Incomplete starvation
b. Complete starvation
c. Water starvation
d. Absolute starvation
e. Protein starvation
30. A 2 year old child with mental and physical retardation has been delivered to a hospital. He
presents with frequent vomiting after having meals. There is phenylpyruvic acid in urine.
Which metabolism abnormality is the reason for this pathology?
a. *Amino-acid metabolism
b. Lipid metabolism
c. Carbohydrate metabolism
d. Water-salt metabolism
e. Phosphoric calcium metabolism
31. A 62 year old woman complains of frequent pain attacks in the area of her chest and
backbone, rib fractures. Her doctor suspected myeloma (plasmocytoma). What of the
following laboratory characteristics will be of the greatest diagnostic importance?
a. *Paraproteinemia
b. Hyperalbuminemia
c. Proteinuria
d. Hypoglobulinemia
e. Hypoproteinemia
32. Toxic affection of liver results in dysfunction of protein synthesis. It is usually accompanied
by the following kind of dysproteinemia:
a. *Absolute hypoproteinemia
b. Relative hypoproteinemia
c. Absolute hyperproteinemia
d. Relative hyperproteinemia
e. Paraproteinemia
33. A 12-year-old patient was found to have blood serum cholesterol at the rate of 25 mmol/L.
The boy has a history of hereditary familial hypercholesterolemia, which is caused by the
impaired synthesis of the following protein receptors:
a. *Low density lipoproteins
b. High density lipoproteins
c. Chylomicrons
d. Very low density lipoproteins
e. Intermediate density lipoproteins
34. A 46-year-old female patient consulted a doctor about pain in the small joints of the upper
and lower limbs. The joints are enlarged and shaped like thickened nodes. Serum test
revealed an increase in urate concentration. This might be caused by a disorder in
metabolism of:
a. *Purines
b. Carbohydrates
c. Lipids
d. Pyrimidines
35. A 49-year-old man complains of pain in his metatarsophalangeal jointsand joint
deformation. In blood hyperuricemycan be observed. X-ray has revealed
metatarsophalangeal joint space narrowing, erosion, periarticular calcification of the both
joints, osteoporosis. Microscopy has revealed inflammatory granulomatous reaction
surrounding necrotizing masses in the area of the first metatarsophalangeal joint.Choose
the most likely diagnosis:
a. *Gout (podagra)
b. Pyrophosphate arthropathy
c. Rheumatoid arthritis
d. Hyperparathyroidism
e. Urolithiasis
36. Upon toxic damage of hepatic cells resulting in disruption of liver function the patient
developed edemas. What changes of blood plasma are the main cause of edema
development?
a. *Decrease of albumin content
b. Increase of globulin content
c. Decrease of fibrinogen content
d. Increase of albumin content
e. Decrease of globulin content
37. Prolonged vomiting resulted in dehydration of the patient’s body. Under these conditions
water retention in the body is ensured primarily due to increased secretion of the following
hormone:
a. *Vasopressin
b. Natriuretic hormone
c. Aldosterone
d. Calcitonin
e. Adrenaline
38. A 40-year-old man with impaired venous patency in the lower limbs developed edemas.
What mechanism plays the main role in the development of this disturbance?
a. *Elevated filtration pressure
b. Positive fluid balance
c. Disturbed humoral regulation of water-mineral balance
d. Hypoproteinemia
e. Decreased gradient of osmotic pressure between blood and tissue
39. A 45-year-old woman exhibits no signs of diabetes mellitus, but her fasting blood glucose
level is elevated (7,2 mmol/l). What should be measured next?
a. *Glycated hemoglobin
b. Residual blood nitrogen
c. Blood urea
d. Urine glucose
e. Glucose tolerance
40. A patient with diabetes mellitus after an insulin injection lost his consciousness and
developed convulsions. What will be the result of a biochemical test for blood glucose
level in this case?
a. *2,5 mmol/L
b. 5,5 mmol/L
c. 8,0 mmol/L
d. 3,3 mmol/L
e. 10 mmol/L
41. A 3-year-old child has been brought by ambulance to the intensive care unit of the
infectious diseases hospital. On examination the child is in the severe condition, skin and
mucosa are dry, tissue turgor is reduced. The patient’s history states that profuse diarrhea
and recurrent vomiting were observed during a day after the child had eaten food of poor
quality. What type of salt and water imbalance is likely to have developed in the patient?
a. *Isoosmolar dehydration
b. Hyposmolar hyperhydration
c. Hyposmolar dehydration
d. Hyperosmolar dehydration
e. Hypermolar hyperhydration
42. A patient was hospitalized in a comatose state. The patient has a 5-year-long history of
diabetes mellitus. Objectively: respiration is noisy, deep, with acetone breath odor. Blood
glucose is 15,2 mmmol/L, ketone bodies – 100micromol/L. These signs are characteristic
of the following diabetes complication:
a. *Ketoacidotic coma
b. Hepatic coma
c. Hyperglycemic coma
d. Hypoglycemic coma
e. Hyperosmolar coma
1. A pneumonia patient has been administered acetylcysteine as a part of complex therapy.
What principle of therapy was taken into consideration when applying this drug?
a. *Pathogenetic
b. Symptomatic
c. Ethiotropic
d. Antimicrobial
e. Immunomodulatory
2. A disaster fighter at a nuclear power plant developed hemorrhagic syndrome on the
background of acute radiation disease. What is the most important factor of syndrome
pathogenesis?
a. *Thrombocytopenia
b. Vascular wall damage
c. Increased activity of fibrinolysis factors
d. Increased activity of anticoagulative system factors
e. Decreased activity of coagulative factors
3. A 10-year-old child had the Mantoux tuberculin test administered. 48 hours later a papule up
to 8 mm in diameter appeared on the site of the injection. What type of hypersensitivity
reaction developed after the tuberculin injection?
a. *Type IV hypersensitivity reaction
b. Arthus phenomenon
c. Seroreaction
d. Atopic reaction
e. Type II hypersensitivity reaction
4. During surgical manipulations a patient has been given novocaine injection for anesthesia. 10
minutes later the patient developed paleness, dyspnea, hypotension. What type of allergic
reaction is it?
a. *Anaphylactic immune reaction
b. Cytotoxic immune reaction
c. Aggregate immune reaction
d. Stimulating immune reaction
e. Cell-mediated immune reaction
5. A child with a history of frequent angine and pharyngitis has been diagnosed with
lymphadenopathy and splenomegaly. His appearance is characterised by pastosity and
paleness, muscular tissue is poorly developed. Lymphocytosis is present. What kind of
diathesis is it?
a. *Lymphohypoplastic diathesis
b. Exudative diathesis
c. Gouty diathesis
d. Asthenic diathesis
e. Hemorrhagic diathesis
6. A child is pale, pastose, muscular tissue is bad developed, lymph nodes are enlarged. He
often suffers from angina and pharyngitis, blood has signs of lymphocytosis. The child is also
predisposed to autoallergic diseases. What type of diathesis can be presumed in this case?
a. *Lymphohypoplastic
b. Exudative
c. Gouty
d. Asthenic
e. Hemorrhagic
7. After an immunoassay a child was diagnosed with immunodeficiency of humoral immunity.
What is the reason for the primary immunodeficiency development in the child?
a. *Hereditary abnormality of immune system
b. Embryonal development abnormalities
c. Pathometabolism in mother`s organism
d. Immune responsiveness and resistance disorders
e. Toxic damage of B-lymphocytes
8. Examination of a child who frequently suffers from infectious diseases revealed that IgG
concentration in blood serum was 10 times less than normal, IgA and IgM concentration was
also significantly reduced. Analysis showed also lack of B-lymphocytes and plasmocytes.
What disease are these symptoms typical for?
a. *Bruton`s disease
b. Swiss-type agammaglobulinemia
c. Dysimmunoglobulinemia
d. Louis-Bar syndrome
e. Di George syndrome
9. A patient with clinical signs of immunodeficiency has unchanged number and functional
activity of T and B lymphocytes. Dysfunction’s defect of antigen-presentation to the
immunocompetent cells was found during investigation on the molecule level. Defect of what
cells is the most probable here?
a. *Macrophages, monocytes
b. T-lymphocytes, B-lymphocytes
c. NK-cells
d. Fibroblasts, T-lymphocytes, B-lymphocytes
e. 0-lymphocytes
10. A patient with clinical signs of a primary immunodeficiency has functionally disturbed
mechanism antigen-presentation to the immunocompetent cells. What cells are likely to have
structural defects?
a. *Macrophages, monocytes
b. T-lymphocytes
c. B-lymphocytes
d. Fibroblasts
e. 0-lymphocytes
11. A child with suspected tuberculosis was given Mantoux test. After 24 hours the site of the
allergen injection got swollen, hyperemic and painful. What are the main components that
determine such response of the body?
a. *Mononuclear cells, T-lymphocytes and lymphokines
b. Granulocytes, T-lymphocytes and IgG
c. Plasma cells, T-lymphocytes and lymphokines
d. B-lymphocytes, IgM
e. Macrophages, B-lymphocytes and monocytes
12. A child entering the school for the first time was given Mantoux test in order to determine if
there was a need for revaccination. The reaction was negative. What is the meaning of this
test result?
a. *No cell-mediated immunity to tuberculosis
b. Availability of cell-mediated immunity to tuberculosis
c. No antibodies to the tuberculosis bacteria
d. No anti-toxic immunity to tuberculosis
e. Presence of antibodies to the tuberculosis bacteria
13. A child cut his leg with a piece of glass while playing and was brought to the clinic for the
injection of tetanus toxoid. In order to prevent the development of anaphylactic shock the
serum was administered by Bezredka method. What mechanism underlies this method of
desensitization of the body?
a. *Binding of IgE fixed to the mast cells
b. Blocking the mediator synthesis in the mast cells
c. Stimulation of immune tolerance to the antigen
d. Stimulation of the synthesis of antigenspecificIgG
e. Binding of IgE receptors to the mast Cells
14. 10 days after having quinsy caused by beta-hemolytic streptococcus a 6- year-old child
exhibited symptoms of glomerulonephritis. What mechanism of glomerular lesion is most
likely in this case?
a. *Immunocomplex
b. Cellular cytotoxicity
c. Anaphylaxis
d. Atopy
e. Antibody-dependent cell-mediated cytolysis
15. A 22-year-old woman ate some seafood. 5 hours later the trunk and the distal parts of limbs
got covered with small itchy papules which were partially fused together. After one day, the
rash disappeared spontaneously. Specify the hypersensitivity mechanism underlying these
changes:
a. *Atopy (local anaphylaxis)
b. Systemic anaphylaxis
c. Cellular cytotoxicity
d. Immune complex hypersensitivity
e. Antibody-dependent cell-mediated cytolysis
16. Parents of 5-year-old child report him to have frequent colds that develop into pneumonias,
presence of purulent rashes on the skin. Laboratory tests have revealed the following:
absence of immunoglobulins of any type, and naked cells are absent from the lymph nodes
punctate. What kind of immune disorder is it?
a. *X-linked hypogammaglobulinemia (Bruton type agammaglobulinemia)
b. Autosomal recessive agammaglobulinaemia (Swiss type)
c. Hypoplastic anemia
d. Agranulocytosis
e. Louis-Barr syndrome
17. 30 minutes after drinking mango juice a child suddenly developed a local swelling in the area
of the soft palate, which impeded swallowing and, eventually, respiration. Mucosa of the
swollen area was hyperemic and painless. Blood test revealed moderate eosinophilia. Body
temperature was normal. Anamnesis states that the elder sister of the child has been
suffering from bronchial asthma attacks. What kind of edema has developed in the child?
a. *Allergic
b. Inflammatory
c. Cardiac
d. Alimentary
e. Hepatic
18. Ionizing radiation or vitamin E deficiency affects the cell by increasing lysosome membrane
permeability. What are the possible consequences of this pathology?
a. *Partial or complete cell destruction
b. Formation of maturation spindle
c. Intensive energy production
d. Restoration of cytoplasmic membrane
e. Intensive protein synthesis
19. A 13-year-old boy presents with eczematous rashes on his shin and torso. Anamnesis states
cases of otitis, pneumonia and furuncles in the patient. Blood test: platelets 70x10*9/L, low
activity of T-helpers and T-supressors, low IgM with normal IgA and IgG. What
immunodeficient disease does this boy have?
a. *Wiskott-Aldrich syndrome
b. DiGeorge syndrome
c. Severe combined immunodeficiency (Swiss type)
d. Louis-Barr syndrome
e. Chediak-Higashi syndrome
20. A patient with clinical presentations of immunodeficiency has undergone immunological tests.
They revealed significant decrease in number of cells that form rosettes with sheep
erythrocytes. What conclusion can be done on the ground of the analysis data?
a. *Decrease in T-lymphocyte level
b. Decrease in B-lymphocyte level
c. Decrease in natural killer level (NK-cells)
d. Decrease in complement system level
e. Lack of effector cells of the humoral immunity
21. A child was born with cleft palate. Examination revealed aorta defects and reduced number of
T-lymphocytes in blood. What immunodeficient syndrome is it?
a. *DiGeorge
b. Wiskott-Aldrich
c. Chediak-Higashi
d. Louis-Bar
e. Swiss-type
22. A patient with clinical signs of immunodeficiency has no changes of the number and functional
activity of T- and B- lymphocytes. Defect with dysfunction of antigen-presentation to the
immunocompetent cells was found during examinatio on the molecule level. Defect of what
cells is the most probable?
a. *Macrophages, monocytes
b. T-lymphocytes, B-lymphocytes
c. NK-cells
d. Fibroblasts, T-lymphocytes, B-lymphocytes
e. 0-lymphocytes
23. A woman resting in the countryside has been stung by a bee. Immediately after she developed
pain in the stung area. In a few minutes there developed a vesicle, erythema and intense itch;
later - urticarial and expiratory dyspnea. What factors resulted in the patient developing
expiratory dyspnea?
a. *Histamine
b. Hageman`s factor
c. Lysosomal enzymes
d. Noradrenaline
e. Adrenaline
24. A 5-year-old child is diagnosed with Bruton syndrome (X-linked agammaglobulinemia) that
manifests itself in severe clinical course of bacterial infections and absence of B lymphocytes
and plasma cells. What changes of immunoglobulin content can be observed in blood serum
of the child with immunodeficiency? Manual
a. *Decreased IgA, IgM
b. Increased IgA, IgM
c. Decreased IgD, IgE
d. Increased IgD, IgE
e. No changes
25. During influenza epidemic morbidity in the schoolchildren who did not participate in sports was
40 percents, while in the schoolchildren who engaged in regular physical activities morbidity
did not exceed 20percents. What adaptation mechanism ensured low morbidity in the
physically active schoolchildren? Manual
a. *Cross adaptation
b. Genetic adaptation
c. Physiologic adaptation
d. Specific adaptation
e. Biochemical adaptation
26. A doctor examined a patient, studied blood analysis, and made a conclusion that of peripheral
organs immunogenesis were affected. What organs are the most likely to be affected?
a. *tonsils
b. red bone marrow
c. yellow bone marrow
d. kidneys
e. thymus
27. A 30-year-old patient has dyspnea fits, mostly at night. He has been diagnosed with bronchial
asthma. What type of allergic reaction according to the Gell-Coombs classification is most
likely in this case?
a. *anaphylactic
b. delayed type hypersensitivity
c. cytotoxic
d. stimulating
e. immune complex
28. During blood transfusion a patient has developed intravascular erythrocyte hemolysis. What
type of hypersensitivity does the patient have?
a. *II type (antibody-dependent)
b. IV type (cellular cytotoxicity)
c. I type (anaphylactic)
d. III type (immune complex)
e. IV type (granulomatosis)
29. Several minutes after a dentist administered Novocain for local tooth anesthesia, sudde
fatigue and skin itching developed in the patient. Objectively: skin hyperemia, tachycardia,
drop of BP to 70/40 mm Hg. What type of allergic reaction can lead to this pathology?
a. *anaphylactic
b. immune complex
c. cytotoxic
d. stimulating
e. cell-mediated immune reaction
30. A 12-year-old child has developed nephritic syndrome (proteinuria, hematuria, cylindruria) 2
week after the case of tonsillitis. Nephritic syndrome is the sign of affected glomerular
basement membrane in the kidneys. What mechanism is the most likely to cause the
basement membrane damage?
a. *immune complex
b. granulomatous
c. cytotoxic
d. reaginic
e. antibody-mediated
31. Which condition may develop 15-30 minutes after re-administration of an antigen as a result of
the increased level of antibodies, mainly IgE, that are absorbed on the surface of tissue
basophils (mast cells) and blood basophils?
a. *anaphylaxis
b. immune complex hyperresposiveness
c. antibody-dependent cytotoxicity
d. serum sickness
e. delayed-type hypersensitivity
32. It is known that typical pathological processes develop on the same laws in different organs
and tissues and in different species of animals. Which of the following phenomena can be
attributed to a typical pathological process? Manual
a. *Tumor
b. Tuberculosis
c. Hypertonic disease
d. Intestinal obstruction
e. Myocardial infarction
33. A 49 years old man, who was ill 12 years ago with a rheumatic myocarditis and an
endocarditis, has a mitral valve deficiency. The researches have shown that there is no
inflammatory process in the heart now, the minute volume of blood circulation is sufficient.
What concept of general nosology responds to this condition? Manual
a. *Pathological condition
b. Pathological reaction
c. Pathological process
d. Typical pathological process
e. Compensatory reaction
34. A 60 year old man, as a result of long stay in wet clothes at low temperature, became ill with
croupous pneumonia. What is the cause of this form of lung inflammation? Manual
a. *Pneumococcus
b. Age
c. Reduced reactivity of the body
d. Effect of low temperature on the body
e. Effect of high humidity on the body
35. A man aged 50 years who was treated for stomach ulcer had normalized digestion,
disappearing of pain, improved mood. But in a few weeks, pain in the epigastrium, heartburn
and acid belching appeared again. How can one characterize such manifestation of the
disease?
a. *Relapse of the disease
b. Remission period
c. Terminal condition
d. Prodromal period
e. Latent period
36. During the work on the elimination of the consequences of an accident at the nuclear power
plant, the worker received a radiation dose of 500 roentgens. He complains of headache,
nausea, dizziness. What changes in the number of leukocytes of patient can be expected 10
hours after irradiation? Manual
a. *Neutrophil leukocytosis
b. Lymphocytosis
c. Leukopenia
d. Agranulocytosis
e. Leukemia
37. One of the leading pathogenetic links in the development of pathology caused by ionizing
radiation is the intensification of processes of free radicals oxidation of substances. Which
substances are the main source of free radicals? Manual
a. *Water
b. Lipids
c. Carbohydrates
d. Proteins
e. Ions of metals
38. The irradiation of the patient with a single dose of ionizing radiation caused the development
of the bone-marrow form of the radiation disease. Which variant of blood count will be typical
for the period of apparent well-being? Manual
a. *Increasing lymphopenia, leukopenia
b. Redistributive leukocytosis, lymphocytosis
c. Anemia, leukopenia
d. Thrombocytopenia, anemia
e. Thrombocytopenia, leukocytosis
39. The patient came to the hospital after ionizing radiation with complaints about vomiting,
anorexia, pain in different parts of abdomen, presence of blood in stool, increase of body
temperature, lethargy. For what form of acute radiation sickness this clinical picture is typical?
a. *Intestinal Manual
b. Bone marrow
c. Cerebral
d. Mixed
e. Toxemic
40. Coronary artery thrombosis caused the development of myocardial infarction. What
mechanisms of cardiomyocyte damage are dominant in this pathology?
a. *Calcium Manual
b. Lipid
c. Acidic
d. Electrolyte-osmotic
e. Protein
41. For stomach ulcer modelling, atophan was injected into the gastric arteries of the animal, that
causes sclerosis of gastric arteries. What mechanism of damage of the gastric mucosa is
leading in this experiment? Manual
a. *Hypoxic
b. Neurodystrophic
c. Mechanic
d. Disregulation
e. Neurohumoral
42. A 15-years boy has been diagnosed with acute viral hepatitis. The research of which blood
index should be carried out to confirm acute liver damage? Manual
a. *Activity of aminotransferases [AlT and AST]
b. Free and conjugated bilirubin content
c. Erythrocytes sedimentation rate [ESR].
d. Cholesterol level
e. Content of protein fractions
43. In reperfusion syndrome, processes of free radical oxidation are activated, resulting in damage
to cell membranes and a violation of specific cell functions. These changes are associated
with excessive accumulation in the cytoplasm of which ions? Manual
a. *Calcium
b. Magnesium
c. Chlorine
d. Sodium
e. Potassium
44. In hepatitis, myocardial infarction, the activity of alanine and aspartate aminotransferases
greatly increases in blood plasma of the patient. Which process causes the increase of
activity of these enzymes in the blood? Manual
a. *Damage of cell membranes and release of enzymes in the blood
b. Enhancement of enzymes by hormones
c. Lack of pyridoxine
d. Increased speed of amino acid synthesis in tissues
e. Increase of amino acids catabolism in the tissues
45. In a body of a patient with hypertension a significant increase in the mass of the left ventricule
myocardium was found. This is due to: Manual
a. *Increased volume of cardiomyocytes
b. Increased number of cardiomyocytes
c. Growth of connective tissue
d. Water retention in the myocardium
e. Fat infiltration of the myocardium
46. In experimental animal, an experimental diabetes mellitus was caused by intraperitoneal
injection of alloxone . What is the mechanism of action of this substance? Manual
a. *Damage of beta-cells of pancreatic islets
b. Zinc bounding
c. Formation of antibodies to insulin
d. Activation of insulinase
e. Activation of the production of counter-insulin hormones
47. In case of damage of cell by the ionizing radiation, mechanisms of protection and adaptation
are activated. What is the mechanism of recovery of impaired intracellular homeostasis ?
a. *Activation of the antioxidant system Manual
b. Activation of Ca-mediated cellular functions
c. Accumulation of Na + in the cells
d. Decrease of adenylate cyclase activity
e. Hypertrophy of mitochondria
48. A 52-year-old woman suffering from breast cancer has undergone a course of radiation
therapy. The size of the tumor has decreased. Which of the following mechanisms of damage
of the cell cause the efficiency of radiotherapy the most? Manual
a. *Formation of free radicals
b. Hyperthermia
c. Lyzis by NK-cells
d. Mutagenesis
e. Vessel thrombosis
49. Blood serum analysis of the patient with acute hepatitis shows increased levels of alanine
aminotransferase (ALT) and aspartate aminotransferase (AST). What changes on the cellular
level can result in such findings? Manual
a. *Cell destruction
b. Disturbed energy supply to the cells
c. Disturbed cellular enzyme systems
d. Damage to the genetic apparatus of the cells
e. Disturbed intercellular interactions
50. In order to prevent transplant rejection after organ transplantation, it is necessary to conduct a
course of hormone therapy for immunosuppression. What hormones are used for this
purpose? Manual
a. *Glucocorticoids
b. Mineralocorticoids
c. Sex hormones
d. Catecholamines
e. Thyroid hormones
51. In the research of the condition of the immune system of the patient with chronic fungal skin
lesions, violations of cellular immunity were revealed. Reducing of which indicators are the
most typical for this? Manual
a. *T-lymphocytes
b. Immunoglobulins G
c. Immunoglobulins E
d. B-lymphocytes
e. Plasmocytes
52. A 2-years old child has been diagnosed with thymus hypoplasia. What immune system cell
number change is the most typical for this immunodeficiency? Manual
a. *Decreased number of T-lymphocytes
b. Decreased number of B-lymphocytes
c. Deficiency of T and B-lymphocytes
d. Absence of plasmatic cells
e. Reduction of immunoglobulins M
53. The mice with no hair (i.e., nude) have no cellular reactions of the delayed type. For this
pathology most likely is: Manual
a. *Lack of thymus gland
b. Absence of gammaglobulins in the blood
c. Violation of hemopoiesis
d. Defective phagocytosis
e. Deficiency of components of the complement system
54. The body uses the same mechanisms of the immune system response to the antigen both in
the development of the immune and allergic reactions. Define the main difference between
allergic reactions and the immune response: Manual
a. *development of the damage to the tissues
b. of the amount of antigen present
c. feature of the structure of antigens
d. ways of receiving antigens to the body
e. hereditary predisposition
55. In experiment the nephrocytotoxic serum of guinea pig was injected to rabbit. What kidney
disease was modeled in this experiment? Manual
a. *Acute diffuse glomerulonephritis
b. Nephrotic syndrome
c. Acute pyelonephritis
d. Chronic renal failure
e. Chronic pyelonephritis
56. The patient P., after the trauma, was a need the injection of anti-tetanus serum, but a
sensitivity test for serum was positive. How to conduct hyposensitization in a patient?
Injection: Manual
a. *of small doses of a specific allergen
b. of physiological doses of glucocorticoids
c. of the dissolving dose of a specific allergen
d. of therapeutic doses of antihistamines
e. of decreasing-sensitizing drugs
57. Antileukocytic antibodies are detected in the blood of a patient with leukopenia. What type of
Coombs-Gell hypersensitivity reaction developed in this case? Manual
a. *Cytotoxic
b. Stimulating
c. Immune complex-mediated
d. Anaphylactic
e. Delayed-type hypersensitivity
58. Blood test of the patient revealed albumine content 20g/L and increased activity of lactate
dehydrogenase enzyme. These results indicate disorder of the following organ function:
a. *Liver
b. Heart
c. Lung
d. Spleen
e. Kidneys
59. A patient presents with absence of T-lymphocytes, facial defects, defects of thyroid and
parathyroid glands, heart disease. Cellular immune response do not develop. The patient
was diagnosed with DiGeorge syndrome. This syndrome is caused by:
a. *Thymus hypoplasia
b. Thymus hyperplasia
c. Primary T cell deficiency
d. Primary B cell deficiency
e. Combined immunodeficiency
60. A 5-year-old girl for diagnostic purpose underwent Mantoux tuberculin test. 48 hours later in
the place of tuberculine injection a dense papule 15 mm in diameter with the signs of
hyperemia and necrosis developed. What is the mechanism of hypersensitivity that resulted
in these changestion?
a. *Cellular cytotoxicity
b. Antibody-dependent cytotoxicity
c. Immune complex cytotoxicity
d. Anaphylactic reaction
e. Granulomatosis
1. A patient with chronic heart failure presents with increased blood viscosity. Capillaroscopy
detected damage to the vessel walls of the microcirculation system. What disorder is
possible in the given case?
a. *Blood `sludge` phenomenon
b. Embolism
c. Thrombosis
d. Venous hyperemia
e. Arterial hyperemia
2. 2015 A 54-year-old woman was brought to a casualty department after a car accident. A
traumatologist diagnosed her with multiple fractures of the lower extremities. What kind of
embolism is most likely to develop in this case?
a. *Adipose
b. Tissue
c. Thromboembolism
d. Gaseous
e. Air
3. 2018 A 45-year-old man diagnosed with hepatic cirrhosis and ascites underwent drainage
of 5 liters of fluid from abdominal cavity. It resulted in the development of syncopal state
due to insufficient blood supply to the brain. Which circulatory disorder occurred in the
abdominal cavity in this case?
a. *Arterial hyperemia
b. Venous hyperemia
c. Embolism
d. Thrombosis
e. Ischemia
4. 2018 Coronary artery thrombosis resulted in development of myocardial infarction. What
mechanisms of cell damage are leading in this disease?
a. *Calcium
b. Lipid
c. Acidotic
d. Protein
e. Electroosmotic
5. 2017 2018 A 30-year-old man complains of suffocation, heaviness in the chest on the right,
general weakness. Body temperature is 38,9 o C. Objectively the right side of the chest
lags behind the left side during respiration. Pleurocentesis yielded exudate. What is the
leading factor of exudation in the patient?
a. *Increased permeability of the vessel wall
b. Erythrocyte aggregation
c. Increased blood pressure
d. Decreased resorption of pleural fluid
e. Hypoproteinemia
6. A patient, having suffered a thermal burn, developed painful boils filled with turbid liquid in
the skin. What morphological type of inflammation has developed in the patient?
a. *Serous
b. Proliferative
c. Croupous
d. Granulomatous
e. Diphtheritic
7. 2013 The cellular composition of exudate largely depends on the etiological factor of
inflammation. What leukocytes are the first to get into the focus of inflammation caused by
pyogenic bacteria?
a. *Neutrophil granulocytes
b. Monocytes
c. Myelocytes
d. Eosinophilic granulocytes
e. Basophils
8. 2018 Microscopy of the puncture sample obtained from the inflammation focus of the
patient with cutaneous abscess revealed numerous blood cells of different types. What
cells are the first to transfer from vessels to tissue during inflammation?
a. *Neutrophils
b. Monocytes
c. Lymphocytes
d. Eosinophils
e. Basocytes
9. After transfusion of 200 ml of blood a patient presented with body temperature rise up to
37,9 C. Which of the following substances is the most likely cause of temperature rise?
a. *Interleukin-1
b. Interleukin-2
c. Tumour necrosis factor
d. Interleukin-3
e. Interleukin-4
10. Blood plasma of a healthy man contains several dozens of proteins. During an illness new
proteins can originate, namely the protein of `acute phase`. Select such protein from the
listed below:
a. *C-reactive protein
b. Prothrombin
c. Fibrinogen
d. G immunoglobulin
e. A immunoglobulin
11. A patient has been diagnosed with influenza. His condition became drastically worse after
taking antipyretic drugs. His consciousness is confused, AP is 80/50mm Hg, Ps is 140/m,
body temperature droped down to 35, 8oC. What complication developed in this patient?
a. *Collapse
b. Hyperthermia
c. Hypovolemia
d. Acidosis
e. Alkalosis
12. A 25-year-old man has spent a long time in the sun under high air humidity. As a result of it
his body temperature rose up to 39 oC. What pathological process is it?
a. *Hyperthermia
b. Infectious fever
c. Hypothermia
d. Noninfectious fever
e. Burn disease
13. At the end of the working day a worker of a hot work shop has been delivered to a hospital.
The patient complains of a headache, dizziness, nausea, general weakness. Objectively:
the patient is conscious, his skin is hyperemic, dry, hot to the touch. Heart rate is of
130/min. Respiration is rapid, superficial. What disorder of thermoregulation is most likely
to have occurred in this patient?
a. *Reduced heat loss
b. Increased heat loss and reduced heat production
c. Increased heat loss and heat production
d. Increased heat production with no changes to the heat loss
e. Reduced heat production
14. This year influenza epidemic is characterised by patients` body temperature varying from
36, 9oC to 37, 9oC. Such fever is called:
a. *Subfebrile
b. High
c. Hyperpyretic
d. Apyretic
e. Moderate
15. A patient with lobar pneumonia has had body temperature of 39oC with daily temperature
fluctuation of no more than 1oC for 9 days. This fever can be characterized by the
following temperature curve:
a. *Persistent
b. Hectic
c. Remittent
d. Hyperpyretic
e. Recurrent
16. A patient has acute bronchitis. The fever up to 38, 5oC had lasted for a week, presently
there is a decrease in temperature down to 37, 0oC. Specifythe leading mechanism in the
3rd stage of fever:
a. *Peripheral vasodilation
b. Increased heat production
c. Development of chill
d. Increased diuresis
e. Increased respiratory rate
17. A patient with pneumonia has body temperature of 39,2 o C . What cells are the
main producers of endogenous pyrogen that had caused such temperature rise?
a. *Monocytes
b. Eosinophils
c. Neutrophils
d. Endotheliocytes
e. Fibroblasts
18. A female patient has been diagnosed with cervical erosion, which is a precancerous
pathology. What defense mechanism can prevent the development of a tumor?
a. *Increase in natural killer level (NK cells)
b. High-dose immunological tolerance
c. Increase in the activity of lysosomalenzymes
d. Simplification of the antigenic structure of tissues
e. Low-dose immunological tolerance
19. From the group of children who were eating sweet sappy watermelon two kids developed
the signs of poisoning: rapid weakness, dizziness, headache, vomiting, edema,
tachycardia, cyanosis of mouth, ears, tips of the fingers cyanosis. High concentration of
nitrates was detected. What is the leading mechanism of the pathogenesis of the
poisoning in the two children?
a. *Insufficiency of met-Hb-reductase
b. Insufficiency of superoxiddismutase
c. Block cytochrome oxidase
d. Insufficiency glutathione pyroxidase
e. Insufficiency of catalase
20. Measurements of the arterial pCO2 and pO2 during an attack of bronchial asthma revealed
hypercapnia and hypoxemia respectively. What kind of hypoxia occurred in this case?
a. *Respiratory
b. Hemic
c. Circulatory
d. Tissue
e. Histotoxic
21. Cyanide is a poison that causes instant death of the organism due to fulminant tissue
hypoxia. What enzymes found in mitochondria are affected by cyanide?
a. *Cytochrome oxidase (aa3)
b. Flavin enzymes
c. Cytochrome 5
d. NAD+-dependent dehydrogenase
e. Cytochrome P-450
22. Diseases of respiratory system and circulatory disorders impair the transport of oxygen,
thus causing hypoxia. Under these conditions the energy metabolism is carried out by
anaerobic glycolysis. As a result,the following substance is generated and accumulated in
blood:
a. *Lactic acid
b. Pyruvic acid
c. Glutamic acid
d. Citric acid
e. Fumaric acid
23. 2017 During ascent into mountains a person develops increased respiration rate and rapid
heart rate. What is the cause of these changes?
a. *Decrease of O2 partial pressure
b. Increase of CO2 partial pressure
c. Increase of blood pH
d. Increase of nitrogen content in air
e. Increase of air humidity
24. 2017 2018 A 63-year-old man suffers from esophageal carcinoma, presents with
metastases into the mediastinal lymph nodes and cancerous cachexia. What pathogenetic
stage of neoplastic process is observed in the patient?
a. *Progression
b. Promotion
c. Transformation
d. Initiation
e. -
25. 2018 Due to prolonged stay in the mountains at the altitude 3000 m above the sea level a
person developed increased oxygen capacity of blood, which was directly caused by
intensified production of:
a. *Erythropoietins
b. Catecholamines
c. 2,3 – biphosphoglycerate
d. Carbaminohemoglobin
e. Leukopoietins
26. At an altitude of 14000 m an aircraft experienced a sudden loss of cabin pressure. The
pilot must have developed the following type of embolism:
a. *Gas embolism
b. Embolism with an foreign body
c. Thromboembolism
d. Air
e. Fat
27. During the volleyball game, the sportsman after the jump landed on the outer edge of the
foot. There was acute pain in the ankle joint, active movements in it are limited, passive -
in full, but painful. Then the swelling appeared in the area of the external stone, the skin
reddened, it became warmer to the touch. What type of peripheral circulation disorder has
developed in this case?
a. *Arterial hyperemia.
b. Stasis
c. Embolism
d. Venous hyperemia.
e. Thrombosis.
28. In the experiment, it has been shown that in the case of Jensen's sarcoma, the
consumption of glucose from the incoming arterial vessels tumor increases significantly,
and there is also an increase in the content of lactic acid in the removal vein. What is the
evidence of this phenomenon?
a. *Activation of anaerobic glycolysis
b. Activation of oxidative processes
c. Increasing oxidation of proteins
d. Reduction of anaerobic glycolysis
e. Reduction of oxidative processes
29. It was established that in the development of hepatoma the synthesis of bile acids is often
stopped in it. What kind of anaplasia does this indicate?
a. *functional
b. energy
c. morphological
d. biochemical
e. physico-chemical
30. The resuscitation unit has admitted a patient in grave condition. It is known that he had
mistakenly taken sodium fluoride which blocks cytochrome oxidase. What type of hypoxia
developed in the patient?
a. *Tissue
b. Hemic
c. Cardiovascular
d. Hypoxic
e. Respiratory
31. A public utility specialist went down into a sewer well without protection and after a while
lost consciousness. Ambulance doctors diagnosed him with hydrogen sulfide intoxication.
What type of hypoxia developed?
a. *Hemic
b. Overload
c. Tissue
d. Circulatory
e. Respiratory
32. A patient with obliterating atherosclerosis has undergone a ganglionic sympathectomy of
femoral artery in the region of femoral trigone. Positive therapeutic effect of this surgery is
associated with development of arterial hyperemia of the lower limbs. What type of arterial
hyperemia was induced by the operation?
a. *Neuroparalytic
b. Reactive
c. Metabolic
d. Neurotonic
e. Functional
33. A 42 year old woman with neuralgia of trifacial nerve complains about periodical reddening
of the right part of her face and neck, sense of warmth gush, increased skin sensitivity.
These effects can be explained by the following type of arterial hyperemia:
a. *Neurotonic
b. Neuroparalytic
c. Metabolic
d. Functional
e. Reactive
34. An inflammation can be characterized by hemocapillary dilation in the affected area,
decreased blood circulation and increased vessel wall permeability. What cells play the
key role in this process?
a. *Tissue basophils
b. Fibroblasts
c. Macrophages
d. Plasma cells
e. Eosinophils
35. A 30-year old person has been stung by a bee. The stung area exhibits edema,
hyperemia, and elevated temperature. What is the initial pathogenetic factor of
inflammatory edema in this case?
a. *Increase of microvascular permeability
b. Increase of capillary blood pressure
c. Decrease of oncotic blood pressure
d. Increase of osmotic pressure in the inflammation focus
e. Disturbed lymphatic efflux
36. A man has been working for a long time in oil processing. What type of of carcinogens
does he encounter at his workplace?
a. *Polycyclic aromatic hydrocarbons
b. Amines
c. Nitrosamines
d. Amino-azo compounds
e. Biological carcinogens
37. A 7-year-old child has acute onset of disease: temperature rise up to 38°C, rhinitis, cough,
lacrimation, and large-spot rash on the skin. Pharyngeal mucosa is edematous,
hyperemic, with whitish spots in the buccal area. What kind of inflammation causes the
changes in the buccal mucosa?
a. *Catarrhal inflammation
b. Suppurative inflammation
c. Fibrinous inflammation
d. Hemorrhagic inflammation
e. Serous inflammation
38. The process of collapse progression is associated with nervous system dysfunctions,
disturbed pulmonary gas exchange, and disturbances in the systems of blood and
hemostasis. These signs of collapse are caused by initial development of the following
type of hypoxia:
a. *Circulatory hypoxia
b. Hemic hypoxia
c. Respiratory hypoxia
d. Tissue hypoxia
e. Hypoxic hypoxia
1. A pneumonia patient has been administered acetylcysteine as a part of complex therapy.
What principle of therapy was taken into consideration when applying this drug?
a. *Pathogenetic
b. Symptomatic
c. Ethiotropic
d. Antimicrobial
e. Immunomodulatory
2. A disaster fighter at a nuclear power plant developed hemorrhagic syndrome on the
background of acute radiation disease. What is the most important factor of syndrome
pathogenesis?
a. *Thrombocytopenia
b. Vascular wall damage
c. Increased activity of fibrinolysis factors
d. Increased activity of anticoagulative system factors
e. Decreased activity of coagulative factors
3. A 10-year-old child had the Mantoux tuberculin test administered. 48 hours later a papule up
to 8 mm in diameter appeared on the site of the injection. What type of hypersensitivity
reaction developed after the tuberculin injection?
a. *Type IV hypersensitivity reaction
b. Arthus phenomenon
c. Seroreaction
d. Atopic reaction
e. Type II hypersensitivity reaction
4. During surgical manipulations a patient has been given novocaine injection for anesthesia. 10
minutes later the patient developed paleness, dyspnea, hypotension. What type of allergic
reaction is it?
a. *Anaphylactic immune reaction
b. Cytotoxic immune reaction
c. Aggregate immune reaction
d. Stimulating immune reaction
e. Cell-mediated immune reaction
5. A child with a history of frequent angine and pharyngitis has been diagnosed with
lymphadenopathy and splenomegaly. His appearance is characterised by pastosity and
paleness, muscular tissue is poorly developed. Lymphocytosis is present. What kind of
diathesis is it?
a. *Lymphohypoplastic diathesis
b. Exudative diathesis
c. Gouty diathesis
d. Asthenic diathesis
e. Hemorrhagic diathesis
6. A child is pale, pastose, muscular tissue is bad developed, lymph nodes are enlarged. He
often suffers from angina and pharyngitis, blood has signs of lymphocytosis. The child is also
predisposed to autoallergic diseases. What type of diathesis can be presumed in this case?
a. *Lymphohypoplastic
b. Exudative
c. Gouty
d. Asthenic
e. Hemorrhagic
7. After an immunoassay a child was diagnosed with immunodeficiency of humoral immunity.
What is the reason for the primary immunodeficiency development in the child?
a. *Hereditary abnormality of immune system
b. Embryonal development abnormalities
c. Pathometabolism in mother`s organism
d. Immune responsiveness and resistance disorders
e. Toxic damage of B-lymphocytes
8. Examination of a child who frequently suffers from infectious diseases revealed that IgG
concentration in blood serum was 10 times less than normal, IgA and IgM concentration was
also significantly reduced. Analysis showed also lack of B-lymphocytes and plasmocytes.
What disease are these symptoms typical for?
a. *Bruton`s disease
b. Swiss-type agammaglobulinemia
c. Dysimmunoglobulinemia
d. Louis-Bar syndrome
e. Di George syndrome
9. A patient with clinical signs of immunodeficiency has unchanged number and functional
activity of T and B lymphocytes. Dysfunction’s defect of antigen-presentation to the
immunocompetent cells was found during investigation on the molecule level. Defect of what
cells is the most probable here?
a. *Macrophages, monocytes
b. T-lymphocytes, B-lymphocytes
c. NK-cells
d. Fibroblasts, T-lymphocytes, B-lymphocytes
e. 0-lymphocytes
10. A patient with clinical signs of a primary immunodeficiency has functionally disturbed
mechanism antigen-presentation to the immunocompetent cells. What cells are likely to have
structural defects?
a. *Macrophages, monocytes
b. T-lymphocytes
c. B-lymphocytes
d. Fibroblasts
e. 0-lymphocytes
11. A child with suspected tuberculosis was given Mantoux test. After 24 hours the site of the
allergen injection got swollen, hyperemic and painful. What are the main components that
determine such response of the body?
a. *Mononuclear cells, T-lymphocytes and lymphokines
b. Granulocytes, T-lymphocytes and IgG
c. Plasma cells, T-lymphocytes and lymphokines
d. B-lymphocytes, IgM
e. Macrophages, B-lymphocytes and monocytes
12. A child entering the school for the first time was given Mantoux test in order to determine if
there was a need for revaccination. The reaction was negative. What is the meaning of this
test result?
a. *No cell-mediated immunity to tuberculosis
b. Availability of cell-mediated immunity to tuberculosis
c. No antibodies to the tuberculosis bacteria
d. No anti-toxic immunity to tuberculosis
e. Presence of antibodies to the tuberculosis bacteria
13. A child cut his leg with a piece of glass while playing and was brought to the clinic for the
injection of tetanus toxoid. In order to prevent the development of anaphylactic shock the
serum was administered by Bezredka method. What mechanism underlies this method of
desensitization of the body?
a. *Binding of IgE fixed to the mast cells
b. Blocking the mediator synthesis in the mast cells
c. Stimulation of immune tolerance to the antigen
d. Stimulation of the synthesis of antigenspecificIgG
e. Binding of IgE receptors to the mast Cells
14. 10 days after having quinsy caused by beta-hemolytic streptococcus a 6- year-old child
exhibited symptoms of glomerulonephritis. What mechanism of glomerular lesion is most
likely in this case?
a. *Immunocomplex
b. Cellular cytotoxicity
c. Anaphylaxis
d. Atopy
e. Antibody-dependent cell-mediated cytolysis
15. A 22-year-old woman ate some seafood. 5 hours later the trunk and the distal parts of limbs
got covered with small itchy papules which were partially fused together. After one day, the
rash disappeared spontaneously. Specify the hypersensitivity mechanism underlying these
changes:
a. *Atopy (local anaphylaxis)
b. Systemic anaphylaxis
c. Cellular cytotoxicity
d. Immune complex hypersensitivity
e. Antibody-dependent cell-mediated cytolysis
16. Parents of 5-year-old child report him to have frequent colds that develop into pneumonias,
presence of purulent rashes on the skin. Laboratory tests have revealed the following:
absence of immunoglobulins of any type, and naked cells are absent from the lymph nodes
punctate. What kind of immune disorder is it?
a. *X-linked hypogammaglobulinemia (Bruton type agammaglobulinemia)
b. Autosomal recessive agammaglobulinaemia (Swiss type)
c. Hypoplastic anemia
d. Agranulocytosis
e. Louis-Barr syndrome
17. 30 minutes after drinking mango juice a child suddenly developed a local swelling in the area
of the soft palate, which impeded swallowing and, eventually, respiration. Mucosa of the
swollen area was hyperemic and painless. Blood test revealed moderate eosinophilia. Body
temperature was normal. Anamnesis states that the elder sister of the child has been
suffering from bronchial asthma attacks. What kind of edema has developed in the child?
a. *Allergic
b. Inflammatory
c. Cardiac
d. Alimentary
e. Hepatic
18. Ionizing radiation or vitamin E deficiency affects the cell by increasing lysosome membrane
permeability. What are the possible consequences of this pathology?
a. *Partial or complete cell destruction
b. Formation of maturation spindle
c. Intensive energy production
d. Restoration of cytoplasmic membrane
e. Intensive protein synthesis
19. A 13-year-old boy presents with eczematous rashes on his shin and torso. Anamnesis states
cases of otitis, pneumonia and furuncles in the patient. Blood test: platelets 70x10*9/L, low
activity of T-helpers and T-supressors, low IgM with normal IgA and IgG. What
immunodeficient disease does this boy have?
a. *Wiskott-Aldrich syndrome
b. DiGeorge syndrome
c. Severe combined immunodeficiency (Swiss type)
d. Louis-Barr syndrome
e. Chediak-Higashi syndrome
20. A patient with clinical presentations of immunodeficiency has undergone immunological tests.
They revealed significant decrease in number of cells that form rosettes with sheep
erythrocytes. What conclusion can be done on the ground of the analysis data?
a. *Decrease in T-lymphocyte level
b. Decrease in B-lymphocyte level
c. Decrease in natural killer level (NK-cells)
d. Decrease in complement system level
e. Lack of effector cells of the humoral immunity
21. A child was born with cleft palate. Examination revealed aorta defects and reduced number of
T-lymphocytes in blood. What immunodeficient syndrome is it?
a. *DiGeorge
b. Wiskott-Aldrich
c. Chediak-Higashi
d. Louis-Bar
e. Swiss-type
22. A patient with clinical signs of immunodeficiency has no changes of the number and functional
activity of T- and B- lymphocytes. Defect with dysfunction of antigen-presentation to the
immunocompetent cells was found during examinatio on the molecule level. Defect of what
cells is the most probable?
a. *Macrophages, monocytes
b. T-lymphocytes, B-lymphocytes
c. NK-cells
d. Fibroblasts, T-lymphocytes, B-lymphocytes
e. 0-lymphocytes
23. A woman resting in the countryside has been stung by a bee. Immediately after she developed
pain in the stung area. In a few minutes there developed a vesicle, erythema and intense itch;
later - urticarial and expiratory dyspnea. What factors resulted in the patient developing
expiratory dyspnea?
a. *Histamine
b. Hageman`s factor
c. Lysosomal enzymes
d. Noradrenaline
e. Adrenaline
24. A 5-year-old child is diagnosed with Bruton syndrome (X-linked agammaglobulinemia) that
manifests itself in severe clinical course of bacterial infections and absence of B lymphocytes
and plasma cells. What changes of immunoglobulin content can be observed in blood serum
of the child with immunodeficiency?
a. *Decreased IgA, IgM
b. Increased IgA, IgM
c. Decreased IgD, IgE
d. Increased IgD, IgE
e. No changes
25. During influenza epidemic morbidity in the schoolchildren who did not participate in sports was
40%, while in the schoolchildren who engaged in regular physical activities morbidity did not
exceed 20%. What adaptation mechanism ensured low morbidity in the physically active
schoolchildren?
a. *Cross adaptation
b. Genetic adaptation
c. Physiologic adaptation
d. Specific adaptation
e. Biochemical adaptation
26. A doctor examined a patient, studied blood analysis, and made a conclusion that of peripheral
organs immunogenesis were affected. What organs are the most likely to be affected?
a. *tonsils
b. red bone marrow
c. yellow bone marrow
d. kidneys
e. thymus
27. A 30-year-old patient has dyspnea fits, mostly at night. He has been diagnosed with bronchial
asthma. What type of allergic reaction according to the Gell-Coombs classification is most
likely in this case?
a. *anaphylactic
b. delayed type hypersensitivity
c. cytotoxic
d. stimulating
e. immune complex
28. During blood transfusion a patient has developed intravascular erythrocyte hemolysis. What
type of hypersensitivity does the patient have?
a. *II type (antibody-dependent)
b. IV type (cellular cytotoxicity)
c. I type (anaphylactic)
d. III type (immune complex)
e. IV type (granulomatosis)
29. Several minutes after a dentist administered Novocain for local tooth anesthesia, sudde
fatigue and skin itching developed in the patient. Objectively: skin hyperemia, tachycardia,
drop of BP to 70/40 mm Hg. What type of allergic reaction can lead to this pathology?
a. *anaphylactic
b. immune complex
c. cytotoxic
d. stimulating
e. cell-mediated immune reaction
30. A 12-year-old child has developed nephritic syndrome (proteinuria, hematuria, cylindruria) 2
week after the case of tonsillitis. Nephritic syndrome is the sign of affected glomerular
basement membrane in the kidneys. What mechanism is the most likely to cause the
basement membrane damage?
a. *immune complex
b. granulomatous
c. cytotoxic
d. reaginic
e. antibody-mediated
31. Which condition may develop 15-30 minutes after re-administration of an antigen as a result of
the increased level of antibodies, mainly IgE, that are absorbed on the surface of tissue
basophils (mast cells) and blood basophils?
a. *anaphylaxis
b. immune complex hyperresposiveness
c. antibody-dependent cytotoxicity
d. serum sickness
e. delayed-type hypersensitivity
32. It is known that typical pathological processes develop on the same laws in different organs
and tissues and in different species of animals. Which of the following phenomena can be
attributed to a typical pathological process?
a. *Tumor
b. Tuberculosis
c. Hypertonic disease
d. Intestinal obstruction
e. Myocardial infarction
33. A 49 years old man, who was ill 12 years ago with a rheumatic myocarditis and an
endocarditis, has a mitral valve deficiency. The researches have shown that there is no
inflammatory process in the heart now, the minute volume of blood circulation is sufficient.
What concept of general nosology responds to this condition?
a. *Pathological condition
b. Pathological reaction
c. Pathological process
d. Typical pathological process
e. Compensatory reaction
34. A 60 year old man, as a result of long stay in wet clothes at low temperature, became ill with
croupous pneumonia. What is the cause of this form of lung inflammation?
a. *Pneumococcus
b. Age
c. Reduced reactivity of the body
d. Effect of low temperature on the body
e. Effect of high humidity on the body
35. A man aged 50 years who was treated for stomach ulcer had normalized digestion,
disappearing of pain, improved mood. But in a few weeks, pain in the epigastrium, heartburn
and acid belching appeared again. How can one characterize such manifestation of the
disease?
a. *Relapse of the disease
b. Remission period
c. Terminal condition
d. Prodromal period
e. Latent period
36. During the work on the elimination of the consequences of an accident at the nuclear power
plant, the worker received a radiation dose of 500 roentgens. He complains of headache,
nausea, dizziness. What changes in the number of leukocytes of patient can be expected 10
hours after irradiation?
a. *Neutrophil leukocytosis
b. Lymphocytosis
c. Leukopenia
d. Agranulocytosis
e. Leukemia
37. One of the leading pathogenetic links in the development of pathology caused by ionizing
radiation is the intensification of processes of free radicals oxidation of substances. Which
substances are the main source of free radicals?
a. *Water
b. Lipids
c. Carbohydrates
d. Proteins
e. Ions of metals
38. The irradiation of the patient with a single dose of ionizing radiation caused the development
of the bone-marrow form of the radiation disease. Which variant of blood count will be typical
for the period of apparent well-being?
a. *Increasing lymphopenia, leukopenia
b. Redistributive leukocytosis, lymphocytosis
c. Anemia, leukopenia
d. Thrombocytopenia, anemia
e. Thrombocytopenia, leukocytosis
39. The patient came to the hospital after ionizing radiation with complaints about vomiting,
anorexia, pain in different parts of abdomen, presence of blood in stool, increase of body
temperature, lethargy. For what form of acute radiation sickness this clinical picture is typical?
a. *Intestinal
b. Bone marrow
c. Cerebral
d. Mixed
e. Toxemic
40. Coronary artery thrombosis caused the development of myocardial infarction. What
mechanisms of cardiomyocyte damage are dominant in this pathology?
a. *Calcium
b. Lipid
c. Acidic
d. Electrolyte-osmotic
e. Protein
41. For stomach ulcer modeling, atophan was injected into the gastric arteries of the animal, that
causes sclerosis of gastric arteries. What mechanism of damage of the gastric mucosa is
leading in this experiment?
a. *Hypoxic
b. Neurodystrophic
c. Mechanic
d. Disregulation
e. Neurohumoral
42. A 15-years boy has been diagnosed with acute viral hepatitis. The research of which blood
index should be carried out to confirm acute liver damage?
a. *Activity of aminotransferases [AlT and AST]
b. Free and conjugated bilirubin content
c. Erythrocytes sedimentation rate [ESR].
d. Cholesterol level
e. Content of protein fractions
43. In reperfusion syndrome, processes of free radical oxidation are activated, resulting in damage
to cell membranes and a violation of specific cell functions. These changes are associated
with excessive accumulation in the cytoplasm of which ions?
a. *Calcium
b. Magnesium
c. Chlorine
d. Sodium
e. Potassium
44. In hepatitis, myocardial infarction, the activity of alanine and aspartate aminotransferases
greatly increases in blood plasma of the patient. Which process causes the increase of
activity of these enzymes in the blood?
a. *Damage of cell membranes and release of enzymes in the blood
b. Enhancement of enzymes by hormones
c. Lack of pyridoxine
d. Increased speed of amino acid synthesis in tissues
e. Increase of amino acids catabolism in the tissues
45. In a body of a patient with hypertension a significant increase in the mass of the left ventricule
myocardium was found. This is due to:
a. *Increased volume of cardiomyocytes
b. Increased number of cardiomyocytes
c. Growth of connective tissue
d. Water retention in the myocardium
e. Fat infiltration of the myocardium
46. In experimental animal, an experimental diabetes mellitus was caused by intraperitoneal
injection of alloxone. What is the mechanism of action of this substance?
a. *Damage of beta-cells of pancreatic islets
b. Zinc bounding
c. Formation of antibodies to insulin
d. Activation of insulinase
e. Activation of the production of counter-insulin hormones
47. In case of damage of cell by the ionizing radiation, mechanisms of protection and adaptation
are activated. What is the mechanism of recovery of impaired intracellular homeostasis ?
a. *Activation of the antioxidant system
b. Activation of Ca-mediated cellular functions
c. Accumulation of Na + in the cells
d. Decrease of adenylate cyclase activity
e. Hypertrophy of mitochondria
48. A 52-year-old woman suffering from breast cancer has undergone a course of radiation
therapy. The size of the tumor has decreased. Which of the following mechanisms of damage
of the cell cause the efficiency of radiotherapy the most?
a. *Formation of free radicals
b. Hyperthermia
c. Lyzis by NK-cells
d. Mutagenesis
e. Vessel thrombosis
49. Blood serum analysis of the patient with acute hepatitis shows increased levels of alanine
aminotransferase (ALT) and aspartate aminotransferase (AST). What changes on the cellular
level can result in such findings?
a. *Cell destruction
b. Disturbed energy supply to the cells
c. Disturbed cellular enzyme systems
d. Damage to the genetic apparatus of the cells
e. Disturbed intercellular interactions
50. In order to prevent transplant rejection after organ transplantation, it is necessary to conduct a
course of hormone therapy for immunosuppression. What hormones are used for this
purpose?
a. *Glucocorticoids
b. Mineralocorticoids
c. Sex hormones
d. Catecholamines
e. Thyroid hormones
51. In the research of the condition of the immune system of the patient with chronic fungal skin
lesions, violations of cellular immunity were revealed. Reducing of which indicators are the
most typical for this?
a. *T-lymphocytes
b. Immunoglobulins G
c. Immunoglobulins E
d. B-lymphocytes
e. Plasmocytes
52. A 2-years old child has been diagnosed with thymus hypoplasia. What immune system cell
number change is the most typical for this immunodeficiency?
a. *Decreased number of T-lymphocytes
b. Decreased number of B-lymphocytes
c. Deficiency of T and B-lymphocytes
d. Absence of plasmatic cells
e. Reduction of immunoglobulins M
53. The mice with no hair (i.e., nude) have no cellular reactions of the delayed type. For this
pathology most likely is:
a. *Lack of thymus gland
b. Absence of gammaglobulins in the blood
c. Violation of hemopoiesis
d. Defective phagocytosis
e. Deficiency of components of the complement system
54. The body uses the same mechanisms of the immune system response to the antigen both in
the development of the immune and allergic reactions. Define the main difference between
allergic reactions and the immune response:
a. *development of the damage to the tissues
b. of the amount of antigen present
c. feature of the structure of antigens
d. ways of receiving antigens to the body
e. hereditary predisposition
55. In experiment the nephrocytotoxic serum of guinea pig was injected to rabbit. What kidney
disease was modeled in this experiment?
a. *Acute diffuse glomerulonephritis
b. Nephrotic syndrome
c. Acute pyelonephritis
d. Chronic renal failure
e. Chronic pyelonephritis
56. The patient P., after the trauma, was a need the injection of anti-tetanus serum, but a
sensitivity test for serum was positive. How to conduct hyposensitization in a patient?
Injection:
a. *of small doses of a specific allergen
b. of physiological doses of glucocorticoids
c. of the dissolving dose of a specific allergen
d. of therapeutic doses of antihistamines
e. of decreasing-sensitizing drugs
57. Antileukocytic antibodies are detected in the blood of a patient with leukopenia. What type of
Coombs-Gell hypersensitivity reaction developed in this case?
a. *Cytotoxic
b. Stimulating
c. Immune complex-mediated
d. Anaphylactic
e. Delayed-type hypersensitivity
58. Blood test of the patient revealed albumine content 20g/L and increased activity of lactate
dehydrogenase enzyme. These results indicate disorder of the following organ function:
a. *Liver
b. Heart
c. Lung
d. Spleen
e. Kidneys
59. A patient presents with absence of T-lymphocytes, facial defects, defects of thyroid and
parathyroid glands, heart disease. Cellular immune response do not develop. The patient
was diagnosed with DiGeorge syndrome. This syndrome is caused by:
a. *Thymus hypoplasia
b. Thymus hyperplasia
c. Primary T cell deficiency
d. Primary B cell deficiency
e. Combined immunodeficiency
60. A 5-year-old girl for diagnostic purpose underwent Mantoux tuberculin test. 48 hours later in
the place of tuberculine injection a dense papule 15 mm in diameter with the signs of
hyperemia and necrosis developed. What is the mechanism of hypersensitivity that resulted
in these changes?
a. *Cellular cytotoxicity
b. Antibody-dependent cytotoxicity
c. Immune complex cytotoxicity
d. Anaphylactic reaction
e. Granulomatosis
61. A patient with chronic heart failure presents with increased blood viscosity. Capillaroscopy
detected damage to the vessel walls of the microcirculation system. What disorder is
possible in the given case?
a. *Blood `sludge` phenomenon
b. Embolism
c. Thrombosis
d. Venous hyperemia
e. Arterial hyperemia
62. A 54-year-old woman was brought to a casualty department after a car accident. A
traumatologist diagnosed her with multiple fractures of the lower extremities. What kind of
embolism is most likely to develop in this case?
a. *Adipose
b. Tissue
c. Thromboembolism
d. Gaseous
e. Air
63. A 45-year-old man diagnosed with hepatic cirrhosis and ascites underwent drainage of 5 liters
of fluid from abdominal cavity. It resulted in the development of syncopal state due to
insufficient blood supply to the brain. Which circulatory disorder occurred in the abdominal
cavity in this case?
a. *Arterial hyperemia
b. Venous hyperemia
c. Embolism
d. Thrombosis
e. Ischemia
64. Coronary artery thrombosis resulted in development of myocardial infarction. What
mechanisms of cell damage are leading in this disease?
a. *Calcium
b. Lipid
c. Acidotic
d. Protein
e. Electroosmotic
65. A 30-year-old man complains of suffocation, heaviness in the chest on the right, general
weakness. Body temperature is 38,9 o C. Objectively the right side of the chest lags behind
the left side during respiration. Pleurocentesis yielded exudate. What is the leading factor of
exudation in the patient?
a. *Increased permeability of the vessel wall
b. Erythrocyte aggregation
c. Increased blood pressure
d. Decreased resorption of pleural fluid
e. Hypoproteinemia
66. A patient, having suffered a thermal burn, developed painful boils filled with turbid liquid in the
skin. What morphological type of inflammation has developed in the patient?
a. *Serous
b. Proliferative
c. Croupous
d. Granulomatous
e. Diphtheritic
67. The cellular composition of exudate largely depends on the etiological factor of inflammation.
What leukocytes are the first to get into the focus of inflammation caused by pyogenic
bacteria?
a. *Neutrophil granulocytes
b. Monocytes
c. Myelocytes
d. Eosinophilic granulocytes
e. Basophils
68. Microscopy of the puncture sample obtained from the inflammation focus of the patient with
cutaneous abscess revealed numerous blood cells of different types. What cells are the first
to transfer from vessels to tissue during inflammation?
a. *Neutrophils
b. Monocytes
c. Lymphocytes
d. Eosinophils
e. Basocytes
69. After transfusion of 200 ml of blood a patient presented with body temperature rise up to 37,9
C. Which of the following substances is the most likely cause of temperature rise?
a. *Interleukin-1
b. Interleukin-2
c. Tumour necrosis factor
d. Interleukin-3
e. Interleukin-4
70. Blood plasma of a healthy man contains several dozens of proteins. During an illness new
proteins can originate, namely the protein of `acute phase`. Select such protein from the listed
below:
a. *C-reactive protein
b. Prothrombin
c. Fibrinogen
d. G immunoglobulin
e. A immunoglobulin
71. A patient has been diagnosed with influenza. His condition became drastically worse after
taking antipyretic drugs. His consciousness is confused, AP is 80/50mm Hg, Ps is 140/m,
body temperature droped down to 35, 8oC. What complication developed in this patient?
a. *Collapse
b. Hyperthermia
c. Hypovolemia
d. Acidosis
e. Alkalosis
72. A 25-year-old man has spent a long time in the sun under high air humidity. As a result of it his
body temperature rose up to 39oC. What pathological process is it?
a. *Hyperthermia
b. Infectious fever
c. Hypothermia
d. Noninfectious fever
e. Burn disease
73. At the end of the working day a worker of a hot work shop has been delivered to a hospital.
The patient complains of a headache, dizziness, nausea, general weakness. Objectively: the
patient is conscious, his skin is hyperemic, dry, hot to the touch. Heart rate is of 130/min.
Respiration is rapid, superficial. What disorder of thermoregulation is most likely to have
occurred in this patient?
a. *Reduced heat loss
b. Increased heat loss and reduced heat production
c. Increased heat loss and heat production
d. Increased heat production with no changes to the heat loss
e. Reduced heat production
74. This year influenza epidemic is characterised by patients` body temperature varying from 36,
9oC to 37, 9oC. Such fever is called:
a. *Subfebrile
b. High
c. Hyperpyretic
d. Apyretic
e. Moderate
75. A patient with lobar pneumonia has had body temperature of 39oC with daily temperature
fluctuation of no more than 1oC for 9 days. This fever can be characterized by the following
temperature curve:
a. *Persistent
b. Hectic
c. Remittent
d. Hyperpyretic
e. Recurrent
76. A patient has acute bronchitis. The fever up to 38, 5oC had lasted for a week, presently there
is a decrease in temperature down to 37, 0oC. Specifythe leading mechanism in the 3rd
stage of fever:
a. *Peripheral vasodilation
b. Increased heat production
c. Development of chill
d. Increased diuresis
e. Increased respiratory rate
77. A patient with pneumonia has body temperature of 39,2 o C . What cells are the main
producers of endogenous pyrogen that had caused such temperature rise?
a. *Monocytes
b. Eosinophils
c. Neutrophils
d. Endotheliocytes
e. Fibroblasts
78. A female patient has been diagnosed with cervical erosion, which is a precancerous
pathology. What defense mechanism can prevent the development of a tumor?
a. *Increase in natural killer level (NK cells)
b. High-dose immunological tolerance
c. Increase in the activity of lysosomalenzymes
d. Simplification of the antigenic structure of tissues
e. Low-dose immunological tolerance
79. From the group of children who were eating sweet sappy watermelon two kids developed the
signs of poisoning: rapid weakness, dizziness, headache, vomiting, edema, tachycardia,
cyanosis of mouth, ears, tips of the fingers cyanosis. High concentration of nitrates was
detected. What is the leading mechanism of the pathogenesis of the poisoning in the two
children?
a. *Insufficiency of met-Hb-reductase
b. Insufficiency of superoxiddismutase
c. Block cytochrome oxidase
d. Insufficiency glutathione pyroxidase
e. Insufficiency of catalase
80. Measurements of the arterial pCO2 and pO2 during an attack of bronchial asthma revealed
hypercapnia and hypoxemia respectively. What kind of hypoxia occurred in this case?
a. *Respiratory
b. Hemic
c. Circulatory
d. Tissue
e. Histotoxic
81. Cyanide is a poison that causes instant death of the organism due to fulminant tissue hypoxia.
What enzymes found in mitochondria are affected by cyanide?
a. *Cytochrome oxidase (aa3)
b. Flavin enzymes
c. Cytochrome 5
d. NAD+-dependent dehydrogenase
e. Cytochrome P-450
82. Diseases of respiratory system and circulatory disorders impair the transport of oxygen, thus
causing hypoxia. Under these conditions the energy metabolism is carried out by anaerobic
glycolysis. As a result,the following substance is generated and accumulated in blood:
a. *Lactic acid
b. Pyruvic acid
c. Glutamic acid
d. Citric acid
e. Fumaric acid
83. During ascent into mountains a person develops increased respiration rate and rapid heart
rate. What is the cause of these changes?
a. *Decrease of O2 partial pressure
b. Increase of CO2 partial pressure
c. Increase of blood pH
d. Increase of nitrogen content in air
e. Increase of air humidity
84. A 63-year-old man suffers from esophageal carcinoma, presents with metastases into the
mediastinal lymph nodes and cancerous cachexia. What pathogenetic stage of neoplastic
process is observed in the patient?
a. *Progression
b. Promotion
c. Transformation
d. Initiation
e. -
85. Due to prolonged stay in the mountains at the altitude 3000 m above the sea level a person
developed increased oxygen capacity of blood, which was directly caused by intensified
production of:
a. *Erythropoietins
b. Catecholamines
c. 2,3 – biphosphoglycerate
d. Carbaminohemoglobin
e. Leukopoietins
86. At an altitude of 14000 m an aircraft experienced a sudden loss of cabin pressure. The pilot
must have developed the following type of embolism:
a. *Gas embolism
b. Embolism with an foreign body
c. Thromboembolism
d. Air
e. Fat
87. During the volleyball game, the sportsman after the jump landed on the outer edge of the foot.
There was acute pain in the ankle joint, active movements in it are limited, passive - in full,
but painful. Then the swelling appeared in the area of the external stone, the skin reddened, it
became warmer to the touch. What type of peripheral circulation disorder has developed in
this case?
a. *Arterial hyperemia.
b. Stasis
c. Embolism
d. Venous hyperemia.
e. Thrombosis.
88. In the experiment, it has been shown that in the case of Jensen's sarcoma, the consumption
of glucose from the incoming arterial vessels tumor increases significantly, and there is also
an increase in the content of lactic acid in the removal vein. What is the evidence of this
phenomenon?
a. *Activation of anaerobic glycolysis
b. Activation of oxidative processes
c. Increasing oxidation of proteins
d. Reduction of anaerobic glycolysis
e. Reduction of oxidative processes
89. It was established that in the development of hepatoma the synthesis of bile acids is often
stopped in it. What kind of anaplasia does this indicate?
a. *functional
b. energy
c. morphological
d. biochemical
e. physico-chemical
90. The resuscitation unit has admitted a patient in grave condition. It is known that he had
mistakenly taken sodium fluoride which blocks cytochrome oxidase. What type of hypoxia
developed in the patient?
a. *Tissue
b. Hemic
c. Cardiovascular
d. Hypoxic
e. Respiratory
91. A public utility specialist went down into a sewer well without protection and after a while lost
consciousness. Ambulance doctors diagnosed him with hydrogen sulfide intoxication. What
type of hypoxia developed?
a. *Hemic
b. Overload
c. Tissue
d. Circulatory
e. Respiratory
92. A patient with obliterating atherosclerosis has undergone a ganglionic sympathectomy of
femoral artery in the region of femoral trigone. Positive therapeutic effect of this surgery is
associated with development of arterial hyperemia of the lower limbs. What type of arterial
hyperemia was induced by the operation?
a. *Neuroparalytic
b. Reactive
c. Metabolic
d. Neurotonic
e. Functional
93. A 42 year old woman with neuralgia of trifacial nerve complains about periodical reddening of
the right part of her face and neck, sense of warmth gush, increased skin sensitivity. These
effects can be explained by the following type of arterial hyperemia:
a. *Neurotonic
b. Neuroparalytic
c. Metabolic
d. Functional
e. Reactive
94. An inflammation can be characterized by hemocapillary dilation in the affected area,
decreased blood circulation and increased vessel wall permeability. What cells play the key
role in this process?
a. *Tissue basophils
b. Fibroblasts
c. Macrophages
d. Plasma cells
e. Eosinophils
95. A 30-year old person has been stung by a bee. The stung area exhibits edema, hyperemia,
and elevated temperature. What is the initial pathogenetic factor of inflammatory edema in
this case?
a. *Increase of microvascular permeability
b. Increase of capillary blood pressure
c. Decrease of oncotic blood pressure
d. Increase of osmotic pressure in the inflammation focus
e. Disturbed lymphatic efflux
96. A man has been working for a long time in oil processing. What type of carcinogens does he
encounter at his workplace?
a. *Polycyclic aromatic hydrocarbons
b. Amines
c. Nitrosamines
d. Amino-azo compounds
e. Biological carcinogens
97. A 7-year-old child has acute onset of disease: temperature rise up to 38°C, rhinitis, cough,
lacrimation, and large-spot rash on the skin. Pharyngeal mucosa is edematous, hyperemic,
with whitish spots in the buccal area. What kind of inflammation causes the changes in the
buccal mucosa?
a. *Catarrhal inflammation
b. Suppurative inflammation
c. Fibrinous inflammation
d. Hemorrhagic inflammation
e. Serous inflammation
98. The process of collapse progression is associated with nervous system dysfunctions,
disturbed pulmonary gas exchange, and disturbances in the systems of blood and
hemostasis. These signs of collapse are caused by initial development of the following type of
hypoxia:
a. *Circulatory hypoxia
b. Hemic hypoxia
c. Respiratory hypoxia
d. Tissue hypoxia
e. Hypoxic hypoxia
99. A 12-year-old teenager has significantly put off weight within 3 months; glucose concentration
rose up to 50 mmol/L. He fell into a coma. What is the main mechanism of its development?
a. *Hyperosmolar
b. Hypoglycemic
c. Ketonemic
d. Lactacidemic
e. Hypoxic
100. The patient with complaints about permanent thirst applied to the doctor. Hyperglycemia,
polyuria and increased concentration of 17-ketosteroids in the urine were revealed. What
disease is the most likely?
a. *Steroid diabetes
b. Insulin-dependent diabetes mellitus
c. Myxedema
d. Type I glycogenosis
e. Addison`s disease
101. Before the cells can utilize the glucoze, it is first transported from the extracellular space
through the plasmatic membrane inside them. This process is stimulated by the following
hormone:
a. *Insulin
b. Glucagon
c. Thyroxin
d. Aldosterone
e. Adrenalin
102. According to the results of glucose tolerance test, the patient has no disorder of carbohydrate
tolerance. Despite that, glucose is detected in the patients`s urine (5 mmol/L). The patient has
been diagnosed with renal diabetes. What renal changes cause glucosuria in this case?
a. *Decreased activity of glucose reabsorption enzymes
b. Increased activity of glucose reabsorption enzymes
c. Exceeded glucose reabsorption threshold
d. Increased glucose secretion
e. Increased glucose filtration
103. Examination of a 56-year-old female patient with a history of type 1 diabetes revealed a
disorder of protein metabolism that is manifested by aminoacidemia in the laboratory blood
test values, and clinically by the delayed wound healing and decreased synthesis of
antibodies. Which of the following mechanisms causes the development of aminoacidemia?
a. *Increased proteolysis
b. Albuminosis
c. Decrease in the concentration of amino acids in blood
d. Increase in the oncotic pressure in the blood plasma
e. Increase in low-density lipoproteinLevel
104. A patient with diabetes mellitus suffers from persistently nonhealing surgical wound, which is a
sign of disrupted tissue trophism. What is the cause of such disorder?
a. *Disruption of protein metabolism regulation
b. Hypoglycemia
c. Ketonemia
d. Increased lipid catabolism
e. Anemia
105. A 40-year-old woman with Cushing`s disease presents with steroid diabetes. On biochemical
examination she has hyperglycemia and hypochloremia. What process activates in the first
place to such patient?
a. *Gluconeogenesis
b. Glycogenolysis
c. Glucose reabsorption
d. Glucose transport to the cell
e. Glycolysis
106. A woman complains of visual impairment. Examination revealed obesity in the patient and her
fasting plasma glucose level is hyperglycemic. What diabetes complication can cause visual
impairment/blindness?
a. *Microangiopathy
b. Macroangiopathy
c. Atherosclerosis
d. Neuropathy
e. Glomerulopathy
107. A 30-year-old man with diabetes mellitus type I was hospitalised. The patient is comatose.
Laboratory tests revealed hyperglycemia and ketonemia. What metabolic disorder can be
detected in this patient?
a. *Metabolic acidosis
b. Metabolic alkalosis
c. Respiratory acidosis
d. Respiratory alkalosis
e. Normal acid-base balance
108. A 15-year-old patient has fasting plasma glucose level 4,8 mmol/L, one hour after glucose
challenge it becomes 9,0 mmol/L, in 2 hours it is 7,0 mmol/L, in 3 hours it is 4,8 mmol/L. Such
parameters are characteristic of:
a. *Subclinical diabetes mellitus
b. Diabetes mellitus type 1
c. Diabetes mellitus type 2
d. Healthy person
e. Cushing`s disease
109. A 50-year-old inpatient during examination presents with glucosuria and blood glucose of 3,0
mmol/L, which are the most likely to be caused by:
a. *Renal disorder
b. Diabetes insipidus
c. Pellagra
d. Myxedema
e. Essential hypertension
110. A newborn child with pylorostenosis has often repeating vomiting accompanied by apathy,
weakness, hypertonicity, sometimes convulsions. What disorder of acid-base balance is it?
a. *Nongaseous alkalosis
b. Gaseous alkalosis
c. Gaseous acidosis
d. Metabolic acidosis
e. Excretory acidosis
111. An infant has pylorospasm, weakness, hypodynamia, convulsions as a result of frequent
vomiting. What kind of acid-base disbalance is it?
a. *Excretory alkalosis
b. Excretory acidosis
c. Metabolic acidosis
d. Exogenous nongaseous acidosis
e. Gaseous alkalosis
112. A patient with enteritis accompanied by massive diarrhea has low water content in the
extracellular space, high water content inside the cells and low blood osmolarity. Name this
type of water-electrolyte metabolism imbalance:
a. *Hypoosmolar hypohydration
b. Hyperosmolar hypohydration
c. Osmolar hypohydration
d. Hypo-osmolar hyperhydration
e. Hyperosmolar hyperhydration
113. D A female patient with toxemia of pregnancy has hypersalivation resulting in a daily loss of 3-
4 liters of saliva. What disorder of water-salt metabolism occurs in such cases?
a. *Hyperosmolar hypohydration
b. Hypoosmolar hypohydration
c. Isoosmolar hypohydration
d. Hypokalemia
e. Hyponatremia
114. A 56 year old patient suffering from cardiac insufficiency has edema of feet and shins,
edematous skin is pale and cold. What is the leading mechanism of edema pathogenesis?
a. *Rise of hydrostatic pressure in venules
b. Drop of oncotic pessure in capillaries
c. Increase of capillary permeability
d. Disorder of lymph outflow
e. Positive water balance
115. A 49 year old woman spent a lot of time standing. As a result of it she got leg edema. What is
the most likely cause of the edema?
a. *Increase in hydrostatic pressure of blood in veins
b. Decrease in hydrostatic pressure of blood in veins
c. Decrease in hydrostatic pressure of blood in arteries
d. Increase in oncotic pressure of blood plasma
e. Increase in systemic arterial pressure
116. After taking poor-quality food a patient developed repeated episodes of diarrhea. On the next
day he presented with decreased arterial pressure, tachycardia, extrasystole. Blood pH is
7,18. These abnormalities were caused by the development of:
a. *Nongaseous acidosis
b. Gaseous acidosis
c. Nongaseous alkalosis
d. Gaseous alkalosis
e. Metabolic alkalosis
117. A patient with diabetes developed a diabetic coma due to the acid-base imbalance. Specify
the kind of this imbalance:
a. *Metabolic acidosis
b. Metabolic alkalosis
c. Respiratory acidosis
d. Gaseous alkalosis
e. Non-gaseous alkalosis
118. Ketosis develops in the patients with diabetes mellitus as the result of activation of fatty acids
oxidation process. What acid-base imbalance can result from accumulation of excessive
ketone bodies in the blood?
a. *Metabolic acidosis
b. Metabolic alkalosis
c. Respiratory acidosis
d. Respiratory alkalosis
e. No imbalance occurs
119. A patient with respiratory failure has blood pH of 7,35. pCO2 test revealed hypercapnia. Urine
pH test revealed an increase in the urine acidity. What form ofacid-base imbalance is the
case?
a. *Compensated respiratory acidosis
b. Compensated metabolic acidosis
c. Decompensated metabolic acidosis
d. Compensated respiratory alkalosis
e. Decompensated respiratory alkalosis
120. A hypertensive patient had been keeping to a salt-free diet and taking antihypertensive drugs
together with hydrochlorothiazide for a long time. This resulted in electrolyte imbalance. What
disorder of the internal environment occurred in the patient?
a. *Hypochloremic alkalosis
b. Metabolic acidosis
c. Hyperkalemia
d. Hypermagnesemia
e. Increase in circulating blood volume
121. A patient with a pathology of the cardiovascular system developed edemata of the lower
extremities. What is the mechanism of cardiac edema development?
a. *Increased hydrostatic pressure at the venous end of the capillary
b. Increased oncotic pressure
c. Increased hydrostatic pressure at the arterial end of the capillary
d. Reduced osmotic pressure
e. Lymph outflow disorder
122. A patient has severe blood loss caused by an injury. What kind of dehydration will be
observed in this particular case?
a. *Iso-osmolar
b. Hyposmolar
c. Hyperosmolar
d. Normosmolar
123. A patient suffers from disrupted patency of the airways at the level of small and medium-sized
bronchial tubes. What changes of acid-base balance can occur in the patient?
a. *Respiratory acidosis
b. Respiratory alkalosis
c. Metabolic acidosis
d. Metabolic alkalosis
e. Acid-base balance remains unchanged
124. A patient developed increased blood content of HCO3- against the background of repeated
and uncontrollable vomiting. What will be the leading mechanism in compensation of
developed acid-base imbalance?
a. *Decreased pulmonary ventilation
b. Increased renal reabsorption of bicarbonate
c. Increased pulmonary ventilation
d. Increased renal reabsorption of ammonia
125. During starvation muscle proteins break up into free amino acids. These compounds will be
the most probably involved into the following process:
a. *Gluconeogenesis in liver
b. Gluconeogenesis in muscles
c. Synthesis of higher fatty acids
d. Glycogenolysis
e. Decarboxylation
126. A patient with a chemical burn has developed esophageal stenosis. The patient presents with
acute weight loss due to problematic food intake. Blood count: erythrocytes 3,0 x 10*12/L, Hb
- 106 g/L, total protein - 57 g/l. What type of starvation does this patient suffer from?
a. *Incomplete starvation
b. Complete starvation
c. Water starvation
d. Absolute starvation
e. Protein starvation
127. A 2 year old child with mental and physical retardation has been delivered to a hospital. He
presents with frequent vomiting after having meals. There is phenylpyruvic acid in urine.
Which metabolism abnormality is the reason for this pathology?
a. *Amino-acid metabolism
b. Lipid metabolism
c. Carbohydrate metabolism
d. Water-salt metabolism
e. Phosphoric calcium metabolism
128. A 62 year old woman complains of frequent pain attacks in the area of her chest and
backbone, rib fractures. Her doctor suspected myeloma (plasmocytoma). What of the
following laboratory characteristics will be of the greatest diagnostic importance?
a. *Paraproteinemia
b. Hyperalbuminemia
c. Proteinuria
d. Hypoglobulinemia
e. Hypoproteinemia
129. Toxic affection of liver results in dysfunction of protein synthesis. It is usually accompanied by
the following kind of dysproteinemia:
a. *Absolute hypoproteinemia
b. Relative hypoproteinemia
c. Absolute hyperproteinemia
d. Relative hyperproteinemia
e. Paraproteinemia
130. A 12-year-old patient was found to have blood serum cholesterol at the rate of 25 mmol/L. The
boy has a history of hereditary familial hypercholesterolemia, which is caused by the impaired
synthesis of the following protein receptors:
a. *Low density lipoproteins
b. High density lipoproteins
c. Chylomicrons
d. Very low density lipoproteins
e. Intermediate density lipoproteins
131. A 46-year-old female patient consulted a doctor about pain in the small joints of the upper and
lower limbs. The joints are enlarged and shaped like thickened nodes. Serum test revealed
an increase in urate concentration. This might be caused by a disorder in metabolism of:
a. *Purines
b. Carbohydrates
c. Lipids
d. Pyrimidines
132. A 49-year-old man complains of pain in his metatarsophalangeal jointsand joint deformation.
In blood hyperuricemycan be observed. X-ray has revealed metatarsophalangeal joint space
narrowing, erosion, periarticular calcification of the both joints, osteoporosis. Microscopy has
revealed inflammatory granulomatous reaction surrounding necrotizing masses in the area of
the first metatarsophalangeal joint.Choose the most likely diagnosis:
a. *Gout (podagra)
b. Pyrophosphate arthropathy
c. Rheumatoid arthritis
d. Hyperparathyroidism
e. Urolithiasis
133. Upon toxic damage of hepatic cells resulting in disruption of liver function the patient
developed edemas. What changes of blood plasma are the main cause of edema
development?
a. *Decrease of albumin content
b. Increase of globulin content
c. Decrease of fibrinogen content
d. Increase of albumin content
e. Decrease of globulin content
134. Prolonged vomiting resulted in dehydration of the patient’s body. Under these conditions water
retention in the body is ensured primarily due to increased secretion of the following hormone:
a. *Vasopressin
b. Natriuretic hormone
c. Aldosterone
d. Calcitonin
e. Adrenaline
135. A 40-year-old man with impaired venous patency in the lower limbs developed edemas. What
mechanism plays the main role in the development of this disturbance?
a. *Elevated filtration pressure
b. Positive fluid balance
c. Disturbed humoral regulation of water-mineral balance
d. Hypoproteinemia
e. Decreased gradient of osmotic pressure between blood and tissue
136. A patient with pollinosis after a travel to the countryside developed edemas of lips and eyelids,
lacrimation, nasal discharge, and a burning sensation in the eyes. What is the main
mechanism of edema development in this case?
a. *Increased permeability of the capillaries
b. Increased capillary hydrostatic pressure
c. Increased blood oncotic pressure
d. Increased interstitial oncotic pressure
e. Disturbed lymph efflux
137. A 45-year-old woman exhibits no signs of diabetes mellitus, but her fasting blood glucose level
is elevated (7,2 mmol/l). What should be measured next?
a. *Glycated hemoglobin
b. Residual blood nitrogen
c. Blood urea
d. Urine glucose
e. Glucose tolerance
138. A patient with diabetes mellitus after an insulin injection lost his consciousness and developed
convulsions. What will be the result of a biochemical test for blood glucose level in this case?
a. *2,5 mmol/L
b. 5,5 mmol/L
c. 8,0 mmol/L
d. 3,3 mmol/L
e. 10 mmol/L
139. A 3-year-old child has been brought by ambulance to the intensive care unit of the infectious
diseases hospital. On examination the child is in the severe condition, skin and mucosa are
dry, tissue turgor is reduced. The patient’s history states that profuse diarrhea and recurrent
vomiting were observed during a day after the child had eaten food of poor quality. What type
of salt and water imbalance is likely to have developed in the patient?
a. *Hypoosmolar dehydration
b. Isoosmolar dehydration
c. Hypoosmolar hyperhydration
d. Hypersmolar dehydration
e. Hyperosmolar hyperhydration
140. After a case of severe infectious disease the patient developed signs of diabetes insipidus,
which was indicated by daily urine output increased up to 10 liters. What mechanism of
dehydration development is leading in this case?
a. *Decreased renal reabsorption of water
b. Decreased renal reabsorption of sodium
c. Decreased plasma oncotic pressure
d. Inhibited intestinal absorption of water
e. Increased osmolarity of ultrafiltrate
141. A patient was hospitalized in a comatose state. The patient has a 5-year-long history of
diabetes mellitus. Objectively: respiration is noisy, deep, with acetone breath odor. Blood
glucose is 15,2 mmmol/L, ketone bodies – 100micromol/L. These signs are characteristic of
the following diabetes complication:
a. *Ketoacidotic coma
b. Hepatic coma
c. Hyperglycemic coma
d. Hypoglycemic coma
e. Hyperosmolar coma
142. Blood count of an athlete is as follows: erythrocytes - 5, 5 · 10*12/L, Hb-180 g/L, leukocytes -
7 · 10*9/L, neutrophils -64%, basophils - 0,5%, eosinophils - 0,5%,monocytes - 8%,
lymphocytes - 27%. First of all, such results indicate the stimulation of:
a. *Erythropoiesis
b. Leukopoiesis
c. Lymphopoiesis
d. Granulocytopoiesis
e. Immunogenesis
143. A 3-year-old boy with pronounced hemorrhagic syndrome doesn’t have antihemophilic globulin
A (factor VIII) in the blood plasma. Hemostasis has been impaired at the following stage:
a. *Internal mechanism of prothrombinase activation
b. External mechanism of prothrombinase activation
c. Conversion of prothrombin to thrombin
d. Conversion of fibrinogen to fibrin
e. Blood clot retraction
144. Examination of a 52-year-old female patient has revealed a decrease in the amount of red
blood cells and an increase in free hemoglobin in the blood plasma (hemoglobinemia). Color
index is 0,85. What type of anemia is being observed in the patient?
a. *Acquired hemolytic
b. Hereditary hemolytic
c. Acute hemorrhagic
d. Chronic hemorrhagic
e. Anemia due to diminished erythropoiesis
145. A patient is diagnosed with irondeficiency sideroachrestic anemia, progression of which is
characterized by skin hyperpigmentation, pigmentary cirrhosis, heart and pancreas affection.
Iron level in the blood serum is increased. What disorder of iron metabolism causes this
disease?
a. *Failure to assimilate iron leading to iron accumulation in tissues
b. Excessive iron intake with food
c. Disorder of iron absorption in bowels
d. Increased iron assimilation by body
146. A 30-year-old patient’s blood test revealed the following: erythrocyte count is 6 · 10*2/L,
hemoglobin is 10,55 mmol/L Vaquez’s disease was diagnosed. Name the leading part of
pathogenesis:
a. *Neoplastic erythroid hyperplasia
b. Iron-deficiency
c. B12-deficiency
d. Hypoxia
e. Acidosis
147. A 37-year-old female patient complains of headache, vertigo, troubled sleep, numbness of
limbs. For the last 6 years she has been working at the gas-discharge lamp-producing factory
in the lead-processing shop. Blood test findings: low hemoglobin and RBC level, serum iron
concentration exceeds the norm by several times. Specify the type of anemia:
a. *Iron refractory anemia
b. Iron-deficiency anemia
c. Minkowsky-Shauffard disease
d. Hypoplastic anemia
e. Metaplastic anemia
148. A woman has come to the hospital with complaints about genral weakness, dizziness and
dyspnea. Recently she has been taking levomycetin (chloramphinicol) for prevention of
enteric infection. Blood test: erythrocytes – 1,9 × 10*12/L, Hb – 58 g/L, color index – 0,9;
leukocytes – 2,2G/L; reticulocytes – 0,3%. What type of anemia is it indicative of?
a. *Hypoplastic
b. Metaplastic
c. Iron-deficiency
d. Hemolytic
e. Aplastic
149. A patient is diagnosed with chronic atrophic gastritis attended by deficiency of Castle’s intrinsic
factor. What type of anemia does the patient have?
a. *B12-deficiency anemia
b. Iron refractory anemia
c. Hemolytic anemia
d. Iron-deficiency anemia
e. Protein-deficiency anemia
150. A15 year old girl has pale skin, glossitis, gingivitis. Blood count: erythrocytes – 3,3x10*12/L,
hemoglobin – 70 g/L, colour index - 0,5. Examination of blood smear revealed hypochromia,
microcytosis, poikilocytosis. What type of anemia is it?
a. *Iron-deficient
b. B12-folic acid-deficient
c. Sickle-cell
d. Hemolytic
e. Thalassemia
151. Biochemical analysis of an infant’s erythrocytes revealed evident glutathioneperoxidase
deficiency and low concentration of reduced glutathione. What pathological condition can
develop in this infant?
a. *Hemolytic anemia
b. Pernicious anemia
c. Megaloblastic anemia
d. Sicklemia
e. Iron-deficiency anemia
152. A tooth extraction in a patient with chronic persistent hepatitis was complicated with prolonged
hemorrhage.What is the reason for the haemorrhagic syndrome?
a. *Decrease in thrombin production
b. Increase in thromboplastin production
c. Decrease in fibrin production
d. Increase in fibrinogen synthesis
e. Fibrinolysis intensification
153. A patient visited a dentist to extract a tooth. After the tooth had been extracted, bleeding from
the tooth socket continued for 15 minutes. Anamnesis states that the patient suffers from
active chronic hepatitis. What phenomenon can extend the time of hemorrhage?
a. *Decrease of fibrinogen content in blood
b. Thrombocytopenia
c. Hypocalcemia
d. Increased activity of anticoagulation system
e. Decrease of albumine content in blood
154. A 43-year-old-woman against the background of septic shock developed thrombocytopenia;
her blood fibrinogen levels are low; fibrin degradation products appeared. The patient
developed petechial hemorrhages. What is the cause of these changes?
a. *DIC syndrome
b. Platelet production disorder
c. Autoimmune thrombocytopenia
d. Exogenous intoxication
e. Hemorrhagic diathesis
155. A 3-year-old child has eaten some strawberries. Soon he developed a rash and itching. What
was found in the child’s leukogram?
a. *Eosinophilia
b. Lymphocytosis
c. Hypolymphemia
d. Neutrophilic leucocytosis
e. Monocytosis
156. A 59-year-old woman has been hospitalized in a surgical ward due to exacerbation of chronic
osteomyelitis of the left shin. Blood test: leukocytes - 15, 0 · 10*9/L. Leukogram: myelocytes -
0%, metamyelocytes - 8%, stab neutrophils - 28%, segmented neutrophils - 32%,
lymphocytes - 29%, monocytes - 3%. Such blood count would be called:
a. *Regenerative left shift
b. Right shift
c. Hyperregenerative left shift
d. Degenerative left shift
e. Regenerative-degenerative left shift
157. A 21-year-old patient complains of weakness, fever up to 38oC - 40oC. Objectively: the liver
and spleen are enlarged, lymph nodes on palpation are slightly enlarged, dense, painless.
Blood test: Нb- 100 g/L;erythrocytes - 2,9 x10*12/L; leukocytes -4, 4 x 10*9/L, platelets – 48 x
10*9/L, segmented neutrophils -17%,lymphocytes – 15%, blast cells - 68 All cytochemical
reactions are negative (to glycogen, peroxidase, non-specific esterase, lipids). What
hematological conclusion can be made?
a. *Acute undifferentiated leukemia
b. Acute myeloid leukemia
c. Acute monoblastic leukemia
d. Acute lymphoblastic leukemia
e. Acute megakaryoblastic leukemia
158. A 19-year-old female patient has had low haemoglobin rate of 90-95 g/L since childhood.
Blood count results obtained after hospitalisation are as follows: erythrocytes - 3,2*10*12/L,
Hb- 85 g/L, colour index - 0,78; leukocytes - 5,6 * 10*9/L, platelets - 210 *10*9/L. Smear
examination revealed anisocytosis, poikilocytosis and target cells. Reticulocyte rate is 6%.
Iron therapy was ineffective. What blood pathology corresponds with the described clinical
presentations?
a. *Thalassemia
b. Enzymopathy
c. Membranopathy
d. Sickle-cell anemia
e. Favism
159. A 26-year-old man is in the torpid shock phase as a result of a car accident. In blood:
leukocytes - 3, 2 · 10*9/L. What is the leading mechanism of leukopenia development?
a. *Redistribution of leukocytes in bloodstream
b. Leikopoiesis inhibition
c. Disturbed going out of mature leukocytes from the marrow into the blood
d. Lysis of leukocytes in the blood-forming organs
e. Intensified elimination of leukocytes from the organism
160. After a tourniquet application a patient was found to have petechial haemorrhages. The
reason for it is the dysfunction of the following cells:
a. *Platelets
b. Eosinophils
c. Monocytes
d. Lymphocytes
e. Neutrophils
161. After an attack of bronchial asthma a patient had his peripheral blood tested. What changes
can be expected?
a. *Eosinophilia
b. Leukopenia
c. Lymphocytosis
d. Thrombocytopenia
e. Erythrocytosis
162. A patient suffering from chronic myeloleukemia has got the following symptoms of anemia:
decreased number of erythrocytes and low haemoglobin concentration, oxyphilic and
polychromatophilic normocytes, microcytes. What is the leading pathogenetic mechanism of
anemia development?
a. *Substitution of haemoblast
b. Intravascular hemolysis of erythrocytes
c. Deficiency of vitamin B12
d. Reduced synthesis of erythropoietin
e. Chronic haemorrhage
163. Blood test of a patient suffering from atrophic gastritis gave the following results: RBC - 2, 0 ·
1012/L, Hb- 87 g/L, colour index - 1,3, WBC - 4, 0 · 10*9/L, thrombocytes - 180 · 10*9/L.
Anaemia migh have been caused by the following substance deficiency:
a. *Vitamin B12
b. Vitamin A
c. Vitamin K
d. Iron
e. Zinc
164. On the fifth day after the acute blood loss a patient has been diagnosed with hypochromic
anemia. What is the main mechanism of hypochromia development?
a. *Release of immature red blood cells from the bone marrow
b. Impaired iron absorption in the intestines
c. Increased destruction of red blood cells in the spleen
d. Impaired globin synthesis
e. Increased excretion of body iron
165. A 12-year-old patient has been admitted to a hospital for hemarthrosis of the knee joint. From
early childhood he suffers from frequent bleedings. Diagnose the boy’s disease:
a. *Hemophilia
b. Hemorrhagic vasculitis
c. Hemolytic anemia
d. B12 (folic acid)-deficiency anemia
e. Thrombocytopenic purpura
166. In a traffic accident a man suffered a trauma with massive blood loss. What changes in the
peripheral blood are most likely on the 2nd day after the trauma?
a. *Erythropenia
b. Hypochromia
c. Anisocytosis
d. Microcytosis
e. Significant reticulocytosis
167. After the prolonged vomiting a pregnant 26-year-old woman was found to have the reduced
volume of circulating blood. What change in the total blood volume can be the case?
a. *Polycythemic hypovolemia
b. Simple hypovolemia
c. Oligocythemic hypovolemia
d. Polycythemic hypervolemia
e. Oligocythemic hypervolemia
168. In a dysentery patient undergoing treatment in the contagious isolation ward, a significant
increase in packed cell volume (hematocrite) has been observed (60%). What other value will
be affected by this change?
a. *Increasing blood viscosity
b. Increasing volume of blood circulation
c. Leukopenia
d. Thrombocytopenia
e. Increasing erythrocyte sedimentation rate (ESR)
169. A patient is diagnosed with hereditary coagulopathy that is characterised by factor VIII
deficiency. Specify the phase of blood clotting during which coagulation will be disrupted in
the given case:
a. *Thromboplastin formation
b. Thrombin formation
c. Fibrin formation
d. Clot retraction
170. 10 minutes after the beginning of heavy physical work a person demonstrates increase of
erythrocyte number in blood from 4, 0 · 10*12/L to 4, 5 · 10*12/L. What is the cause of this
phenomenon?
a. *Erythrocytes exit from depot
b. Suppression of erythrocyte destruction
c. Erythropoiesis activation
d. Increase of cardiac output
e. Water loss
171. A 60-year-old man suffering from chronic hepatitis frequently observes nasal and gingival
hemorrhages, spontaneous hemorrhagic rashes on the skin and mucosa. Such presentations
result from:
a. *Decreased synthesis of prothrombin and fibrinogen
b. Increased blood content of aminotransferases
c. Decreased synthesis of serum albumins
d. Increased blood content of macroglobulinsand cryoglobulins
e. Decreased blood content of cholinesterase
172. After a prolonged fasting therapy, the patient presents with decreased ratio of albumins and
globulins in blood plasma. What will be the result of this changed ratio?
a. *Decreased ESR
b. Increased ESR
c. Increased hematocrit
d. Hypercoagulability
e. Decreased hematocrit
173. Degenerative changes in posterior and lateral columns of spinal cord (funicular myelosis)
caused by methylmalonic acid accumulation occur in patients with B12 -deficiency anemia.
This results in synthesis disruption of the following substance:
a. *Myelin
b. Acetylcholine
c. Norepinephrine
d. Dopamine
e. Serotonin
174. A3 year old child with fever was given aspirin. It resulted in intensified erythrocyte haemolysis.
Hemolytic anemia might have been caused by congenital insufficiency of the following
enzyme:
a. *Glucose 6-phosphate dehydrogenase
b. Glucose 6-phosphatase
c. Glycogen phosphorylase
d. Glycerol phosphate dehydrogenase
e. y-glutamiltransferase
175. Examination of a 43 y.o. anephric patient revealed anemia symptoms. What is the cause of
these symptoms?
a. *Reduced synthesis of erythropoietins
b. Enhanced destruction of erythrocytes
c. Iron deficiency
d. Vitamin B12 deficiency
e. Folic acid deficiency
176. A patient with hypochromic anemia has splitting hair and loss of hair, increased nail brittling
and taste alteration. What is the mechanism of the development of these symptoms?
a. *Deficiency of iron-containing enzymes
b. Deficiency of vitamin B12
c. Decreased production of parathyrin
d. Deficiency of vitamin A
e. Decreased production of thyroid hormones
177. A patient with hypochromic anemia has hair with splitted ends and suffers from hair loss. The
nails are brittle. Gustatory sensations are affected. What is the mechanism of development of
these symptoms?
a. *Iron enzymes deficiency
b. Low production of thyroid hormones
c. Low production of parathyroid hormone
d. Vitamin A deficiency
e. Vitamin B12 deficiency
178. A year after subtotal stomach resection on account of ulcer of lesser curvature the following
blood changes were revealed: anemia, leukocytopenia and thrombocytopenia, color index -
1,3, megaloblasts and megalocytes. What factor deficiency caused the development of thos
pathology?
a. *Castle’s factor
b. Hydrochloride acid
c. Mucin
d. Pepsin
e. Gastrin
179. As a result of an accident (snakebite) a male patient has the following blood values: Hb- 80 g/l,
RBC- 3, 0·1012/l; WBC- 5, 5·109/l.What type of anemia is observed in this case?
a. *Hemolytic
b. Aplastic
c. Iron-deficiency
d. Folic acid-deficiency
e. Posthemorrhagic
180. A 32-year-old welder complains of weakness and fever. His illness initially presented as
tonsillitis one month earlier. On examination: BT- 38, 9°C, RR- 24/min., HR- 100/min., BP-
100/70 mm Hg, hemorrhages on the legs, enlargement of the lymph nodes. CBC shows: Hb-
70 g/L, RBC- 2, 2 × 10*12/L, WBC- 3, 0 × 10*9/L with 32% of blasts, 1% of eosinophiles, 3%
of bands, 36% of segments, 20% of lymphocytes, and 8% of monocytes, ESR- 47 mm/hour.
What is the cause of anemia?
a. *Acute leukemia
b. Aplastic anema
c. Chronic hemolytic anemia
d. B12-deficient anemia
e. Chronic lympholeukemia
181. A 54-year-old man complains of general weakness, frequent colds, bruises constantly
appearing on his body. Blood test: erythrocytes – 2,5 × 10*12/L, Hb – 80 g/L, color index –
0,9; reticulocytes – absent, platelets - 50 × 10*9/L, leukocytes - 58 × 10*9/L. Leukogram:
basocytes – 5%, eosinophils – 15%, myeloblasts – 6%, myelocytes – 10%, juvenile – 18%,
stab neutrophils – 26%, segmented neutrophils – 10%, lymphocytes – 8%, monocytes – 2%.
ESR- 40mm/hour. What hematologic conclusion can be made?
a. Myeloblastic leukemia
b. *Chronic myelogenous leukemia
c. Chronic lymphocytic leukemia
d. Lukemoid response
e. Basophilic eosinophilic leukocytosis
182. Due to trauma the patient has lost 25% of circulating blood volume. Name the emergency
compensatory mechanism against blood loss:
a. Restoration of blood protein composition
b. Erythropoiesis activation
c. *Interstitial fluid inflow to the vessels
d. Restoration of erythrocyte number
e. Increase of reticulocyte number
183. A 68-year-old man comes to his physician with complaints of severe fatigue and altered
sensations in his extremities. Past medical history is remarkable for chronic gastritis. He
drinks alcohol almost every day. BP -130/80 mmHg, heart rate 95/min, respiratory rate -
14/min, body T – 37,1C. Neurologic examination reveals loss of touch and vibration sense in
both upper and lower limbs. Laboratory data: hemoglobin 80g/L, Mean Corpuscular Volume –
115 fL (reference range – 80-100 fL), WBC – 3,0810#9/L. Which of the following is the most
likely diagnosis&
a. *Vitamin B12 deficiency
b. Vitamin A deficiency
c. Vitamin C deficiency
d. Iron deficiency
184. A patient has been suffering from bronchial asthma for 15 years. What changes in the
patient’s leukogram can be expected?
a. *Eosinophilia
b. Leukopenia
c. Leukocytosis
d. Left shift
e. Basophilia
185. Hematologic pattern shows the following pattern:: erythrocytes – 2,8 × 10*12/L, Hb – 80 g/L,
color index – 0,85; reticulocytes – 0,1%, platelets - 160 thousand per microliter, leukocytes -
60 × 10*9/L. Basocytes – 2%, eosinophils – 5%, promyelocytes – 5%, myelocytes – 5%,
juvenile – 16%, stab neutrophils – 20%, segmented neutrophils – 34%, lymphocytes – 5%,
monocytes –5%. This clinical presentation indicates the following blood pathology:
a. *Chronic myeloleukemia
b. Undifferentiated leukemia
c. Acute myeloleukemia
d. Hemolytic anemia
e. Hypoplastic anemia
186. A 34-year-old man visits his dentist complaining of toothache. After a dental procedure that
involved extraction of several teeth, he developed severe bleeding lasting more than 15
minutes. He has a history of chronic hepatitis C. Which of the following is the most likely
cause of prolonged bleeding in this patient?
a. *Hypofibrinogenemia
b. Hypoalbuminemia
c. Hypocalcemia
d. Thrombocytopenia
187. ECG of a 44-year-old patient shows signs of hypertrophy of both ventricles and the right
atrium. The patient was diagnosed with the tricuspid valve insufficiency. What pathogenetic
variant of cardiac dysfunction is usually observed in case of such insufficiency?
a. *Heart overload by volume
b. Heart overload by resistance
c. Primary myocardial insufficiency
d. Coronary insufficiency
e. Cardiac tamponade
188. Coronary artery thrombosis resulted in development of myocardial infarction. What
mechanisms of cell damage are leading in this disease?
a. *Calcium
b. Lipid
c. Acidotic
d. Protein
e. Electroosmotic
189. A 45-year-old patient was admitted to the cardiological department. ECG data: negative P
wave overlaps QRS complex, diastolic interval is prolonged after extrasystole. What type of
extrasystole is it?
a. *Atrioventricular
b. Sinus
c. Atrial
d. Ventricular
e. Bundle-branch
190. A 17-year-old girl suffers from periodical palpitations that last several minutes. Her heart rate
is 200/min, rhythmic. What heart rhythm disorder developed in this patient?
a. *Paroxysmal tachycardia
b. Sinus bradycardia
c. Sinus tachycardia
d. Atrioventricular block
e. Extrasystole
191. A patient has been diagnosed with influenza. His condition became drastically worse after
taking antipyretic drugs. His consciousness is confused, AP is 80/50mm Hg, Ps is 140/m,
body temperature dropped down to 35, 8oC. What complication developed in this patient?
a. *Collapse
b. Hyperthermia
c. Hypovolemia
d. Acidosis
e. Alkalosis
192. A month after surgical constriction of rabbit’s renal artery the considerable increase of
systematic arterial pressure was observed. What of the following regulation mechanisms
caused the animal’s pressure change?
a. *Angiotensin-II
b. Vasopressin
c. Adrenaline
d. Noradrenaline
e. Serotonin
193. After a serious psychoemotional stress a 48 year old patient suddenly developed acute heart
ache irradiating to the left arm. Nitroglycerine relieved pain after 10 minutes. What is the
leading pathogenetic mechanism of this process development?
a. *Spasm of coronary arteries
b. Dilatation of peripheral vessels
c. Obstruction of coronary vessels
d. Compression of coronary vessels
e. Increase in myocardial oxygen consumption
194. The patient with acute myocardial infarction was given intravenously different solutions during
8 hours with medical dropper 1500 ml and oxygen intranasally. He died because of
pulmonary edema. What caused the pulmonary edema?
a. *Volume overload of the left ventricule
b. Decreased oncotic pressure due to hemodilution
c. Allergic reaction
d. Neurogenic reaction
e. Inhalation of the oxygen
195. A 67 year old patient complains of periodic heart ache, dyspnea during light physical activities.
ECG reveals extraordinary contractions of heart ventricles. Such arrhythmia is called:
a. *Extrasystole
b. Bradycardia
c. Tachycardia
d. Flutter
e. Fibrillation
196. Since a patient has had myocardial infarction, atria and ventricles contract independently from
each other with a frequency of 60-70 and 35-40 per minute respectively. Specify the type of
heart block in this case:
a. *Complete atrioventricular
b. Partial atrioventricular
c. Sino-atrial
d. Intra-atrial
e. Intraventricular
197. A patient in the infectious diseases unit presents with elevated temperature up to 39 C. ECG
shows shortened R-R interval, P wave precedes each QRS complex. Heart rate is 120 bpm.
What characteristic of the cardiac muscle is disturbed, causing the development of this
pathological rhythm?
a. *Automatism
b. Conductivity
c. Rhythm assimilation
d. Automatism and conductivity
e. Conductivity and rhythm assimilation
198. A patient with constant headaches, pain in the occipital region, tinnitus, dizziness has been
admitted to the cardiology department. Objectively: AP- 180/110 mm Hg, heart rate - 95/min.
Radiographically, there is a stenosis of one of the renal arteries. Hypertensive condition in
this patient has been caused by the activation of the following system:
a. *Renin-angiotensin
b. Hemostatic
c. Sympathoadrenal
d. Kinin
e. Immune
199. A patient with a pathology of the cardiovascular system developed edema of the lower
extremities. What is the mechanism of cardiac edema development?
a. *Increased hydrostatic pressure at the venous end of the capillary
b. Increased oncotic pressure
c. Increased hydrostatic pressure at the arterial end of the capillary
d. Reduced osmotic pressure
e. Lymph efflux disorder
200. A 43-year-old-patient has arterial hypertension caused by an increase in cardiac output and
general peripheral resistance. Specify the variant of hemodynamic development of arterial
hypertension in the given case:
a. *Eukinetic
b. Hyperkinetic
c. Hypokinetic
d. Combined
201. A patient has insufficient blood supply to the kidneys, which has caused the development of
pressor effect due to the constriction of arterial resistance vessels. This is the result of the
vessels being greately affected by the following substance:
a. *Angiotensin II
b. Angiotensinogen
c. Renin
d. Catecholamines
e. Norepinephrine
202. When studying the signs of pulmonary ventilation, reduction of forced expiratory volume has
been detected. What is the likely cause of this phenomenon?
a. *Obstructive pulmonary disease
b. Increase of respiratory volume
c. Increase of inspiratory reserve volume
d. Increase of pulmonary residual volume
e. Increase of functional residual lung capacity
203. A patient complains of palpitation after stress. The pulse is 104 bpm, P-Q=0,12 seconds,
there are no changes of QRS complex. What type of arrhythmia does the patient have?
a. *Sinus tachycardia
b. Sinus bradycardia
c. Sinus arrhythmia
d. Ciliary arrhythmia
e. Extrasystole
204. A 67-year-old patient complains of periodic heartache, dyspnea during light physical activities.
ECG reveals extraordinary contractions of heart ventricles. Such arrhythmia is called:
a. *Extrasystole
b. Bradycardia
c. Tachycardia
d. Flutter
e. Fibrillation
205. An athlete (long-distance runner) during a contest developed a case of acute cardiac
insufficiency. This pathology resulted from:
a. *Cardiac volume overload
b. Disrupted coronary circulation
c. Direct damage to myocardium
d. Pericardium pathology
e. Cardiac pressure overload
206. A 67-year-old man was delivered to the cardiology unit with complaints of periodical pain in the
heart , dyspnea after even insignificant physical exertion, cyanosis, and edemas. ECG
revealed additional contractions of the heart ventricles. Name this type of rhythm disturbance:
a. *Extrasystole
b. Tachycardia
c. Flutter
d. Bradycardia
e. Fibrillation
207. A 16-year-old girl fainted when she tried to change quickly her position from horizontal to
vertical. What caused the loss of consciousness in the girl?
a. *Decreased venous return
b. Decreased oncotic plasma pressure
c. Increased arterial pressure
d. Increased central venous pressure
e. Increased venous return
208. A patient with marked pneumofibrosis that developed after infiltrating pulmonary tuberculosis
has been diagnosed with respiratory failure. What is its pathogenetic type?
a. *Restrictive
b. Obstructive
c. Dysregulatory
d. Reflex
e. Apneistic
209. A 30-year-old man has sustained an injury to his thorax in a traffic incident, which caused
disruption of his external respiration. What type of ventilatory difficulty can be observed in the
given case?
a. *Restrictive extrapulmonary ventilatory impairment
b. Restrictive pulmonary ventilatory impairment
c. Obstructive ventilatory impairment
d. Impaired ventilation regulation dysfunction
e. Cardiovascular collapse
210. An unconscious young man in the state of morphine intoxication has been delivered into an
admission room. The patient’s respiration is slow and shallow due to suppression of the
respiratory center. What kind of respiratory failure occurred in this case?
a. *Ventilatory disregulation
b. Ventilatory obstruction
c. Ventilatory restriction
d. Perfusion
e. Diffusion
211. A 23-year-old patient has been admitted to a hospital with a craniocerebral injury. The patient
is in a grave condition. Respiration is characterized by prolonged convulsive inspiration
followed by a short expiration. What kind of respiration is it typical for?
a. *Apneustic
b. Gasping breath
c. Kussmaul’s
d. Cheyne-Stokes
e. Biot’s
212. A patient with bronchial asthma has developed acute respiratory failure. What kind of
respiratory failure occurs in this case?
a. *Obstructive disturbance of alveolar ventilation
b. Restrictive ventilatory defect
c. Perfusion
d. Diffusion
e. Dysregulation of alveolar ventilation
213. Increased HDL levels decrease the risk of atherosclerosis. What is the mechanism of HDL
anti-atherogenic action?
a. *They remove cholesterol from tissues
b. They supply tissues with cholesterol
c. They are involved in the breakdown of cholesterol
d. They activate the conversion of cholesterol to bile acids
e. They promote absorption of cholesterol in the intestine
214. A 15-year-old teenager complains of lack of air, general weakness, palpitations. Heart rate is
130/min. BP is 100/60 mmHg. ECG: QRS complex has normal shape and duration. The
number of P-waves and ventricular complexes is equal. T wave merges with P-wave. What
type of cardiac arrhythmia is observed in the teenager?
a. *Sinus tachycardia
b. Atrial thrill
c. Paroxysmal atrial tachycardia
d. Sinus extrasystole
e. Atrial fibrillation
215. A woman who has been suffering from marked hypertension for 15 years, has lately
developed dyspnea, palpitations, slightly decreased systolic pressure, while diastolic remains
the same. What is the main mechanism of heart failure development in this case?
a. *Cardiac overload due to increased vascular resistance
b. Disorder of impulse conduction in the myocardium
c. Damage to the myocardium
d. Cardiac overload due to increased blood volume
e. Dysregulation of cardiac function
216. During ascent into mountains a person develops increased respiration rate and rapid heart
rate. What is the cause of these changes?
a. *Decrease of O2 partial pressure
b. Increase of CO2 partial pressure
c. Increase of blood pH
d. Increase of nitrogen content in air
e. Increase of air humidity
217. A 48-year-old man is unconscious. He has a history of several syncopal episodes with
convulsions. ECG shows deformed QRS complexes unconnected with P-waves, atrial
contractions are approximately 70/min, ventricular contractions -25-30/min. Name the type of
arrhythmia in this case:
a. *Complete atrioventricular block
b. Intraatrial block
c. Second-degree atrioventricular block
d. First degree atrioventricular block
e. Intraventricular block
218. ECG of a the patient shows increased duration of the QRS complex. What is the most likely
cause?
a. *Increased period of ventricular excitation
b. Increased atrial and ventricular excitability
c. Conduction disturbances in the AV node
d. Increased atrial excitability
e. Increased period of atrial excitation
219. A patient is 59 years old and works as director of a private enterprise. After the inspection by
tax authorities he developed intense burning retrosternal pain radiating to the left arm. In 15
minutes the patient’s condition had normalized. What is the leading mechanism of angina
pectoris development in this patient?
a. *Increased level of blood catecholamines
b. Coronary artery atherosclerosis
c. Intravascular aggregation of blood cells
d. Coronary artery thrombosis
e. Functional cardiac overload
220. At examination, the 16 year-old patient, an increase in heart rate was detected during
inhalation then it slowed down during exhalation. The ECG signs: shortening of the RR
interval during inspiration and extending it during expiration. What is the type of arrhythmia?
a. *Sinus arrhythmia
b. Atrial fibrillation
c. Sinus tachycardia
d. Idioventricular rhythm
e. Sinus bradycardia
221. One hour after the putting of the surgical clamp narrowing the aorta, the strength and
frequency of heart rate significantly increased in the dog, and the volume of circulating blood
and the thickness of the left ventricle wall did not differ from the baseline. What stage of
myocardial hypertrophy is observed in an animal?
a. *Emergency
b. Decompensation
c. Progressive cardiosclerosis
d. Relatively stable hyperfunction
e. Completed hypertrophy
222. While modeling arterial hypertension in a dog after 1 month, the thickness of the wall of the left
ventricle increased by 1.7 times, and the volume of circulating blood did not change
compared with the initial data. What stage of myocardial hypertrophy is observed in an
animal?
a. *Complete hypertrophy
b. Emergency
c. Initial
d. Progressive cardiosclerosis
e. Decompensation
223. The patient for 17 years suffers from chronic glomerulonephritis for 17 years. Pulse 82 per
minute. BP 190/120 mm Hg. What is the primary mechanism of blood pressure increase in
the patient?
a. *Increased total peripheral resistance
b. Increased volume of circulating blood
c. Increased venous tone
d. Increased shock volume
e. Increase in minute volume of blood
224. A 49-year-old man has mitral stenosis. What is the leading mechanism of heart failure in this
case?
a. *Pressure overload
b. Volume overload
c. Myocardial tension
d. Myocardial injury
e. Fluid overload
225. A woman of 25 years complains of permanent pain in the heart, shortness of breath when
moving, general weakness. Objectively: skin is pale and cold, acrocyanosis. Pulse 96 in 1
minute, BP - 105/70 mm Hg The heartline is shifted 2 cm to the left. The first tone above the
top of the heart is weakened, systolic noise is auscultated. Diagnosis of mitral valve
insuficiency is diagnosed. What is the reason of circulatory disorders?
a. *Overload of myocardium with increased blood volume
b. Overload of myocardium with increased resistance to blood outflow
c. Myocardial damage
d. Reduced volume of circulating blood
e. Increased volume of the vascular bed
226. One of the most dangerous moments in the pathogenesis of myocardial necrosis is the further
increase in areas of necrosis, dystrophy and ischemia. An important role in this is to increase
the consumption of oxygen by the myocardium. What substances contribute to this process?
a. *Catecholamines
b. Acetylcholine
c. Adenosine
d. Cholesterol
e. Ions of chlorine
227. The patient 59 years old was hospitalized in a cardiac department in a difficult condition with a
diagnosis: an acute myocardial infarction in the region of the posterior wall of the left ventricle
and septum, an initial pulmonary edema. What is the primary mechanism that causes the
patient to develop pulmonary edema?
a. *Left ventricular insufficiency
b. Pulmonary venous hypertension
c. Pulmonary arterial hypertension
d. Hypoxemia
e. Reduction of alveolar-capillary oxygen diffusion
228. A patient with mechanical jaundice entered the hospital with signs of a cholemic syndrome. At
ECG, arrhythmia has been detected. Which heart rhythm disturbance is most likely in a
patient?
a. *Sinus bradycardia.
b. Sinus tachycardia.
c. Forehead extrasystoles.
d. Ventricular extrasystoles.
e. Atrioventricular blockade.
229. The patient has an increase in the resistance of the outflow of blood from the left ventricle,
which led to the inclusion of the homeometric compensation mechanism. Which of the listed
pathological processes may activate this compensation mechanism in the left ventricle of the
heart
a. *Aortic valve stenosis
b. Aortic valve deficiency
c. Mitral stenosis
d. Arterial hypotension
e. Pulmonary artery embolism
230. A man has 65 years of age had arterial hypertension for 15 years. Recently, systolic pressure
has started to decrease, and the diastolic remains elevated. What is the hemodynamic type of
hypertension in the patient?
a. *Hypokinetic.
b. Normokinetic.
c. Hyperkinetic.
d. Eukinetic.
e. -
231. In a patient with ischemic heart disease caused by atherosclerosis of the coronary arteries
thrombosis of the anterior ventricular coronary artery has developed. What mechanism in the
development of this complication is the most significant?
a. *Damage to the endothelium of the vascular wall
b. Slow speed of blood flow
c. Increased concentration of blood coagulants
d. Reducing blood anticoagulants
e. Reduced activity of the fibrinolytic system
232. A 45-year-old patient complains of shortness of breath with a small physical load, swelling on
the legs, in the anamnesis of frequent tonsillitis, is ill for two years. Diagnosed with
insufficiency of blood circulation. What hemodynamic index of cardiac decompensation is
observed in this case?
a. *Reduce of the minute volume of the heart.
b. Reduce of the volume of circulating blood.
c. Decreased venous pressure.
d. Increased blood pressure.
e. Tachycardia.
233. In a patient with ischemic heart disease suddenly there was a severe attack of angina
pectoris. About him: face pale, wet skin, cold, blood pressure 70/50 mm Hg. Art. extrasystole
Diagnosed myocardial infarction and cardiogenic shock. What is the primary mechanism of
pathogenesis?
a. *Decrease of the minute volume of blood.
b. Extrasystoles.
c. Toxemia
d. Pain syndrome.
e. Hypotension.
234. A man has 25 years of insufficiency of the mitral valve without disturbance of blood circulation.
What immediate mechanism provides for cardiac compensation?
a. *Heterometric
b. Homeometric
c. Myogenic dilation
d. Decreased heart rate.
e. Strengthening the synthesis of catecholamines
235. A 14-year-old adolescent has diphtheria. During the peak of the disease against the
background of acute drop in body temperature and tachycardia ht blood pressure is 70/50
mmHg. What type of vascular tone disturbance is it?
a. *Acute hypotension
b. Somatoform autonomic dysfunction
c. Essential hypotension
d. Chronic hypotension
236. A 4-year-old person developed elevated BP after and emotional excitement. What is the likely
cause of this effect?
a. *Increased sympathetic nervous system tone
b. Increased parasympathetic nervous system tone
c. Arteriolar dilation
d. Decreased cardiac contraction frequency
e. Hyperpolarization of cardiomyocytes
237. In stress conditions an eldery person developed elevated blood pressure. It is caused by
activation of:
a. *Sympathoadrenal system
b. Adrenocortical functions
c. Thyroid functions
d. Parasympathetic nucleus of the vagus nerve
e. Pituitary functions
238. After hyperventilation an athlete developed a brief respiratory arrest. It occurred due to the
following changes in the blood:
f. *Decrease of CO2 pressure
g. Increase of CO2 pressure
h. Decrease of O2 pressure
i. Decrease of pH pressure
j. Increase of CO2 and O2 pressure
239. A patient with asphyxia after a short respiratory arrest developed single infrequent respirations
with passive expiration, after which he stopped breathing completely. What type of respiration
was observed in this case?
a. *Gasping respiration
b. Cheyne-Stokes respiration
c. Kussmaul respiration
d. Apneustic respiration
e. Biot respiration
240. A 46-year-old patient suffering from the diffuse toxic goiter underwent resection of the thyroid
gland. After the surgery the patient presents with appetite loss, dyspepsia, increased
neuromuscular excitement. The body weight remained unchanged. Body temperature is
normal. Which of the following has caused such a condition in this patient?
a. *Reduced production of parathormone
b. Increased production of thyroxin
c. Increased production of calcitonin
d. Increased production of thyroliberin
e. Reduced production of thyroxin
241. A coprological survey revealed light colored feces containing drops of neutral fat. The most
likely reason for this condition is the disorder of:
a. *Bile inflow into the bowel
b. Gastric juice acidity
c. Pancreatic juice secretion
d. Intestinal juice secretion
e. Intestinal absorption
242. The secretion of which hypophysial hormones will be inhibited after taking the oral
contraceptives containing sex hormones?
a. *Gonadotropic hormone
b. Vasopressin
c. Thyrotrophic hormone
d. Somatotropic hormone
e. Oxytocin
243. As a result of continuous starvation the glomerular filtration rate has increased by 20%. The
most probable cause of the glomerular filtration alteratio nunder the mentioned conditions is:
a. *Decrease in the oncotic pressure of-blood plasma
b. Increase in the systemic arterial pressure
c. Increase in the permeability of the renal filter
d. Increase of the filtartion quotient
e. Increase of the renal blood flow
244. Blood analysis of a patient with jaundice reveals conjugated bilirubinemia, increased
concentration of bile acids.There is no stercobilinogen in urine.What type of jaundice is it?
a. *Obstructive jaundice
b. Hepatocellular jaundice
c. Parenchymatous jaundice
d. Hemolytic jaundice
e. Cythemolytic jaundice
245. According to the results of glucose tolerance test a patient has no disorder of carbohydrate
tolerance. Despite that glucose is detected in the patients’s urine (5 mmol/l). The patient has
been diagnosed with renal diabetes. What renal changes cause glucosuria in this case?
a. *Decreased activity of glucose reabsorption enzymes
b. Increased activity of glucose reabsorption enzymes
c. Exceeded glucose reabsorption threshold
d. Increased glucose secretion
e. Increased glucose filtration
246. A concentrated solution of sodium chloride was intravenously injected to ananimal. This
caused decreased reabsorption of sodium ions in the renal tubules. It is the result of the
following changes of hormonal secretion:
a. *Aldosterone reduction
b. Aldosterone increase
c. Vasopressin reduction
d. Vasopressin increase
e. Reduction of atrial natriuretic factor
247. Diabetic nephropathy with uremia has developed in a patient with pancreatic diabetes. The
velocity of glomerular filtration is 9 ml/min. What mechanism of a decrease in glomerular
filtration velocity and chronic renal failure development is most likely in the case of this
patient?
a. *Reduction of active nephron mass
b. Decrease in systemic arterial pressure
c. Obstruction of nephron tubules with hyaline casts
d. Tissue acidosis
e. Arteriolar spasm
248. A 49-year old female patient has limitation of left limbs arbitrary movements. Muscular tonus of
left hand and leg is overstrained and spasmodic, local tendon reflexes are strong,
pathological reflexes are presented. What is the most likely development mechanism of
hypertension and hyperreflexia?
a. *Reduction of descending inhibitory influence
b. Motoneuron activation induced bystroke
c. Activation of excitatory influence fromthe focus of stroke
d. Activation of synaptic transmission
e. Ihibition of cerebral cortex
249. The patient with complaints about permanent thirst applied to the doctor. Hyperglycemia,
polyuria and increased concentration of 17-ketosteroids in the urine were revealed. What
disease is the most likely?
a. *Steroid diabetes
b. Insulin-dependent diabetes mellitus
c. Myxedema
d. Type I glycogenosis
e. Addison’s disease
250. A 32-year-old patient consulted a doctor about the absence of lactation after parturition. Such
disorder might be explained by the deficit of the following hormone:
a. *Prolactin
b. Somatotropin
c. Vasopressin
d. Thyrocalcitonin
e. Glucagon
251. A 41-year-old man has a history of recurrent attacks of heartbeats (paroxysms), profuse
sweating, headaches. Examination revealed hypertension, hyperglycemia, increased basal
metabolic rate, and tachycardia. These clinical presentations are typical for the following
adrenal pathology:
a. *Hyperfunction of the medulla
b. Hypofunction of the medulla
c. Hyperfunction of the adrenal cortex
d. Hypofunction of the adrenal cortex
e. Primary aldosteronism
252. A patient complains of hydruria (7 liters per day) and polydipsia. Examination reveals no
disorders of carbohydrate metabolism. These abnormalities might be caused by the
dysfunction of the following endocrine gland:
a. *Neurohypophysis
b. Adenohypophysis
c. Islets of Langerhans (pancreatic islets)
d. Adrenal cortex
e. Adrenal medulla
253. A patient presents with icteritiousness of skin, scleras and mucous membranes. Blood plasma
the total bilirubin is increased, stercobilin is increased in feces, urobilin is increased in
urine.What type of jaundice is it?
a. *Haemolytic
b. Gilbert’s disease
c. Parenchymatous
d. Obturational
e. Cholestatic
254. A patient with massive burns developed acute renal insufficiency characterized by a significant
and rapid deceleration of glomerular filtration. What is the mechanism of its development?
a. *Reduction of renal blood flow
b. Damage of glomerular filter
c. Reduction of functioning nephron number
d. Rise of pressure of tubular fluid
e. Renal artery embolism
255. A 15-year-old boy has been diagnosed with acute viral hepatitis. What blood value should be
determined to confirm acute affection of hepatic cells?
a. *Aminotransferase activity (AST, ALT)
b. Erythrocytes sedimentation rate (ESR)
c. Protein fraction content
d. Cholesterol content
e. Unconjugated and conjugated bilirubin content
256. Encephalopathy has developed in a child with hemolytic disease of the newborn. What
substance had increased in the child’s blood resulting in damage to CNS?
a. *Unconjugated bilirubin
b. Bilirubin glucuronide
c. Verdohemoglobin
d. Bilirubin-albumin complex
e. Bile acids
257. A newborn develops dyspepsia after the milk feeding. When the milk is substituted by the
glucose solution the dyspepsia symptoms disappear. The newborn has the subnormal activity
of the following enzyme:
a. *Lactase
b. Invertase
c. Maltase
d. Amylase
e. Isomaltase
258. A child has abnormal formation of tooth enamel and dentin as a result of low concentration of
calcium ions in blood. Such abnormalities might be caused by deficiency of the following
hormone:
a. *Parathormone
b. Thyrocalcitonin
c. Thyroxin
d. Somatotropic hormone
e. Triiodothyronine
259. A man has a considerable decrease in diuresis as a result of 1,5 l blood loss. The primary
cause of such diuresis disorder is the hypersecretion of the following hormone:
a. *Vasopressin
b. Corticotropin
c. Natriuretic
d. Cortisol
e. Parathormone
260. A patient with a history of chronic glomerulonephritis presents with azotemia, oliguria, hypo-
and isosthenuria, proteinuria. What is the leading factor in the pathogenesis of these
symptoms development under chronic renal failure?
a. *Mass decrease of active nephrons
b. Intensification of glomerular filtration
c. Tubular hyposecretion
d. Disturbed permeability of glomerular membranes
e. Intensification of sodium reabsorption
261. A 20 year old patient complains of morbid thirst and huperdiuresis (up to 10 l daily). Glucose
concentration in blood is normal but it is absent in urine. The patient has been diagnosed with
diabetes insipidus. What hormonal drug is the most appropriate for management of this
disorder?
a. *Vasopressin
b. Cortisol
c. Thyroxin
d. Oxytocin
e. Insulin
262. A 42 year old patient complains of pain in the epigastral area, vomiting; vomit masses have
the colour of "coffee-grounds", the patient has also melena. Anamnesis records gastric ulcer.
Blood formula: erythrocytes - 2, 8 · 10#12/L, leukocytes - 8 · 10#9/L, Hb- 90 g/L. What
complication is it?
a. *Haemorrhage
b. Penetration
c. Perforation
d. Canceration
e. Pyloric stenosis
263. Blood analysis of a patient with jaundice reveals conjugated bilirubinemia, increased
concentration of bile acids. There is no stercobilinogen in urine. What type of jaundice is it?
a. *Obstructive jaundice
b. Hepatocellular jaundice
c. Parenchymatous jaundice
d. Hemolytic jaundice
e. Cythemolytic jaundice
264. A 5-month-old boy was hospitalized for tonic convulsions. He has a lifetime history of this
disease. Examination revealed coarse hair, thinned and fragile nails, pale and dry skin. In
blood: calcium - 1,5millimole/L, phosphor - 1,9 millimol/L. These changes are associated with:
a. *Hypoparathyroidism
b. Hyperparathyroidism
c. Hyperaldosteronism
d. Hypoaldosteronism
e. Hypothyroidism
265. A patient with adenoma in the zona glomerulosa of the adrenal gland (Conn’s syndrome)
presents with arterial hypertension, convulsive attacks and polyuria. Name the main link in the
pathogenesis of these disorders:
a. *Hyperaldosteronism
b. Hypersecretion of catecholamines
c. Hypersecretion of glucocorticoids
d. Hyposecretion of glucocorticoids
e. Hypoaldosteronism
266. Examination of a 42 year old patient revealed a tumour of adenohypophysis. Objectively: the
patient’s weight is 117 kg, he has moon-like hyperemic face, red-blue striae of skin distension
on his belly. Osteoporosis and muscle dystrophy are present. AP is 210/140 mm Hg. What is
the most probable diagnosis?
a. *Cushing’s disease
b. Cushing’s syndrome
c. Conn’s disease
d. Diabetes mellitus
e. Essential hypertension
267. A 38-year-old female patient complains of general weakness, cardiac pain, increased appetite,
no menstruation. Objectively: the height is 166 cm, weight 108 kg, the patient has moon-
shaped face, subcutaneous fat is deposited mainly in the upper body, torso and hips. There
are also blood-red streaks. Ps- 62/min, AP- 160/105 mm Hg. Which of the following diseases
is the described pattern of obesity most typical for?
a. *Cushing pituitary basophilism
b. Alimentary obesity
c. Myxedema
d. Insulinoma
e. Babinski-Frohlich syndrome
268. A female patient with bronchial asthma had taken prednisolone tablets (1 tablet 3 times a day)
for 2 months. Due to a significant improvement of her conditionthe patient suddenly stopped
taking it. What complication is likely to develop in this case?
a. *Withdrawal syndrome
b. Cushing’s syndrome
c. Gastrorrhagia
d. Upper body obesity
e. Hypotension
269. A patient with jaundice has high total bilirubin that is mainly indirect (unconjugated), high
concentration of stercobilin in the stool and urine. The level of direct (conjugated) bilirubin in
the blood plasma is normal. What kind of jaundice can you think of?
a. *Hemolytic
b. Parenchymal (hepatic)
c. Mechanical
d. Neonatal jaundice
e. Gilbert’s disease
270. A patient simultaneously presents with high levels of conjugated (direct) and unconjugated
(indirect) bilirubin in the blood plasma, while stercobilinogen levels in feces and urine are
sharply decreased. What type of jaundice does the patient have?
a. *Parenchymatous jaundice
b. Hemolytic jaundice
c. Obstructive jaundice
d. Jaundice of the newborn
e. Gilbert’s syndrome
271. 14 days after quinsy a 15-year-old child presented with morning facial swelling, high blood
pressure, "meat slops"urine. Immunohistological study of a renal biopsy sample revealed
deposition of immune complexes on the basement membranes of the capillaries and in the
glomerular mesangium. What disease developed in the patient?
a. *Acute glomerulonephritis
b. Acute interstitial nephritis
c. Lipoid nephrosis
d. Acute pyelonephritis
e. Necrotizing nephrosis
272. Due to the use of poor-quality measles vaccine for preventive vaccination, a 1-year-old child
developed an autoimmune renal injury. The urine was found to contain macromolecular
proteins. What process of urine formation was disturbed?
a. *Filtration
b. Reabsorption
c. Secretion
d. Reabsorption and secretion
e. Secretion and filtration
273. A patient with signs of osteoporosis and urolithiasis has been admitted to the endocrinology
department. Blood test revealed hypercalcemia and hypophosphatemia. These changes are
associated with abnormal synthesis of the following hormone:
a. *Parathyroid hormone
b. Calcitonin
c. Cortisol
d. Aldosterone
e. Calcitriol
274. A patient has been admitted to the contagious isolation ward with signs of jaundice caused by
hepatitis virus. Which of the symptoms given below is strictly specific for hepatocellular
jaundice?
a. *Increase of ALT, AST level
b. Hyperbilirubinemia
c. Bilirubinuria
d. Cholemia
e. Urobilinuria
275. An unconscious patient was delivered by ambulance to the hospital. On objective examination
the patient was found to present no reflexes, periodical convulsions, irregular breathing. After
laboratory examination the patient was diagnosed with hepatic coma. Disorders of the central
nervous system develop due to accumulation of the following metabolite:
a. *Ammonia
b. Urea
c. Glutamine
d. Bilirubin
e. Histamine
276. A 50-year-old man, who has been suffering from chronic hepatic failure for several years, has
developed ascites. What is the main mechanism of this disorder development?
a. *Increased pressure in portal vein system
b. Decrease of albumin and globulin synthesis in liver
c. Increased content of low-density and very low-density lipoproteins in blood
d. Neurotoxins appearing in blood
e. Increase of blood oncotic pressure
277. A 43-year-old patient suffers from acute pancreatitis with disrupted common bile duct patency.
What condition can develop in this case?
a. *Mechanical jaundice
b. Hemolytic jaundice
c. Hepatocellular jaundice
d. Hepatic coma
e. Portal hypertension
278. Administration of doxycycline hydrochloride has caused an imbalance of the symbiotic
intestinal microflora. Specify the kind of imbalance caused by the antibiotic therapy:
a. *Dysbacteriosis
b. Sensibilization
c. Idiosyncrasy
d. Superimposed infection
e. Bacteriosis
279. Cholesterol content in blood serum of a 12-year-old boy is 25 mmol/L. Anamnesis states
hereditary familial hypercholesterolemia caused by synthesis disruption of receptor-related
proteins for:
a. *Low-density lipoproteins
b. High-density lipoproteins
c. Chylomicrons
d. Very low-density lipoproteins
e. Middle-density lipoproteins
280. A patient presents with steatorrhea. This disorder can be linked to disturbed supply of the
intestine with the following substances:
a. *Bile acids
b. Amylase
c. Carbohydrates
d. Tripsin
e. Chymotrypsyn
281. A patient with jaundice has high total bilirubin that is mainly indirect (unconjugated), high
concentration of stercobilin in the feces and urine. The level of direct (conjugated) bilirubin in
the blood plasma is normal. What kind of jaundice can be suspected?
a. *Hemolytic
b. Parenchymal (hepatic)
c. Mechanical
d. Neonatal jaundice
e. Gilbert’s disease
282. Poisoning caused by mercury (II) chloride (corrosive sublimate) occurred in the result of
safety rules violation. In 2 days the patient’s diurnal diuresis became 620 ml. The patient
developed headache, vomiting, convulsions, dyspnea; moist crackles are observed in the
lungs. Name this pathology:
a. *Acute renal failure
b. Chronic renal failure
c. Uremic coma
d. Glomerulonephritis
e. Pyelonephritis
283. A lab rat has subcutaneously received mercury (II) chloride in the amount of 5 mg/kg. 24
hours later the plasma creatinine concentration increased several times. What mechanism of
retention azotemia is observed in this case?
a. *Decreased glomerular filtration
b. Increased creatinine production in the renal tubules
c. Increased glomerular filtration
d. Increased creatinine production in the muscles
e. Increased creatinine reabsorption
284. A 30-year-old woman exhibits signs of virilism (growth of body hair, balding temples, disturbed
menstrual cycle). This condition can be caused by overproduction of the following hormone:
a. *Testosterone
b. Prolactin
c. Relaxin
d. Oestriol
e. Oxytocin
285. After a severe stress a patient was found to have eosinopenia. In this case the decreased
number of eosinophils can explain changes in the level of the following hormones:
a. *Glucocorticoids
b. Adrenaline
c. Insulin
d. Mineralocorticoids
e. Vasopressin
286. A 19-year-old male was examined in the nephrology clinic. High potassium level was found in
his secondary urine. What hormone is likely to cause such change if it is produced in excess?
a. *Aldosterone
b. Oxytocin
c. Adrenaline
d. Glucagon
e. Testosterone
287. After a case of sepsis a 27-year-old woman developed “bronzed” skin discoloration,
characteristic of Addison’s disease. Hyperpigmentation mechanism in this case is based on
increased secretion of:
a. *Melanocyte –stimulating hormone
b. Beta-lipotropin
c. Gonadotropin
d. Thyroid stimulating hormone
e. Somatotropin
288. During removal of the hyperplastic thyroid gland of a 47-year-old woman, the parathyroid
gland was damaged. One month after the surgery the patient developed signs of
hypoparathyroidism: frequent convulsions, hyperreflexia, laryngospasm. What is the most
likely cause of the patient’s condition?
a. *Hypocalcemia
b. Hyponatremia
c. Hyperchlorhydria
d. Hypophosphatemia
e. Hyperkalemia
289. A 46-year old woman suffering from cholelithiasis developed jaundice. Her urine became dark
yellow, while feces are light colored. What substance will be the most increased in
concentration in the blood serum in this case?
a. *Conjugated bilirubin
b. Biliverdine
c. Urobilinogen
d. Mesobilirubin
e. Unconjugated bilirubin
290. A 40-year-old woman with Cushing’s disease presents with steroid diabetes. On biochemical
examination she has hyperglycemia and hypochloremia. What process activates in the first
place to such patient?
a. *Gluconeogenesis
b. Glycogenolysis
c. Glucose reabsorption
d. Glucose transport to the cell
e. Glycolysis
291. On examination the patient presents with hirsutism, moon-shaped face, stretch marks on the
abdomen. BP is 190/100 mm Hg, blood glucose is 17,6 mmol/L. What pathology is such
clinical presentation characteristic of/
a. *Adrenocortical hyperfunction
b. Hypothyroidism
c. Gonadal hypofunction
d. Hyperthyroidism
e. Hyperfunction of the insular apparatus
292. A 50-year-old inpatient during examination presents with glucosuria and blood glucose of 3,0
mmol/L, which are the most likely to be caused by:
a. *Renal disorder
b. Diabetes insipidus
c. Pellagra
d. Myxedema
e. Essential hypertension
293. Exophthalmus observed during thyrotoxicosis is caused by accumulation of highly water-
binding substances within the retrobulbar tissues. Name these substances:
a. *Glycosaminoglycans
b. Phospholipids
c. ATP
d. Cholesterol
e. Kreatine
294. The right leg of a 40-year-old woman measured at the shin level is by 2 cm smaller in the
diameter than the left leg. Ankle-jerk (Achilles) and knee-jerk reflexes are absent on the right.
What is the most likely mechanism of hyporeflexia development during peripheral paralysis?
a. *Disturbed conduction of stimulation
b. Inhibition of pyramidal motoneutron
c. Disturbed synaptic impulse transmission
d. Activation of excitatory impulses from CNS
e. Disturbed perception of stimulation
295. In human organism significant blood loss leads to decreased blood pressure, tachycardia and
weakness. Eventually the sensation of thirst appears. What hormone participates in the
development of this sensation?
a. *Angiotensin 2
b. Serotonin
c. Adrenalin
d. Cortisol
e. Dopamine
296. A 16-year-old girl presents with no hair on the pubis and in the armpits, her mammary glands
are underdeveloped, no menstruations. What hormone imbalance can it be indicative of?
a. *Ovarian failure
b. Pancreatic islet failure
c. Hyperthyroidism
d. Hypothyroidism
e. Adrenal medulla hyperfunction
297. A patient complaining of pain in the left shoulder-blade region has been diagnosed with
myocardial infarction. What kind of pain does the patient have?
a. *Radiating
b. Visceral
c. Phantom
d. Protopathic
e. Epicritic
298. A patient got a gunshot wound of hip which damaged the sciatic nerve. Any impact on the
affected limb causes severe, excruciating pain. What mechanism of pain is most likely in this
case?
a. *Causalgic
b. Reflex
c. Phantom
d. Endorphin hypofunction
e. Enkephalin hypofunction
299. As a result of a trauma a patient has developed traumatic shock that led to the following
disorders: AP is 140/90 mm Hg, Ps is 120 bpm. The patient is fussy, talkative, pale. Such
state relates to the following shock phase:
a. *Erectile
b. Latent period
c. Terminal
d. Torpid
300. As a result of a road accident a 26-year-old man is in the torpid phase of shock. Blood count:
leukocytes - 3, 2 · 109/L. What is the leading mechanism of leukopenia development?
a. *Leukocyte redistribution in the bloodstream
b. Leukopoiesis inhibition
c. Faulty release of mature leukocytes from the bone marrow into the blood
d. Leukocyte destruction in the hematopoietic organs
e. Increased excretion of the leukocytes from the organism
301. After a road accident a victim has tachycardia, arterial blood pressure 130/90 mm Hg,
tachypnoe, the skin is pale and dry, excitation of central nervous system is observed. What
shock stage is the patient most likely in?
a. *Erectile
b. Terminal
c. Torpid
d. Preshock (compensation stage)
e. Agony
302. A 27-year-old patient with injury to the neck has lost approximately 30% of the blood volume.
The patient’s condition is severe: blood pressure is 60/40 mm Hg, heart rate is 140/min.,
respiratory rate is 30/min, conscious. Characterize the condition of the patient’s circulatory
system:
a. *Hypovolemic shock
b. Cardiogenic shock
c. Collapse
d. Coma
e. Arterial hypertension
303. A 62-year-old patient has been hospitalized due to massive cerebral hemorrhage. Blood
pressure is 70/30 mm Hg, heart rate is 120/min., respiratory rate is 4/min., unconscious, no
response to external stimuli. Such condition can be determined as:
a. *Coma
b. Shock
c. Collapse
d. Stress
e. Agony
304. A victim of a traffic accident was taken to the intensive care unit. The patient is in a grave
condition that can be characterized as a severe pathologic process that leads to exhaustion
of vital functions and puts the patient into the marginal state between life and death due to
critical reduction of capillary circulation in the affected organs. The patient is in the state of:
a. *Shock
b. Coma
c. Collapse
d. Preagony
e. Agony
305. A woman aged 67, who suffered from cholecystitis for a long time, after a meal suddenly has
acute pain in the upper abdomen, nausea, vomiting. Diagnosis - acute pancreatitis. What is
the main link of the pathogenesis of this disease?
a. *Premature activation of pancreatic enzymes
b. Reduced secretion of pancreatic polypeptide
c. Enhancement of enzymes in the duodenum
d. Increased cholecystokinin level
e. Decreased enzymes in pancreatic juice
306. The patient complains of heartburn, frequent constipation. During titration of gastric juice, the
following data were obtained: total acidity - 88 t.u., total HCl - 83 t.u., free HCl - 50 t.u., bound
HCl - 33 t.u., acidic phosphates and organic acids - 5 t.u. Evaluate the condition of acidity of
gastric juice
a. *Hyperaсidity
b. Hypoacidity
c. Achylia
d. Normal
e. Hypersecretion
307. A patient with Zollinger-Ellison syndrome [pancreatic tumor] has an increased secretion,
peristalsis of the gastrointestinal tract, as well as diarrhea and peptic ulcer. Which of the
following substances, which is secreted by the specified tumor, causes this complex of
symptoms?
a. *Gastrin
b. Vasoactive intestinal peptide
c. Pepsin
d. Trypsin
e. Secretin
308. The gastrointestinal department got a 57-year-old patient suspected of having a Zolinger-
Ellison syndrome, as evidenced by a acute increase in gastrin levels in serum. What is a
disturbance of the secretory function of the stomach most likely?
a. *Hypersecretory hyperacidic
b. Hyporsecretory hyperacidic
c. Achylia
d. Hyposecretion hypoacidic
e. Hypersecretory hypoacidic
309. The patient, who has increased acidity of gastric juice, was recommended to eat boiled, not
fried meat. This is due to the fact that the mechanism of action of extractives substances
consists of:
a. *Stimulation of gastrin production by G cells
b. Irritation of taste receptors
c. Irritation of the mechanoreceptors of the oral cavity
d. Irritation of the mechanoreceptors of the stomach
e. Stimulation of secretin production in 12-type intestine
310. After the poisoning with mushrooms, the patient has a yellow coloration of the skin and sclera,
a dark color of urine appeared. What kind of pigment causes coloring urine in a hemolytic
jaundice?
a. *Stercobilin
b. Monoglycuronide of bilirubin
c. Unconjugated bilirubin
d. Verdoglobin
e. Biliverdin
311. A patient with chronic hepatitis complains of increased sensitivity to barbiturates, which she
previously tolerated without symptoms of intoxication. With a violation of which of the liver
function is this the most possible?
a. *Metabolic
b. Bile formation
c. Hemodynamic
d. hemopoietic
e. phagocytic
312. A woman 57 years after prolonged pain attack in the right subcostal area has jaundice, after
which the patient visited the doctor. There was a suspicion of the appearance of acute
calculous cholecystitis. Research of what index of a blood test represents the bile duct
obstruction?
a. *Conjugated and unconjugated bilirubin
b. Protein fractions
c. Total lipids
d. Uric acid
e. Residual nitrogen
313. The patient is 25 years old is diagnosed with chronic hepatitis. The patient complains of losing
body weight by 10 kg for 2 hours months. Objectively: the skin is dry, peeling, pale yellowish
shade, skin hemorrhage, bleeding gums. Violations of which the function of the liver shows
hemorrhage and bleeding gums:
a. *Protein-Synthesizing
b. Pigment-forming
c. Glycogen-Synthesizing
d. Detoxication
e. Depositive
314. In a patient 24 years and one and a half weeks after severe streptococcal throat infection,
facial swelling appeared, arterial pressure increased. Hematuria and proteinuria 1.2 g / L are
found. In the blood, antibodies presence and reduction of complement components are
detected. In which kidney structure the localization of immune complexes which determine
the development of nephropathy is most likely?
a. *Glomerules
b. Pyramid
c. Proximal part of tubules
d. Loop Henle
e. Distal part of tubules
315. In a woman with primary hyperparathyroidism periodic attacks of the renal colic are repeated.
An ultrasound examination showed the formation of small stones in the kidneys, the most
likely cause of which is the presence of:
a. *Hypercalcemia
b. Hyperphosphatemia
c. Hypercholesterolemia
d. Hyperuricemia
e. Hyperkalaemia
316. A 30-year –old woman developed facial edemas. Examination detected porteinuria (5.87 g/L),
hypoproteinemia, dysproteinemia and hyperlipidemia. Such combination of signs is
characteristic of:
a. *Nephrotic syndrome
b. Nephritic syndrome
c. Acute kidney failure
d. Chronic kidney failure
e. Chronic pyeolonephritis
317. A 38-year-old man, who has been suffering from systemic lupus erythematosus for 3 years,
developed diffuse renal lesions accompanied by massive edemas, marked proteinuria,
hyperlipidemia, and dysproteinemia. What is the most likely mechanism of proteinuria
development in this case?
a. *Autoimmune damage to the nephron
b. Inflammatory damage to the nephron
c. Increased blood proteins
d. Morbid affection of the urinary tracts
e. Ishemic damage to the tubules
318. A laboratory rat with chronic kidney failure presents with osteoporosis, pathologic calcification
of the inner organs and arterial hypertension. These disturbances are associated with
increased activity of the following hormone:
a. *Parathormone
b. Calcitonin
c. Thyroxin
d. Adrenaline
e. Triiodothyronine
319. A 45-year-old woman comes to her physician with complaints of excessive fatigue and
weakness. She says that these symptoms have been present for the past month. On further
questioning, she admits having lost 3 kilograms in the last 2 weeks. On physical examination,
she is a tired-appearing thin woman. Hyperpigmentation is present over many areas of her
body, most prominently over the face, neck and back of hands (areas exposed to light).
Increased production of which of hte following hormones is the most likely cause of
hyperpigmentation in this patient?
a. *Melanocyte stimulating hormone
b. Thyroid stimulating hormone
c. Growth hormone
d. Beta-lipotropin
e. Gonadotropins
320. A 54-year-old woman has a total thyroidectomy for papillary thyroid carcinoma. 11 hours after
operationshe complains of tingling around her mouth. On physical examination, the
Trosseau’s sign and Chvostek’s signs are present. Her condition rapidly deteriorates with
laryngospasm and local seizures. The surgeon suggests surgical destruction of parathyroid
gland. Which of the following is the most likely cause of this patient’s neurologic abnormality?
a. *Hypocalcemia
b. Hyponatremia
c. Hyperkalemia
d. Hyperchloremia
e. Hypophosphatemia
321. A 16-year-old girl concerned about her sexual development comes to the physician. She
mentions that she has still not had a menstrual period. However, she is otherwise a healthy
girl with no significant medical problems since birth. On physical examination, her vital signs
are stable. She does not have pubic hair and her breast is slightly elevated with areola
remaining in contour with surrounding breast. Which of the following is the most likely cause
of this abnormal physical development?
a. *Ovarian insufficiency
b. Hyperthyroidism
c. Pancreatic islet insufficiency
d. Hypothyroidism
e. Adrenal medulla hyperfunction
322. An 11-year-old girl is brought to the doctor’s office by her mother who states her daughter has
been weak with swollen face for 3 days. The mother states her daughter had always been
healthy and active until the initiation of symptoms. Upon inquiry, the girl describes a foamy
appearance of her urine but denies blood in urine, urinary frequency at night, or pain during
urination. Physical examination reveals generalized swelling of the face and pitting edema on
the lower limbs. Laboratory study shows proteinuria and microscopic hematuria. Which of the
following is the most likely cause of findings in the laboratory study of urine?
a. *Increased permeability across the glomerular capillary wall
b. Increased plasma oncotic pressure
c. –
d. Increased glomerular hydrostatic pressure
e. Increased hydrostatic pressure in Bowman’s capsule
323. A male neonate born to a 24-year-old primagravida had jaundice at 8 hours of life. The
neonate’s red blood cell type was A+, while the mother’s RBC type was 0+. Laboratory
studies revealed elevated titer of mother’s anti-A antibody, normal erythrocyte glucose-6-
phosphate and negative sickle cell test. The infant’s haemoglobin was 106 g/L. Which of the
following is the most likely cause of infant’s jaundice?
a. *Hyperbilirubinemia
b. Sickle cell disease
c. Glucose-6-phosphate dehydrogenase (G6PH) deficiency
d. Rh incompatibility
e. Decrease in haemoglobin level
324. A 38-year-old woman, who was diagnosed with systemic lupus erythematosus (SLE) 3 years
ago, comes to her physician with a complaint of facial swelling and decreased urination that
she first noticed 2 weeks ago. She currently takes azathioprine and corticosteroid. Her vital
signs show blood pressure 150/90 mm Hg, pulse – 91/min, temperature – 36.8 0C and
respiratory rate – 15/min. On physical examination, the doctor notices erythematous rash on
her face exhibiting a butterfly pattern. The laboratory studies reveal hypercholesterolemia,
hypertriglyceridemia and proteinuria. Which of the following is the most likely mechanism of
SLE’s complication in this patient?
a. *Immune complex-mediated glomerular disease
b. Acute infection of the kidney
c. Decrease in renal blood flow (ischemic nephropathy)
d. Increased plasma oncotic pressure
e. –
325. In an experiment a laboratory rat was subjected to a stress factor (electric current) which
resulted in muscular hypotonia, arterial hypotension, hypothermia and hypogltcemia in the
animal. What period of general adaptation syndrome is it?
a. *Exhaustion stage
b. Antishock phase
c. Shock phase
d. Resistance stage
e. –
326. After a prolonged attack of severe headache the patient lost mobility in his left arm and leg.
Muscle tone is increased in the affected limbs, the muscles are spasmed, spinal tendon
reflexes are acutely intensified, reflex zones are increased. What nervous system disorder
can be observed in this patient&
a. *Central paralysis
b. Peripheral paralysis
c. Flaccid paralysis
d. Reflex paralysis
e. Extrapyramidal paralysis
327. L20F A 35-year-old woman has come to her physician with chief complaint of elevated blood
pressure up to 180/100 mm Hg. She currently does not take any medication. During the
physical examination, her blood pressure is 140/80 mm Hg, heart rate is 65/min. and
temperature is 36.8°C. She has a "moon face',' hirsutism, centripetal obesity, and striae on
the skin with atrophy over the abdomen and thighs. Which of the following is the most likely
cause of this patient's condition?
a. *Cortisol-secreting adrenal adenoma
b. Pancreatic islet cells hyperfunction
c. Ovarian insufficiency
d. Hyperthyroidism
e. Hypothyroidism
328. L20F A 65-year-old woman presents to the doctor's office with shortness of breath for 1
month. She complains of easy fatigability and is concerned of being unable to keep up with
her grandchildren at the park. Over the last few weeks, she had to increase the number of
pillows under her head to sleep comfortably. She denies a cough and fever. Her medical
history includes hypercholesterolemia and hypertension. Vitals include blood pressure of
160/90 mm Hg, heart rate of 90/min, respiratory rate of 14/min. Physical examination reveals
distended jugular veins, bilateral pitting edema of the lower limbs and fine crackles at the
base of lungs. An echocardiogram reveals an ejection fraction of 40%. Which of the following
is the most likely mechanism of heart failure in this patient?
a. *Increased afterload
b. Myocardial injury
c. Abnormal impulse conduction in myocardium
d. Abnormal regulation of heart activity
329. L20F A newborn delivered at 33 weeks of gestation has a respiratory rate of 70/min and heart
rate of 148/min 2 hours after birth. He is grunting and has intercostal and subcostal
retractions. He has peripheral cyanosis as well. An immediate chest radiograph is taken
which shows a fine reticular granulation with ground glass opacity on both lungs. A
neonatologist suggests respiratory distress syndrome due to surfactant deficiency. Which of
the following is the most likely mechanism of these clinical and imaging findings?
a. *Tendency of alveoli to collapse
b. Increased lung ventilation
c. Decreased airways resistance
d. Decresed respiratory muscle work
330. L20F A 50-year-old man presents to the office with the complaint of pain in his left great toe.
The pain started 2 days ago and has been progressively getting worse to the point that it is
difficult to walk even a few steps. He adds that his left big toe is swollen and hot to the touch.
He normally drinks 2-3 glasses of red wine each day. Physical examination is notable for an
overweight gentleman (BMI of 35) in moderate pain, with an erythematous, swollen, and
exquisitely tender left great toe. A complete blood count shows: hemoglobin -120 g/L,
hematocrit - 0.45, platelets – 160•109/L, leukocytes - 8.0•109/L, segmented neutrophils –
65%, lymphocytes – 25%, eosinophils – 3%, and monocytes – 7%. Synovial fluid analysis
shows cell count of 15,000 cells/mm3 (80% neutrophils), negatively birefringent crystals
present. Elevated concentration of which of the following compounds is the most common
laboratory finding in these patients?
a. *Uric acid
b. Bilirubin
c. Cystine
d. Cholesterol
e. Urea
331. L20F A 56-year-old woman is rushed to the emergency department with sudden onset of
severe chest pain radiating to her left arm and jaw. She has a history of periodic chest pain
for which she uses nitroglycerine sublingually but today her medication did not relieve the
pain. Her blood pressure is 140/100 mm Hg, pulse – 130/min., respiratory rate – 18/min.,
temperature - 37°C. A bedside electrocardiogram shows ST-segment elevation in leads И, III,
and aVF. Blood is drawn and sent to the lab, which is positive for specific biomarker. A
diagnosis of acute inferior myocardial infarction is made, and the patient is sent to the
catheterization lab for angioplasty with stent placement. Which of the following is the most
likely specific and sensitive biomarker of cardiac injury?
a. *Troponin (cTn) I or T
b. Alanine aminotransferase (ALT)
c. Lactate dehydrogenase 5 (LDH-5)
d. Alanine aminopeptidase (AAP)
e. Lactate dehydrogenase 4 (LDH-4)
332. L20F A junior doctor that works under the physician plots a Darrow-Yannet diagram for the
patient with a history of diarrhea and vomiting for 2 days. This diagram suggests a decreased
extracellular fluid (ECF) volume without any change in the intracellular fluid (ICF) volume or
body osmolality. The doctor assumes that the secretion of renin is most likely to be stimulated
in this patient, which would consequently lead to vasoconstriction and increase in blood
pressure. Which of the following hormones is most likely responsible for this patient's
changes in blood pressure?
a. *Angiotensin II
b. Angiotensinogen
c. Epinephrine
d. Norepinephrine
333. L20F A 25-year-old man is in the middle of an ascent up a mountain, at an elevation of about
4500 meters. On his fourth day of expedition, he became short of breath and developed a
dusky blue tinge around the face and lips. A physician in the climbing party performs a
rudimentary physical exam, which reveals a respiratory rate of 30/min, blood pressure of
130/90 mm Hg, heart rate of 108/min, and temperature of 36.8°C. Which of the following is
the most likely cause of these changes?
a. *Low partial pressure of 02
b. High partial pressure of 02
c. Increased pH of the blood
d. Increased air humidity

You might also like